Trang chủ Đề thi & kiểm tra Tiếng anh Bộ 35 đề thi minh họa Tiếng Anh có đáp án chi tiết năm 2022 !!

Bộ 35 đề thi minh họa Tiếng Anh có đáp án chi tiết năm 2022 !!

Câu hỏi 4 :

A. applicant

A. other

A. smoked

Câu hỏi 6 :

We _________by a loud noise during the night.

A. other

A. smoked

Câu hỏi 7 :

A. from

A. from

A. other

A. smoked

Câu hỏi 8 :

A. More famous / popular

A. More famous / popular

A. other

A. smoked

Câu hỏi 9 :

A. nice Italian new

A. nice Italian new

A. other

A. smoked

Câu hỏi 10 :

A. have/is ringing

A. have/is ringing

A. other

A. smoked

Câu hỏi 11 :

A. because

A. because

A. other

A. smoked

Câu hỏi 12 :

A. Having finished

A. Having finished

A. other

Câu hỏi 13 :

A. marry

A. marry

A. other

Câu hỏi 14 :

A. felt

A. felt

A. other

Câu hỏi 15 :

A. mouth

A. mouth

A. other

Câu hỏi 16 :

A. go into

A. go into

A. other

Câu hỏi 17 :

A. diploma

A. diploma

A. other

Câu hỏi 18 :

A. obstacles

A. obstacles

A. other

Câu hỏi 20 :

A. businesses

A. businesses

A. other

Câu hỏi 22 :

A. bored

A. bored

A. other

Câu hỏi 26 :

A. way

A. other

Câu hỏi 29 :

A. Sound helps build and sustain relationships between films and viewers.

A. Sound helps build and sustain relationships between films and viewers.

A. other

Câu hỏi 30 :

A. score

A. score

A. other

Câu hỏi 31 :

A. It is used to distill wide-ranging and complex ideas into a viewer-friendly package.

A. It is used to distill wide-ranging and complex ideas into a viewer-friendly package.

A. other

Câu hỏi 34 :

A. the more reasonable option

A. the more reasonable option

A. other

Câu hỏi 35 :

A. investments

A. investments

A. other

Câu hỏi 36 :

A. These students are unable to persuade people to use the language they are learning.

A. These students are unable to persuade people to use the language they are learning.

A. other

Câu hỏi 37 :

A. aspects

A. aspects

A. other

Câu hỏi 38 :

A. Local languages can’t unite and create as much wealth as national languages.

A. Local languages can’t unite and create as much wealth as national languages.

A. other

Câu hỏi 44 :

A. Hardly had I arrived home when I was called immediately back to the office 10 kilometers away.

A. Hardly had I arrived home when I was called immediately back to the office 10 kilometers away.

A. other

Câu hỏi 51 :

A. for

A. for

Câu hỏi 52 :

A. more is wood pulp

A. more is wood pulp

Câu hỏi 53 :

A. a brick modern big house

A. a brick modern big house

Câu hỏi 54 :

A. played/took

A. played/took

Câu hỏi 55 :

A. in spite of

A. in spite of

Câu hỏi 56 :

A. after he had finished his homework

A. after he had finished his homework

Câu hỏi 57 :

A. Having won

A. Having won

Câu hỏi 58 :

A. change

A. change

Câu hỏi 59 :

A. sandwich

A. sandwich

Câu hỏi 60 :

A. look for

A. look for

Câu hỏi 61 :

A. break

A. break

Câu hỏi 63 :

A. revises

A. revises

Câu hỏi 65 :

A. generous and gracious

A. generous and gracious

Câu hỏi 70 :

A. which

Câu hỏi 73 :

A. Cloning technology can help cure back and neck injuries.

A. Cloning technology can help cure back and neck injuries.

Câu hỏi 74 :

A. entrance

A. entrance

Câu hỏi 75 :

A. cows

A. cows

Câu hỏi 76 :

A. reproductive cloning

A. reproductive cloning

Câu hỏi 78 :

A. materials that prevent heat loss and absorption

A. materials that prevent heat loss and absorption

Câu hỏi 79 :

A. the problems of environment destruction were not prevalent at the time

A. the problems of environment destruction were not prevalent at the time

Câu hỏi 80 :

A. They are gaining in popularity in different parts of the world

A. They are gaining in popularity in different parts of the world

Câu hỏi 81 :

A. being certified

A. being certified

Câu hỏi 82 :

A. Improving living conditions

A. Improving living conditions

Câu hỏi 83 :

A. solar panels

A. solar panels

Câu hỏi 85 :

A. Anyone

A. Anyone

C. regarded

D. its

Câu hỏi 88 :

A. The teacher insisted his students on cheating in the exam.

A. The teacher insisted his students on cheating in the exam.

Câu hỏi 95 :

A. was discovered

A. was discovered

Câu hỏi 96 :

A. at

Câu hỏi 97 :

A. good/ good

A. good/ good

Câu hỏi 98 :

A. big brick old

A. big brick old

Câu hỏi 99 :

A. am walking

A. am walking

Câu hỏi 100 :

A. Since

A. Since

Câu hỏi 101 :

A. When he will have breakfast tomorrow

A. When he will have breakfast tomorrow

Câu hỏi 102 :

A. responsibly

A. responsibly

Câu hỏi 103 :

A. go away

A. go away

Câu hỏi 104 :

A. did

A. did

Câu hỏi 105 :

A. purposes

A. purposes

Câu hỏi 106 :

A. hat

A. hat

Câu hỏi 107 :

A. look

A. look

Câu hỏi 112 :

A. much

Câu hỏi 113 :

A. expectation

Câu hỏi 114 :

A. who

Câu hỏi 115 :

A. otherwise

Câu hỏi 117 :

A. Tucking a napkin in your shirt.

A. Tucking a napkin in your shirt.

Câu hỏi 120 :

A. To assist people in learning sophisticated manners

A. To assist people in learning sophisticated manners

Câu hỏi 123 :

A. It is believed to be the most quickly warming settlement on earth.

A. It is believed to be the most quickly warming settlement on earth.

Câu hỏi 124 :

A. inappropriate

A. inappropriate

Câu hỏi 125 :

A. seaweed and kelp

A. seaweed and kelp

Câu hỏi 126 :

A. Because of the high number of calves born.

A. Because of the high number of calves born.

Câu hỏi 127 :

A. Arctic reindeer play the most important role in the Arctic ecosystems.

A. Arctic reindeer play the most important role in the Arctic ecosystems.

Câu hỏi 133 :

A. Had they finished one project, they would have started working on the next.

A. Had they finished one project, they would have started working on the next.

Câu hỏi 137 :

A. was cleaned

A. was cleaned

Câu hỏi 139 :

A. with

A. with

Câu hỏi 140 :

A. The more hard

A. The more hard

Câu hỏi 141 :

A. dirty old flannel

A. dirty old flannel

Câu hỏi 142 :

A. Are playing

A. Are playing

Câu hỏi 143 :

A. in spite of

A. in spite of

Câu hỏi 144 :

A. By the time you come 

A. By the time you come 

Câu hỏi 145 :

A. Completing

A. Completing

Câu hỏi 146 :

A. disturbed

A. disturbed

Câu hỏi 147 :

A. take over

A. take over

Câu hỏi 148 :

A. role

A. role

Câu hỏi 150 :

A. bread

A. bread

Câu hỏi 151 :

A. eye

A. eye

Câu hỏi 158 :

A. who

Câu hỏi 161 :

A. The tree was gloomy, withered and crooked.

A. The tree was gloomy, withered and crooked.

Câu hỏi 162 :

A. loaded

A. loaded

Câu hỏi 163 :

A. the four sons

A. the four sons

Câu hỏi 164 :

A. Moral lessons can come from the most unexpected and ordinary things.

A. Moral lessons can come from the most unexpected and ordinary things.

Câu hỏi 166 :

A. the Vocational and Technical schools pay more attention to academic careers than the other.

A. the Vocational and Technical schools pay more attention to academic careers than the other.

Câu hỏi 167 :

A. these schools

A. these schools

Câu hỏi 168 :

A. obscurity

A. obscurity

Câu hỏi 169 :

A. questionable

A. questionable

Câu hỏi 172 :

A. got

A. got

B. he realize

C. her

D. the presence

Câu hỏi 180 :

A. be made

A. be made

Câu hỏi 182 :

A. more and more good

A. more and more good

Câu hỏi 183 :

A. beautiful young blonde Russian

A. beautiful young blonde Russian

Câu hỏi 184 :

A. was sitting

A. was sitting

Câu hỏi 185 :

A. Because

A. Because

Câu hỏi 187 :

A. wiped out

A. wiped out

Câu hỏi 188 :

A. on your guards

A. on your guards

Câu hỏi 189 :

A. gratitude

A. gratitude

Câu hỏi 191 :

A. completely ignored

A. completely ignored

Câu hỏi 193 :

A. offer

A. offer

Câu hỏi 197 :

A. preparation

Câu hỏi 198 :

A. relate

Câu hỏi 199 :

A. whom

Câu hỏi 202 :

A. two-Child policy

A. two-Child policy

Câu hỏi 203 :

A. constant

A. constant

Câu hỏi 205 :

A. dry

A. dry

Câu hỏi 206 :

A. fresh air and water

A. fresh air and water

Câu hỏi 207 :

A. trivial

A. trivial

Câu hỏi 214 :

A. watched

A. hole

Câu hỏi 219 :

A. are destroying

A. are destroying

A. watched

A. hole

Câu hỏi 220 :

A. on

A. on

A. watched

A. hole

Câu hỏi 221 :

A. For more

A. For more

A. watched

A. hole

Câu hỏi 222 :

A. old black leather skipping

A. old black leather skipping

A. watched

A. hole

Câu hỏi 223 :

A.watched

A.watched

A. watched

A. hole

Câu hỏi 224 :

A. because

A. because

A. watched

A. hole

Câu hỏi 225 :

A. Finding

A. Finding

A. watched

A. hole

Câu hỏi 226 :

A. responsibly

A. responsibly

A. watched

A. hole

Câu hỏi 227 :

A. set off

A. set off

A. watched

A. hole

Câu hỏi 228 :

A. done

A. done

A. watched

A. hole

Câu hỏi 229 :

A. hole

A. hole

A. watched

A. hole

Câu hỏi 230 :

A. break the bank

A. break the bank

A. watched

A. hole

Câu hỏi 231 :

A. tears

A. tears

A. watched

A. hole

Câu hỏi 233 :

A. arrange

A. arrange

A. watched

A. hole

Câu hỏi 235 :

A. seriously

A. seriously

A. watched

A. hole

Câu hỏi 239 :

A. preferable

A. watched

A. hole

Câu hỏi 241 :

A. for examples

A. watched

A. hole

Câu hỏi 244 :

The word "she" in the passage refers to    .

A. watched

A. hole

Câu hỏi 245 :

A. angry about Eukhtuul's experience

A. angry about Eukhtuul's experience

A. watched

A. hole

Câu hỏi 246 :

A. After some time she will see as well as before.

A. After some time she will see as well as before.

A. watched

A. hole

Câu hỏi 247 :

A. to describe a dangerous trip

A. to describe a dangerous trip

A. watched

A. hole

Câu hỏi 250 :

A. were discussing

A. were discussing

B. its

C. of

D. during

A. watched

A. hole

Câu hỏi 254 :

A. clicked

C. raced

Câu hỏi 256 :

A.  are spoke

A.  are spoke

A. clicked

B. closed

Câu hỏi 257 :

A. out of work

A. clicked

B. closed

Câu hỏi 258 :

A. less lazy

A. less lazy

A. clicked

B. closed

Câu hỏi 259 :

A. although

A. although

A. clicked

B. closed

Câu hỏi 260 :

A. until she received the parcel

A. until she received the parcel

A. clicked

B. closed

Câu hỏi 261 :

A. Having completed

A. Having completed

A. clicked

B. closed

Câu hỏi 262 :

A. impressive.

A. impressive.

A. clicked

B. closed

Câu hỏi 263 :

A. put off.

A. put off.

A. clicked

B. closed

Câu hỏi 264 :

A. create

A. create

A. clicked

B. closed

Câu hỏi 265 :

A. leave.

A. leave.

A. clicked

B. closed

Câu hỏi 266 :

A. for good

A. for good

A. clicked

B. closed

Câu hỏi 267 :

A. take a fancy to

A. take a fancy to

A. clicked

B. closed

Câu hỏi 271 :

A. benefit

A. benefit

A. clicked

B. closed

Câu hỏi 273 :

Peter is  talking to Laura about her house.

A. clicked

B. closed

Câu hỏi 275 :

A. lots

A. clicked

B. closed

Câu hỏi 278 :

A. have no choice but stay at home to learn.

A. have no choice but stay at home to learn.

A. clicked

B. closed

Câu hỏi 283 :

A. Mary asked me to remember Darin, our childhood friend.

A. Mary asked me to remember Darin, our childhood friend.

A. clicked

B. closed

Câu hỏi 287 :

A. is repairing

A. is repairing

Câu hỏi 288 :

A. a big modern brick house

A. a big modern brick house

Câu hỏi 289 :

A. cleans

A. cleans

Câu hỏi 290 :

A. having spent

A. having spent

Câu hỏi 301 :

A. seem

A. seem

Câu hỏi 310 :

A. The

A. The

B. shrunk

C. intimate

D. approached

Câu hỏi 313 :

A. was discovered

A. was discovered

Câu hỏi 322 :

A. from

A. from

B. other

C. in that

D. their

Câu hỏi 326 :

A. on

A. on

Câu hỏi 327 :

A. the soon  / the good

A. the soon  / the good

Câu hỏi 329 :

I only applied for this business with a view to accumulating first-hand experience.

A. attaining

B. gaining

C. penetrating

D. accomplishing

Câu hỏi 332 :

Mark the letter A, B, C, or D on your answer sheet to indicate the sentence that best completes each of the following exchanges

A. Men are better at certain jobs than women.

B. I agree. This really depends on their physical strengths and preferences.

C. Women and men should cooperate with each other.

D. Men are often favoured in certain jobs.

Câu hỏi 333 :

David is talking to Mary about her hairstyle.

 David  “Your hairstyle is terrific,

Mary?”. “________________”

A. Yes, all right

B. Thanks, Cindy, I had it done last night

C. Never mention it

C. Thanks, but I’m afraid

Câu hỏi 336 :

However, I was quite surprised to see that these same women now seemed (28) _______ around me.

A. uncomfortable

B. amazing

C. exciting

D. surprising

Câu hỏi 339 :

Read the passage and mark the letter A, B, C or D to answer the questions from 31 -35.

A.The birth of the "electronic superhighway"

B. The various applications of fiber-optic technology

C. How fiberscopes have enhanced the Held of medicine

D. How sound may be transformed into light

Câu hỏi 340 :

Fiberscopes are being used to do all of the following EXCEPT  _________

A. assist in delicate surgeries

B. control operations in inaccessible areas

C. convert light pulses to electrical signals

D. transmit images

Câu hỏi 341 :

The word "inaccessible" in paragraph 2 means  __________

A. difficult to find

B. extremely small

C. hard, to reach

D. impossible to climb

Câu hỏi 342 :

It can be inferred from the passage that fiberscopes  _________

A. have more uses in industry than in medicine

B. will play a major role in the information age

C. will decrease in importance as they become more common

D. have reached the peak of their development

Câu hỏi 343 :

As used in the last sentence, the word "they" refers to  ___________

A. fiberscopes

B. light-wave communication systems

C. television programs

D. telephone calls

Câu hỏi 344 :

Read the passage and mark the letter A, B, C or D to answer the following questions from 36 - 42

Environmentalists often fear that tourists will trample all over sensitive natural resource areas, but tourism may bring the needed and only economic incentives to help drive conservation, said Bynum Boley. Ecotourism and natural resource conservation already have a mutually beneficial relationship that is ideal for creating a sustainable partnership.

"Ecotourism destinations benefit in the form of enhanced tourism competitiveness from the protection of quality natural resources," he said. "Meanwhile, the conservation of these natural resources is increasingly valued since these pristine natural resources are the foundation of the ecotourism industry and the driver of all economic benefits associated with ecotourism."

Tourism is a $7.6 trillion global industry, provides 277 million jobs and is a primary income source for 20 of the world's 48 least-developed countries. It also subsidizes environmental protection and helps protect, conserve and value cultural resources that might otherwise be undervalued by the host community, Boley said. In the newspaper, Boley and co-author Gary Green said that despite past tension between the tourism industry and environmentalists, the two should team up as allies to fight off increasing conversion of land away from its natural state, Ecotourists not only provide a boost to the economy in such places, they can also motivate landowners into keeping the environment in its natural state instead of converting it into something unsustainable. They could also influence the public perception of conservation, Boley explained, which does not often favor environmental protection.

“The public has become increasing less prone to respond to environmental messages,” he said. “Economic messages are needed in order to attract the public's interest.” Too often, Boley and Green said, unique natural resource areas are converted into urban, suburban and agricultural developments without considering their ecotourism potential. In addition to the lost ecotourism revenue, there are a host of negative environmental consequences such as biodiversity loss, water and food shortages and the land being unable to mitigate the effects of climate change. These areas are not valued for their unique attributes or the valuable natural resources they provide, Green said, “so we lose them.” Tourists have historically been seen as having a negative impact on the environment. Critics complain that they violate fragile and threatened natural environments while contributing to greenhouse gases from the increased number of flights to these exotic and often remote locales. While these criticisms are justified, Boley and Green said responsible programs promote education of ecological conservation and environmental sustainability, fostering a greater understanding and appreciation of these exotic areas.

(Adapted from https://wvvw.sciencedaily.com/ )

What is the best title for the passage?

A. How to save the environmental resources

B. Ecotourism - benefits and drawbacks

C. The consequences of ecotourism

D. The development of ecotourism

Câu hỏi 345 :

The word “pristine” is closest in meaning to ______.

A. unspoiled

B. touched

C. destroyed

D. spoiled

Câu hỏi 346 :

Which of the following statements is NOT TRUE about Tourism?

A. subsidizes environmental protection.

B. is a primary income source for 20 of the world's 48 least-developed countries.

C. only develops in industrialized nations.

D. provides 277 million jobs.

Câu hỏi 347 :

The word “perception” can be replaced by ______.

A. concept

B. overview

C. insight

D. awareness

Câu hỏi 348 :

It can be inferred from the phrase “Economic messages are needed in order to attract the public's interest.” is ______.

A. People get more interested in issues related to environment.

B. People get less interested in issues related to environment.

C. People get more interested in issues related to economy.

D. People get less interested in issues related to economy.

Câu hỏi 349 :

According to the passage, negative environmental outcomes ______?

A. have bad influence on the biodiversity.

B. boost local economy.

C. reduce the effects of climate change.

D. provide more water and food.

Câu hỏi 350 :

The word “they” in the last paragraph refers to ______.

A. critics

B. tourists

C. these areas

D. flights

Câu hỏi 352 :

Nam was studying (A) for the exam (B) while its (C) roommate was watching (D) TV.

A. was studing

B. for the exam

C. while its

D. was watching

Câu hỏi 354 :

Mark the letter A, B, C, or D on your answer sheet to indicate the sentence that is closest in meaning to each of the following questions

A. I came to this coffee shop last time.

B. It has been a long time since I last came to this coffee shop.

C. I have been to this coffee shop since then.

D. I will never return to this coffee shop.

Câu hỏi 355 :

"Let’s go for a walk. We've been working all day”, said Joanna.

A. Joanna suggested going for a walk because they had been working all day.

B. Joanna insisted on going for a walk because they had been working all day.

C. Joanna suggested going for a walk because they have been working all day.

D. Joanna suggested going for a walk because she had been working all day.

Câu hỏi 356 :

It is high time you made up your mind.

A. You ought to make up your mind now.

B. Making up your mind is necessary.

C. It is possible to make up your mind.

D. You should have made up your mind

Câu hỏi 357 :

Mark the letter A, B, C, or D on your answer sheet to indicate the sentence that best combines each pair of sentences in the following questions

A. I could have seen Susan off at the airport.

B. If only I had seen Susan off at the airport.

C. That I did not see Susan off at the airport escapes me now.

D. It suddenly dawns on me that I should have seen Susan off at the airport.

Câu hỏi 358 :

You can feel more at ease by taking part in group dating. It’s the only way.

A. By taking part in group dating can you only feel more at ease.

B. Only by taking part in group dating can you feel more at ease.

C. The only way you are by taking part in group dating can feel more at ease.

D. The way you can feel more at ease is taking part in only group dating.

Câu hỏi 364 :

My wedding ring  ………… of  white gold.

A. is made

B. is making

C. made

D. maked

Câu hỏi 366 :

The larger the apartment, the................... the rent.

A. expensive

B. more expensive

C. expensively

D. most expensive

Câu hỏi 367 :

I remember she wore a______dress to go out with her boyfriend last week.

A. cotton white Vietnamese

B. Vietnamese white cotton

C. white Vietnamese cotton

D. white cotton Vietnamese

Câu hỏi 368 :

The boy fell while he________down the stairs.

A. run

B. running

C. was running

D. runs

Câu hỏi 369 :

________ sounding very boring, the film was actually pretty interesting. I would recommend it!

A. However

B. Although

C. Despite

D. Nevertheless

Câu hỏi 370 :

She will take management training course ________.

A. as soon as the epidemic was controlled

B. right after the epidemic has been controlled

C. when the epidemic had been controlled

D. until the epidemic will be controlled

Câu hỏi 372 :

The more________ you look, the better you will feel.

A. confident

B. confide

C. confidently

D. confidence

Câu hỏi 373 :

Steve ________ his chances of passing by spending too much time on the first question.

A. threw in

B. threw off

C. threw out

D. threw away

Câu hỏi 375 :

I wasn't happy with the way my career was going so I decided to ________ and look for another job.

A. pick a fight

B. jump the gun

C. bite the bullet

D. cross swords

Câu hỏi 376 :

When I enquired about the time of trains, I was given a ________.

A. timetable

B. programme

C. itinerary

D. schedule

Câu hỏi 380 :

Drunk driving can lead to some tragic results.

A. miserable

B. peaceful

C. delighted

D. lasting

Câu hỏi 381 :

If you are at a loose end this weekend, I will show you round the city.

A. free

B. confident

C. occupied

D. Reluctant

Câu hỏi 382 :

~ Daisy: "What an attractive hair style you have got, Mary!"  ~  Mary: "........."

A. Thank you for your compliment!

B. I don't like your sayings.

C. You are telling a lie.

D. Thank you very much! I am afraid.

Câu hỏi 384 :

Read the following passage and mark the letter A, B, C, or D on your answer sheet to indicate the correct word or phrase that best fits each of the numbered blanks from 26 to 30.

     Why is it that many teenagers have the energy to play computer games until late at night but can't find the energy to get out of bed in time for school? According to a new report, today's generation of children are in danger of getting so (26) ___________sleep that they are putting their mental and physical health at risk. Adults can easily survive on seven to eight hours' sleep a night, (27) ___________teenagers require nine or ten hours. According to medical experts, one in five youngsters gets anything between two and five hours' sleep a night less than their parents did at their age.

     This (28) ___________ serious questions about whether lack of sleep is affecting children's ability to concentrate at school. The connection between sleep deprivation and lapses in memory, impaired reaction time and poor concentration is well (29) ___________Research has shown that losing as little as half an hour's sleep a night can have profound ef fects on how children perform the next day. A good night's sleep is also crucial for teenagers because it is while they are asleep (30)___________they release a hormone that is essential for their 'growth spurt' (the period during teenage years when the body grows at a rapid rate). It's true that they can, to some extent, catch up on sleep at weekends, but that won't help them when they are dropping off to sleep in class on a Friday afternoon.

A. less

B. little

C. few

D. much

Câu hỏi 389 :

Read the following passage and mark the letter A, B, C, or D on your answer sheet to indicate the correct answer to each of the questions from 31 to 35.

The increase in urbanization causes different problems. Air and water pollution are amongst the major issue we have to tackle.

In the first place, cars, factories and burning waste emit dangerous gases that change the air quality in our cities and pose threats to our health. Dangerous gases such as carbon dioxide and nitrogen oxides cause respiratory diseases, for instant, bronchitis and asthma. Those are also proved to have long-term effects on the environment.

Furthermore, with the increased population, it becomes difficult to manage the waste generated in cities. Most of the waste is discharged or dumped into rivers or onto streets. The waste pollutes water and makes it unfit for human consumption. Subsequently, it becomes more and more difficult for city dwellers to get clean water. Some cities in Africa are unable to provide adequate water supply because most of the water is lost in pipe leakages. In fact, most city dwellers in developing countries are forced to boil their water or to buy bottled water, which is very expensive.

There are several actions that could be taken to eradicate the problems described above. Firstly, a simple solution would be joining community efforts to address problems affecting your city. Ask your parents, friends and relatives to join in as well. These efforts might include clean-up campaigns, recycling projects and a signature campaign to ask the government to do something about the situation. A second measure would be encouraging your teacher to talk about these problems and to discuss how young people can help to solve them. Finally, writing to local organizations working on these issues for ideas on how you can contribute to solve them.

The word tackle in paragraph 1 is closest in meaning to ____.

A. deal with

B. make up

C. try on

D. turn down

Câu hỏi 390 :

The word those in paragraph 2 refers to ____.

A. bronchitis and asthma

B. carbon dioxide and nitrogen oxides

C. dangerous gases

D. respiratory diseases

Câu hỏi 391 :

According to the passage, in some cities in Africa ____.

A. people are allowed to dump waste into rivers and on streets

B. people aren't provided enough water due to leaking pipes

C. people have found some solutions to the problems

D. people would rather use boiling water and bottled water

Câu hỏi 392 :

Which of the following is NOT true according to the passage?

A. City problems should be taught and be topic for students to discuss at school.

B. Children must ask for their parent's permission before joining community efforts.

C. Participators might take part in different kinds of projects and campaigns.

D. People can contribute in solving the problems by writing to local organizations working on these issues.

Câu hỏi 393 :

Which of the following would serve as the best title for the passage?

A. Environment degradation: Air and water pollution

B. Environmental pollution: Problems and actions

C. Increasing urbanization: Causes and effects

D. Increasing urbanization: Effects and solutions

Câu hỏi 394 :

Read the following passage and mark the letter A, B, C, or D to indicate the answer to each of the question.

A. Urbanization – Pros and cons

B. Urbanization – How people’s health is impacted?

C. Urban cities – The new opportunity for community development

D. Developing countries – The fastest urbanization

Câu hỏi 395 :

What does the phrase “these problems” in paragraph 2 refer to?

A. industrialized countries

B. lots of jobs and living spaces

C. immigrants

D. negative physical health effects

Câu hỏi 396 :

According to paragraph 2, which of the following is TRUE about China?

A. This is a developed country in the past 30-40 years.

B. The population of cities now more than doubles that in the past.

C. Most of the new urban areas’ populations are the emigrants from remote places.

D. Because of urbanization, more immigrants find their ways to this country.

Câu hỏi 397 :

The word “congested” in paragraph 3 is closest in meaning to ___________.

A. overcrowded

B. sparse

C. contaminated

D. fresh

Câu hỏi 398 :

The word “detrimental” in paragraph 3 mostly means ___________.

A. beneficial

B. positive

C. harmful

D. dangerous

Câu hỏi 399 :

Why are urban populations easy to get diseases from food, according to the last paragraph?

A. Because of the low quality and the high proportion of sodium and sugar in this food.

B. Because the way people get this food is rather accessible, quick and easy.

C. Because this food is so delicious that people have a tendency to eat more than normal.

D. Because of the change in people’s diet.

Câu hỏi 400 :

What can be inferred from the passage?

A. Living in urban areas for a long time will not make the life expectancy of inhabitants shorten.

B. One of the negative health effects comes from the overpopulation in industrialized countries.

C. People in developed countries suffer less harmful health effects from urbanization than those in developing nations.

D. The bad health effects from urbanization are not greater than the benefits it brings to people in urban cities.

Câu hỏi 402 :

Most workers seem (A) to be happy with (B) its new (C)  working conditions (D).

A. seem

B. with

C. its new

D. conditions

Câu hỏi 403 :

The party has won (A)  a (B) historical (C) victory at (D) the polls.

A. has won

B. a

C. historical

D. at

Câu hỏi 404 :

Mark the letter A, B, C, or D on your answer sheet to indicate the sentence that is closest in meaning to each of the following questions.

A. We haven’t talked for a long time.

B. We have talked for a long time.

C. We will never talk.

D. We talked for a long time.

Câu hỏi 405 :

"Don't forget to submit your assignments by Thursday," said the teacher to the students.

A. The teacher reminded the students to submit their assignments by Thursday.

B. The teacher allowed the students to submit their assignments by Thursday.

C. The teacher ordered the students to submit their assignments by Thursday.

D. The teacher encouraged the students to submit their assignments by Thursday.

Câu hỏi 406 :

Every student is required to write an essay on the topic.

A. Every student might write an essay on the topic.

B. Every student must write an essay on the topic.

C. They require every student write an essay on the topic.

D. Every student should write an essay on the topic.

Câu hỏi 407 :

Mark the letter A, B, C, or D on your answer sheet to indicate the sentence that best combines each pair of sentences in the following questions.

A. Julian regrets not having dropped out of college after his first year.

B. Julian wishes he didn’t drop out of college after his first year.

C. If only Julian hadn’t dropped out of college after his first year.

D. Only if Julian had dropped out of college after his first year would he not regret it.

Câu hỏi 408 :

We arrived at airport. We realized our passports were still at home.

A. It was until we arrived at the airport that we realize our passports were still at home.

B. We arrived at the airport and realized that our passports are still at home.

C. Not until had we arrived at the airport we realized our passports were still at home.

D. Not until we arrived at the airport did we realize that our passports were still at home.

Câu hỏi 414 :

This flower ________by my father every morning.

A. are watered

B. is watered

C. watered

D. waters

Câu hỏi 415 :

I found her letter__________as I was looking through my files.

A. by air

B. by name

C. by accident

D. by far

Câu hỏi 416 :

Bill is ………………

A. lazier and lazier

B. more and more lazy

C. lazier and more lazy

D. more lazy and lazier

Câu hỏi 417 :

Pass me the______cups to have my sister clean them.

A. red big plastic

B. big red plastic

C. red plastic big

D. plastic big blue

Câu hỏi 418 :

We ____ to the hospital to visit Mike when he called to say that he was fine.

A. was driving

B. drove

C. were driving

D. driven

Câu hỏi 419 :

We do not need much furniture _______ the room is small.

A. although

B. despite

C. because

D. because of

Câu hỏi 420 :

They had sold out all the tickets ________ .

A. until we were arriving at the theater

B. when we arrived at the theater

C. because we have arrived at the theater

D. in case we had arrived at the theater

Câu hỏi 421 :

…. that her father was angry, she left the room quietly.

A.Feel

B. Having felt

C. to feel

D. felt

Câu hỏi 423 :

Deborah is going to take extra lessons to _______ what she missed when she was away.

A. catch up on

B. take up with

C. cut down on

D. put up with

Câu hỏi 424 :

You can't always play it safe. Sometimes you need to _______ a risk.

A. take

B. make

C. have

D. put

Câu hỏi 425 :

The talks were meant to break down ________ between the two groups.

A. gates

B. walls

C. barriers

D. fences

Câu hỏi 426 :

There's a lot more to Willie than one would think: still waters run_______

A. deep

B. deeply

C. deepness

D. depth

Câu hỏi 429 :

Darwin's theory of evolution incorporates the principle of natural selection.

A. radical revolution

B. gradual development

C. radical resolution

D. practical involvement

Câu hỏi 433 :

Mai and Tuan are talking about modern Robot

            Mai: I believe that modern robots will be more intelligent and replace humans in many dangerous jobs.

 Tuan: ____

A. That's just what I think of course.

B. That sounds interesting.

C. That's a good idea.

D. Why not? Believe me!

Câu hỏi 434 :

Read the following passage and mark the letter A, B, C, or D on your answer sheet to indicate the correct word or phrase that best fits each of the numbered blanks.

        The "greenhouse effect" is the warming that happens when certain gases in Earth's atmosphere (26) ________ heat. These gases let in light but keep heat from escaping, like the glass walls of a greenhouse. First, sunlight shines onto the Earth's surface, (27) ______ it is absorbed and then radiates back into the atmosphere as heat. In the atmosphere, “greenhouse gases trap some of this heat, and the the rest escapes into space. The more greenhouse gases are in the atmosphere, the more heat gets trapped.

        Scientists have known about the greenhouse effect since 1824, when Joseph Fourier calculated that the Earth would be much colder if it had no atmosphere. This greenhouse effect is what keeps the Earth's climate (28) _________. Without it, the Earth's surface would be an average of about 60 degrees Fahrenheit cooler. Scientists often use the term "climate change" instead of global warming. This is because as the Earth's average temperature climbs, winds and ocean currents move heat around the globe in ways that can cool some areas, warm (29) ___________, and change the amount of rain and snow falling. (30)_________, the climate changes differently in different areas.

                                   (Source: https://www.open.edu/openlearncreate/mod/oucontent)

A. seize

B. capture

C. trap

D. grasp

Câu hỏi 438 :

(30)_________, the climate changes differently in different areas.

A. However

B. In addition

C. On the contrary

D. As a result

Câu hỏi 439 :

Read the following passage and mark the letter A, B, C, or D on your answer sheet to indicate the correct answer to each of the questions from 31to 35.

            A report from the United Nations given at the conference unveils how worrying the pollution caused by plastic utensils, especially plastic bags, is becoming. In a single minute, the world consumes one million plastic bottles; in a year, the world consumes five billion single-use plastic bags, according to Vietnamplus. The scary fact is that such plastic bottles and bags are not properly treated, as each year, the world discharges 300 million tons of plastic waste, accounting for some 10% of all solid waste, putting the environment and human health under tenterhooks, according to the news site.

The conference also drew attention to another fact: “For a plastic bag that can be used for five minutes, it takes five seconds to produce, one second to discard, but 500 to 1,000 years to become totally decomposed,” says Vietnamplus.

In another message given by the UN General Secretary and delivered at the conference, it is reported that since more than eight million tons of plastic bags end up in the oceans each year, “microplastics in the seas now outnumber stars in our galaxy.” “If present trends continue, by 2050 our oceans will have more plastic than fish,” Dan Tri reports, quoting Caitlin Wiesen, country director of the UN Development Programme in Vietnam.

These above-mentioned fearful facts, however, are not merely global issues, but domestic problems as well, according to local media. Many local news outlets, when relating data from the conference, point out that white pollution – a term used to indicate the overwhelming discharge of plastic bags into the environment – is even getting worse in Vietnam than elsewhere.

(Source: https://english.thesaigontimes.vn)

Which of the following could be the main idea of the passage?

A. White pollution is getting worse and worse.

B. White pollution – a new type of pollution.

C. The facts about plastic bags are scarier than what we can see.

D. The problems caused by white pollution are increasing.

Câu hỏi 440 :

The word “tenterhooks” in the first paragraph mostly means _______________.

A. The feeling of happiness to know the good results.

B. The possibility of harm or death to someone.

C. Worry or nervousness about something that is going to happen.

D. A statement about what you think will happen in the future.

Câu hỏi 441 :

The following are the facts about white pollution, EXCEPT ___________.

A. Five billion is the number of single-use plastic bags consumed by the world population in one year.

B. Plastic waste makes up about one tenth of the solid waste on the Earth.

C. It is as effortless to decay plastic bags as to manufacture them.

D. Every year over eight million tons of plastic bags are littered in the oceans.

Câu hỏi 442 :

What does the phrase “present trends” in paragraph 3 refer to?

A. Microplastics in the seas now are much more than the stars in the sky.

B. Million tons of plastics bags are discharged into the oceans.

C. It takes 500 to 1,000 years to decompose the whole plastic bags.

D. There are more plastic bags in the oceans than fish.

Câu hỏi 443 :

What does the author imply in the last paragraph?

A. White pollution in Vietnam is almost out of control.

B. Vietnamese should solve this environmental problem themselves.

C. The state of plastic pollution in Vietnam is becoming the worst in the world.

D. To reduce pollution, every country in the world has to join hands together.

Câu hỏi 444 :

Read the following passage and mark the letter A, B, C, or D on your answer sheet to indicate the correct answer to each of the questions from 35 to 42.

In America, when dining, people consider it rude for a guest or dining partner to belch or burp, eat with an open mouth, smack, or lick your fingers. Napkins, generally provided are available at every meal and should be placed in one's lap and then used throughout the meal to clean one's fingers and mouth.

It is acceptable to refuse additional servings of food by saying "No, thank you" and the host or hostess will not be insulted if you do so. Similarly, if you leave a small amount of uneaten food on your plate at a restaurant or in a home, it is not considered an insult. If you eat everything on the plate, a host or hostess may possibly feel that they have not prepared enough food and might be embarrassed. People in the United States serve and eat food with either hand, but never take food from a communal serving dish with their hands. Generally, a serving utensil is used.

Americans typically use forks, spoons and knives to eat, but there are some types of foods that are acceptable to eat with one's fingers, like sandwiches or pizza. When in doubt, look to see what others are doing. In formal dining situations, if you wonder whether or not it is acceptable to begin eating, you should wait until the oldest woman (or oldest man if no women are present) begins to eat. When eating, do not pick up the bowl or plate from the table to hold underneath your mouth. Even noodles, soup, and rice are eaten with the plate or bowl remaining on the table. When consuming soup and hot liquids, it is considered impolite to slurp - do not do this. When consuming noodles, twirl them around your fork and then put it in your mouth.

If you are a man taking out a woman for dinner, you are almost always expected to pay. This is for the woman to gauge your intentions and interest with her. For example, taking a woman for coffee, versus tacos, versus a fancy dinner, versus for drinks at 11:30 pm, all signal many different things to them. So, the date is a "test" of many. Paying is just as important as where you take her, and how late. So, don't assume she is just trying to get a "free meal". Most girls aren't. Also, if you are going out with a friend to eat, almost always, the bill is expected to be split in half, or each person pays for themselves.

If you are eating in a restaurant, you will be expected to add a 15 to 20 % tip for the server to your bill. In America, wait staff might occasionally stop by your table to ask how your meal is, which is considered good service. They will also bring you your check when it seems reasonable that you are finished with your meal, however, this is not necessarily an indication that you must leave right away (Do not be too embarrassed to ask for the check either: waiters and waitresses cannot read minds.) Take your time to finish your meal, and unless there is a line of people waiting at the door, it is not considered rude to linger at your table for as long as you like.

(Source: https://www.tripadvisor.com/)

Which of the following could be the best title of the passage?

A. What is acceptable in dining etiquette in America?

B. Polite behavior at American’s restaurants.

C. Table etiquette in America.

D. What should we do when being invited to the American’s dinner?

Câu hỏi 445 :

According to the passage, what action may make the American unpleasant?

A. Eating food with the guests’ hands.

B. Leaving the left-overs on the plate.

C. Cleaning the food on the fingers by the tongue.

D. Refusing the supplementary food they serve.

Câu hỏi 446 :

The word "it” in paragraph 3 refers to _______.

A. soup

B. your fork

C. your mouth

D. noodles

Câu hỏi 447 :

What could the word "gauge” in paragraph 4 best be replaced by?

A. impress

B. estimate

C. determine

D. express

Câu hỏi 448 :

When will the bill be divided for the people having the meal?

A. When a girl is testing a boy.

B. When a man is having a date with a woman.

C. When people are having meals with their friends.

D. When people are eating with the elderly.

Câu hỏi 449 :

The word "linger" in the last paragraph is closest in meaning to ________.

A. appear

B. arrange

C. reserve

D. remain

Câu hỏi 450 :

Which of the following can be inferred from the passage?

A. It is impolite if you give extra money for the waiter.

B. Imitating others if you are not sure what to do at the meal is a good idea.

C. Paying meals for another may insult them.

D. Whenever you receive your bill, you should think of leaving soon.

Câu hỏi 452 :

My son says her (A) new English teacher is modest (B), generous (C) and sociable (D).

A. says her

B. modest

C. generous

D. sociable

Câu hỏi 453 :

A. dependent

B. care

C. children

D. work

Câu hỏi 454 :

Mark the letter A, B, C, or D on your answer sheet to indicate the sentence that is closest in meaning to each of the following questions

A. I visited Singapore last month.

B. I have visited Singapore twice.

C. This is the first time I have visited Singapore.

D. I had never been to Singapore before.

Câu hỏi 455 :

"Stop smoking or you'll be ill", the doctor told me.

A. The doctor advised me to give up smoking to avoid illness.

B. The doctor suggested smoking to treat illness.

C. I was ordered not to smoke to recover from illness.

D. I was warned against smoking a lot of cigarettes.

Câu hỏi 456 :

They’ll offer a sneak peek at the movie and then they'll release it a week later on 5th May.

A. They might have offered a sneak peek at the movie on 5th May.

B. They might be offering a sneak peek at the movie as soon as they release it on 5th May.

C. They will offer a sneak peek at the movie a week before they release it on 5th May.

D. They will be offering a sneak peek at the movie on 5th May.

Câu hỏi 457 :

Mark the letter A, B, C, or D on your answer sheet to indicate the sentence that best combines each pair of sentences in the following questions

A. I could have seen Tim off at the airport.

B. If only I had seen Tim off at the airport.

C. That I did not see Tim off at the airport escapes me now.

D. It suddenly dawns on me that I should have seen Tim off at the airport.

Câu hỏi 458 :

As soon as James started working, he realized that his decision had not been a good one.

A. Just before James took up his new post, he realized that he was not suited for it.

B. No sooner had James begun his new job than he knew his decision was wrong.

C. Had James not begun his new job, he would have gone looking for a better one.

D. Since James did not like his new job, he began looking for a better one.

Câu hỏi 464 :

Fiona _______to John’s birthday by him last night.

A. is invited

B. is inviting

C. invited

D. was invited

Câu hỏi 465 :

You are always asking me________the money. Ask somebody else_______a change.

A.to/ at

B. of/ for

C. for/ for

D. on/in

Câu hỏi 466 :

It is necessary for parents these days to be ______  in dealing with their children.

A. patient and patient

B. more and more patient

C. more patient and more

D. more patient and more patient

Câu hỏi 467 :

He bought a______house as a gift for his new wife.

A. big pink beautiful

B. beautiful pink big

C. big beautiful pink

D. beautiful big pink

Câu hỏi 468 :

She hurt herself while she________hide-and-seek with her friends.

A. is playing

B. had played

C. played

D. was playing

Câu hỏi 469 :

We didn’t go anywhere yesterday ________ the rain.

A. because

B. in spite of

C. because of

D. though

Câu hỏi 470 :

……., I will complete every exercise in my textbook.

A.When I master English

B. The moment I mastered English

C.As soon as I had mastered English

D. Before I am mastered English.

Câu hỏi 471 :

……the report to the manager, she decided to take a rest.

A. Having handed in

B. Handed in

C. Being handed in

D. To hand in

Câu hỏi 473 :

Everybody in the house woke up when the burglar alarm     .

A. went out

B. went off

C. came about

D. rang off

Câu hỏi 474 :

This director has _______ some famous films but I think this one is the best.

A. done

B. composed

C. made

D. conducted

Câu hỏi 475 :

The biggest fear is that humans might ________ control over robots.

A. lose

B. take

C. keep

D. gain

Câu hỏi 476 :

I won't buy that car because it has too much                           on it.

A. ups and downs

B. odds and ends

C. wear and tear

D. white lie

Câu hỏi 477 :

________ of the financial crisis, all they could do was hold on and hope that things would improve.

A. At the bottom

B. At the height

C. On the top

D. In the end

Câu hỏi 481 :

She was unhappy that she lost contact with a lot of her old friends when she went abroad to study.

A. made room for

B. put in charge of

C. lost control of 

D. got in touch with

Câu hỏi 483 :

Peter is  talking to Laura about her house.

 Peter : “What a lovely house you have!”

Laura:  _____________________________________


A. Of course not, it’s not costly.


B. Thank you. Hope you will drop in.

C. I think so.

D. No problem.

Câu hỏi 484 :

Read the passage and mark the letter A, B, C or D to complete the passage.

Some doctors think that you should drink a glass of water each morning. You should drink this water first thing before doing anything else. The (26) ______ of the water should be similar to body temperature; (27) ______ too hot nor too cold. Why should you drink this water? Water helps your body in many ways. It helps clean out your kidneys. It prepares your stomach for (28) ______, Water can also help your intestines work better. After drinking water, the intestines can more easily take out nutrients from our food. Water also helps us go to the bathroom more easily,

Scientists suggest that people (29) ______ 1,600 milliliters of water each day. But don't drink all of that water in one sitting. If you do, your kidneys will have to work much harder to eliminate it. It's better to drink some in the morning and some in the afternoon. Some people think it's better to drink between meals and not during meals. They think water dilutes the juices produced in our stomachs, (30) ______ can interfere with normal digestion. Are you drinking enough water every day? Check the color of your urine. If it is light yellow, you are probably drinking enough. If your urine is very dark yellow, you probably need to drink more water. A little more water each day could make you much healthier!

(Adapted from Reading Challenge 1 by Casey Malarcher and Andrea Janzen)

A. moisture

B. heat

C. coolness

D. temperature

Câu hỏi 486 :

It prepares your stomach for (28) ______. Water can also help your intestines work better.

A. digestion

B. digestive

C. digestible

D. digest

Câu hỏi 487 :

Scientists suggest that people (29) ______ 1,600 milliliters of water each day.

A. take out

B. take up

C. take in

D. take off

Câu hỏi 489 :

Read the following passage and mark the letter A, B, C, or D to indicate the answer to each of the question.

Accidents do not occur at random. People, eighty-five years of age and older, are twenty-two times likely to die accidentally than are children five to nine years old. The risk for native Americans is four times that for Asian-Americans and twice that for white Americans or African-Americans. Males suffer accidents at more than twice the rate of females, in part because they are more prone to risky behavior. Alaskans are more than three times as likely as Rhode Islanders to die in an accident. Texans are twenty-one times more likely than New Jerseyites to die in a natural disaster. Among the one hundred most populous counties, Kern County, California (Bakersfield), has an accident fatality rate three times greater than Summit County, Ohio (Akron).

Accidents happen more often to poor people. Those living in poverty receive inferior medical care, are more apt to reside in houses with faulty heating and electrical systems, drive older cars with fewer safety features, and are less likely to use safety belts. People in rural areas have more accidents than city or suburban dwellers because farming is much riskier than working in a factory or office and because emergency medical services are less readily available. These two factors - low income and rural residence - may explain why the south has a higher accident rate than the north.

                                                                                                                    (Source: Proficiency Reading)

Which of the following is true according to the passage?

A. Children aged five to nine face the greatest accident risk.

B. All people face an equal risk of having an accident.

C. One in every 22 people aged 85 and over will die in an accident.

D. The risk of having an accident is greater among certain groups of people.

Câu hỏi 490 :

The word “accidentally” in the passage is closest in meaning to _______.

A. in an accident

B. by chance

C. by mistake

D. without a plan

Câu hỏi 491 :

According to the passage, which of the following groups of people in America face the highest risk of having an accident?

A. Native Americans

B. Asian-Americans

C. White Americans

D. African-Americans

Câu hỏi 492 :

What does the word “that” in the passage refer to _______?

A. males

B. native Americans

C. the risk

D. African-Americans

Câu hỏi 493 :

Which of the following is NOT mentioned as a reason for a higher accident rate among the poor?

A. Little knowledge about safety.

B. Inadequate medical services.

C. Poor housing and working conditions.

D. Use of cars which incorporate fewer safety features.

Câu hỏi 494 :

Read the following passage and mark the letter A, B, C, or D to indicate the answer to each of the question.

Hibernation is typically linked to seasonal changes that limit food supplies. It is identified by metabolic suppression, a drop in body temperature and torpor- a sleep- like state- interspersed with brief bouts of wakefulness. Though certain species of fish, amphibians, birds and reptiles are known to lie dormant during cold winter months, hibernation is generally associated with mammals, according to Don Wilson, a curator emeritus of vertebrate zoology at the Smithsonian National Museum of Natural History.

Endothermic mammals- “warm- blooded” animals that generate body heat internally- need a constant energy source to keep their engines running, Wilson told Live Science. And when that energy source becomes difficult to find, hibernation can help them weather harsh conditions.

“During times of the year when that energy source is missing- especially in northern climates- one coping mechanism is to just shut down,” he said. “They’ll feed heavily during the few months when food is plentiful and build up fat, then go to sleep and live off their fat reserves”.

A special type of fat called “brown fat” accumulates in hibernating mammals, Wilson said. Bats that hibernate develop brown fat on their backs between their shoulder blades, but mammals can also store brown fat in their bellies and elsewhere in their bodies, Wilson said.

Brown fat goes a long way because the hibernating animal draws on it very slowly, reducing their metabolism to as little as 2 percent of their normal rate, according to a 2007 study published in the Journal of Neurochemistry.

Their core body temperature is also greatly reduced. It generally hovers close to the air temperature in the animal’s den but can sometimes fall as low as 27 degrees Fahrenheit (minus 3 degrees Celcius) in Arctic ground squirrels, according to Kelly Drew, a neurochemist and professor with the Institute of Artic Biology at the University of Alaska Fairbanks.

Arctic ground squirrels’ bouts or torpor last about two or three weeks, Drew told Live Science, and the animals rouse “ pretty consistently” for about 12 to 24 hours, before resuming their winter sleep. They repeat this process for up to eight months.

But even though Arctic squirrels maintain a lower body temperature than any other hibernating mammal, the changes in their bodies overall aren’t that different from those that occur in other hibernating mammals, Drew said.

“The quality of mammalian hibernation is similar from bears to hamsters to ground squirrels,” Drew said. “The distinguishing feature is how cold they get”.

The word “weather” in the second paragraph is closest in meaning to ______.

A. rely on

B. adapt to

C. involve in

D. suffer from

Câu hỏi 495 :

What is the main idea of the passage?

A. The change in body temperature during hibernation.

B. Why hibernation is not popular among mammals.

C. How hibernation works in mammals.

D. The process of hibernation in Artic ground squirrels.

Câu hỏi 496 :

Where on the hibernating bat’s body is brown fat mostly likely to be found?

A. shoulders

B. bellies

C. blades

D. backs

Câu hỏi 497 :

According to the passage, the metabolism in hibernating animals is reduced by ______.

A. 2% of the normal rate

B. half of the normal rate

C. 20% of the normal rate

D. 98% of the normal rate

Câu hỏi 498 :

What does the word “their” in paragraph 4 refer to _______?

A. bats

B. mammals

C. backs

D. blades

Câu hỏi 499 :

The word “dormant” in the first paragraph is closest in meaning to ______.

A. busy

B. inactive

C. active

D. awake

Câu hỏi 500 :

How much can the Artic ground squirrel’s body temperature reduce to?

A. Always as low as -3 degree Celsius

B. Almost the same as their den’s temperature

C. Always as low as 3 degree Celsius

D. Never lower than the polar bear’s temperature

Câu hỏi 503 :

Anyone who (A) gambles on (B) the stock exchange has to be (C) prepared to loose money (D).

A. Anyone who

B. on

C. has to be

D. to loose money.

Câu hỏi 504 :

Mark the letter A, B, C, or D on your answer sheet to indicate the sentence that is closest in meaning to each of the following questions.

A. I have played basketball three times.

B. I haven’t played basketball before.

C. I played basketball first.

D. I never played basketball.

Câu hỏi 505 :

‘No, no, you really must stay a bit longer!’ said the boys.

A. The boys denied my staying a bit longer

B. The boys refused to let me stay a bit longer.

C. The boys didn't agree to let stay a bit longer.

D. The boys insisted on my staying a bit longer.

Câu hỏi 506 :

It was a mistake of Tony to buy that house.

A. Tony couldn’t have bought that house.

B. Tony can’t have bought that house.

C. Tony needn’t have bought that house.

D. Tony shouldn’t have bought that house.

Câu hỏi 507 :

Mark the letter A, B, C, or D on your answer sheet to indicate the sentence that best combines each pair of sentences in the following questions

A. He regrets to have been so rude to them last night.

B. He regrets having so rude to them last night.

C. He wishes he hadn’t been so rude to them last night.

D. He wishes he wasn’t so rude to them last night.

Câu hỏi 508 :

Harry had packed his luggage. After that, he loaded it into the car and set off for the airport.

A. Packing his luggage, Harry loaded it into the car and set off for the airport.

B. Only after Harry had packed his luggage, did he load it into the car and set off for the airport.

C. No sooner had Harry packed his luggage when he loaded it into the car and set off for the airport.

D. Not until had Harry packed his luggage did he load it into the car and set off for the airport.

Câu hỏi 514 :

This beautiful dress ______ by Mary last weekend.

A. was bought

B. has been bought

C. was buy

D. bought

Câu hỏi 516 :

Women's employment rate is getting _____

A. high and high

B. higher and  the highest

C. high and higher

D. higher and higher

Câu hỏi 517 :

Visitors to the local museum are mostly attracted by______rocking chair.

A. an old wooden European beautiful

B. a beautiful old European wooden

C. an old beautiful wooden European

D. a wooden old beautiful European

Câu hỏi 518 :

The second-hand car Patrick bought was almost new ______ it was made in the 1995s.

A. or

B. because

C. although

D. however

Câu hỏi 519 :

Paul noticed a job advertisement while he________along the street.

A. was walking

B. would walk

C. walked

D. had walked

Câu hỏi 520 :

Will you recognize him____?

A. the moment you see him

B. until you are seeing him

C. after you will see him

D. as soon as you saw him

Câu hỏi 521 :

____in this town for a long time, Mary doesn’t want to move to another place.

A.live

B. Having lived

C. To live

D. seeing

Câu hỏi 524 :

She's been ______ gymnastics since she was 5 years old.

A. doing

B. going

C. playing

D. practicing

Câu hỏi 526 :

Experts say that another outbreak of flu epidemic is on the   .

A. cards

B. boards

C. papers

D. days

Câu hỏi 529 :

His boss has had enough of his impudence, and doesn’t want to hire him anymore.

A. agreement

B. obedience

C. rudeness

D. respect

Câu hỏi 531 :

We have to work against the clock so as to meet the deadline of the clients tomorrow.

A. work in a haste

B. work slowly

C. work as fast as possible

D. work strenuously

Câu hỏi 532 :

Mark the letter A, B, C, or D on your answer sheet to indicate the option that best completes each of the following exchanges.

A. I don't think so. You can say that again

B. I'm afraid I'm not with you. It gives me a headache

C. That's OK. As long as you like it

D. That's not true. I can't understand how you feel

Câu hỏi 533 :

Two friends Peter and Linda are talking about pets.

- Peter: “Well, cats are very good at catching mice around the house."

- Linda: “______________”

A. Nothing more to say.

B. You can say that again.

C. Yes, I hope so.

D. No, dogs are very good, too.

Câu hỏi 534 :

Read the following passage and mark the letter A, B, C, or D on your answer sheet to indicate the correct word or phrase that best fits each of the numbered blanks from 26 to 30.

The World Health Organization (WHO) is warning young people all over the world that they are also at risk from COVID-19. The WHO said young people are not exempt from catching the coronavirus. Many young people ___ (26) _____believe they will not catch the virus because of their age. They think it is a disease that only older people catch. The WHO said the truth is ____ (27) ____young people are catching the coronavirus and becoming ill or dying from it. It added that young people are also spreading the disease to their parents, grandparents and _____ (28) ___people. The White House also urged young adults to follow advice and to avoid gathering in large groups to help prevent the spread of the virus.

The Director-General of the WHO said: "Today, I have a message for young people: You are not invincible." He added: "This coronavirus could put you in hospital for weeks, or even kill you. Even if you don't get sick, the choices you make about where you go could be the difference between life and death for someone else." The WHO said: "A significant proportion of patients___ (29) _____in hospital for COVID-19 around the world are aged under 50." New York Governor Andrew Cuomo said many young people are not ___ (30) _____the state's social-distancing rules. He told young people that: "This is a public health issue and you cannot be endangering other peoples' health."

                                                                                ( Source: https://breakingnewsenglish.com/)

A.really

B. mistakenly

C.strongly

D. frequently

Câu hỏi 539 :

Read the following passage and mark the letter A, B, C, or D to indicate the answer to each of the question.

A. The ceremony that marks the beginning of Masai boys' adulthood

B. The importance of Eunoto in African people's lives

C. The journey by Masai teenage boys to the festival of Eunoto

D. The reasons for the Masai's popularity over other African tribes

Câu hỏi 540 :

The word “them” in paragraph 3 refers to _______.

A. the boys

B. different tribes

C. the senior elders

D. their mothers

Câu hỏi 541 :

The word “alter” in paragraph 4 is closest in meaning to _______.

A. change

B. differ

C. distinguish

D. maintain

Câu hỏi 542 :

According to the passage, what do the teenage boys do at Eunoto?

A. Painting their bodies

B. Receiving new names

C. Changing their clothes

D. Fighting with other tribes

Câu hỏi 543 :

According to the passage, which of the following is TRUE?

A. The Masai teenage boys are given advice from senior elders before attending Eunoto.

B. The Masai teenage boys will become adults and get married after attending Eunoto.

C. Eunoto lasts for a day across the region between Kenya and Tanzania.

D. Eunoto is the ceremony for both Masai teenage boys and girls.

Câu hỏi 544 :

Read the following passage and mark the letter A, B, C, or D to indicate the answer to each of the question.

For hundreds of years, giving flowers has been a social means of communication. In the United States, flowers are often given during rites of passage, for commemorating special occasions or as a heartfelt gift between loved ones and friends. Flower gifting also occurs in most countries around the world. However, the meanings and traditions often vary.

While students traditionally gave their favorite teacher an apple in past years, in China, teachers are given flowers. Peonies are by far the flower most often given in China. They are also quite popularly used for weddings. Strangely, potted plants are not considered a pleasant gift among Asian cultures. The people believe that like a plant confined by a pot, the gift symbolizes a binding or restriction.

In Russia, in lieu of giving birthday presents, the guest of honor receives a single flower or an unwrapped bouquet. Floral arrangements or baskets are not given. Russians celebrate a holiday known as Woman’s Day. Traditional gifts include red roses, hyacinths or tulips. When there is a funeral or other occasion where someone wishes to express sympathy, carnations, lilies or roses are given in circular configurations, which signify the transition of birth, life and death to rebirth. In this instance, the color of choice is commonly yellow. For joyous occasions, arrangements and bouquets generally contain an odd number of flowers.

In the times of ancient Rome, brides carried flowers to scare away evil spirits and encourage fertility. The Dutch believed that flowers were food for the soul. When invited to someone’s home in Great Britain, it is tradition to bring a gift of flowers. All types are acceptable except white lilies, which are usually seen at funerals. Not unlike the United States, red roses are a symbol of love. Flowers are generally gifted in odd numbered increments regardless of the occasion. However, the Brits also have superstitions regarding the number 13, so the number is avoided.

In the southern region of the continent, flowers are traditionally given during Christmas. Egyptians are much more conservative and restrict flower gifting to funerals and weddings. While certain flowers may have significant meanings for some, flowers in Las Vegas and across the United States flowers are an accepted gift for any reason desired.

                                                                                      (Source: http://www.flowersofthefieldlv.com/ )

What does the topic mainly discuss?

A. The fascinating tradition of giving flowers.

B. The different meaning of flowers in different cultures.

C. The comparison of giving flowers between Asian and European cultures.

D. The kinds of flowers people often give others in different cultures.

Câu hỏi 545 :

What does the word “They” in paragraph 2 refer to?

A. Students

B. Teachers

C. Flowers

D. Peonies

Câu hỏi 546 :

According to the passage, the following flowers are given at Woman’s Day in Russia, EXCEPT _______.

A. red roses

B. hyacinths

C. tulips

D. yellow roses

Câu hỏi 547 :

What could the word “fertility” in paragraph 4 best be replaced by?

A. fecundity

B. good spirit

C. happiness

D. loyalty

Câu hỏi 548 :

The word “superstitions” in paragraph 4 is closest in meaning to _______.

A. deep-seated belief

B. unfounded belief

C. religious belief

D. traditional belief

Câu hỏi 549 :

In which country should not people bring white lilies to other houses?

A. China

B. Russia

C. Great Britain

D. United States

Câu hỏi 550 :

It can be inferred from the passage that _______.

A. People can give flowers to the American in any occasion.

B. Egyptians are rather comfortable when receiving flowers at funerals and weddings.

C. Flowers given in Britain are in even numbers in any case.

D. At the funerals in any cultures, flowers are gifted in circular configurations.

Câu hỏi 554 :

Mark the letter A, B, C, or D on your answer sheet to indicate the sentence that is closest in meaning to each of the following questions

A. Thanh practiced swimming when he was a kid.

B. Thanh has practiced swimming since he was a kid.

C. Thanh was a swimmer when he was a kid.

D. Thanh was a swimmer as a kid.

Câu hỏi 555 :

“Don’t touch the electric wires. It might be deadly.” said Steve to Mike.

A. Steve advised Mike not touch the electric wires as it might be deadly.

B. Steve warned Mike not to touch the wires as it might be deadly.

C. Steve suggested that Mike not touch the electric wires as it might be deadly.

D. Steve accused Mike of touching the electric wires as it might be deadly.

Câu hỏi 556 :

It isn’t necessary for us to discuss this matter in great detail.

A. We should discuss this matter in great detail

B. We might discuss this matter in great detail

C. We needn’t discuss this matter in great detail

D. We mustn’t discuss this matter in great detail

Câu hỏi 557 :

Mark the letter A, B, C, or D on your answer sheet to indicate the sentence that best combines each pair of sentences in the following questions
The room was in a mess. I hope you won’t leave your clothes all over the floor.

A. I wish you wouldn’t leave your clothes all over the floor.

B. I expect you to clean the floor.

C. I regret not leaving your clothes all over the floor.

D. I wish you hadn’t left your clothes all over the floor.

Câu hỏi 558 :

We arrived at airport. We realized our passports were still at home.

A. Had we arrived at the airport, we would have realized our passports were still at home.

B. Only after we realized our passports were still at home, did we arrive at the airport.

C. Only when we realized our passports were still at home, did we arrive at the airport.

D. Not until we arrived at the airport, did we realize that our passports were still at home.

Câu hỏi 565 :

Although the queen of UK has in fact little authority of her own, she ________ informed of events, and sometimes, she ________ by the government.

A. keeps - has consulted

B. can be kept – should consult

C. is kept - is consulted

D. will keep - was consulted

Câu hỏi 566 :

My neighbor is driving me mad! It seems that _________ it is at night, _________ he plays his music!

A. the less/ the more loud

B. the later/the louder

C. the latter / the less noisy

D. the more late/ the more loudly

Câu hỏi 567 :

It is a ________.

A. blue sleeping polyester bag

B. polyester sleeping blue bag

C. blue polyester sleeping bag

D. sleeping blue polyester bag

Câu hỏi 568 :

When I got home I found that water _______ down the kitchen walls.

A. ran

B. was running

C. has run

D. had been running

Câu hỏi 569 :

She was able to finish university _______ her financial difficulties.

A. although

B. because

C. despite

D. because of

Câu hỏi 570 :

________ all the papers already, Sarah put them back in the file.

A. To have photocopied

B. To photocopy

C. Photocopy

D. Having photocopied

Câu hỏi 571 :

_____, we will be able to leave for the airport.

A. After the taxi had arrived

B. As soon as the taxi arrives

C. Until the taxi will arrive

D. When the taxi arrived

Câu hỏi 573 :

I thought the party                     really well.

A. came off

B. came up

C. came about

D. came across

Câu hỏi 575 :

I had all the information at my before attending the meeting.

A. fingertips

B. thumbs

C. hands

D. fingers

Câu hỏi 579 :

Trees have to be pruned seasonally or annually to ensure that they continue to bear fruit.

A. harvested

B. fertilized

C. trimmed

D. Weeded

Câu hỏi 581 :

There is no excuse for your discourtesy. Think twice before you are going to say anything.

A. bravery

B. politeness

C. impoliteness

D. boldness

Câu hỏi 587 :

There is also (29) ___________ evidence that texting has affected literacy skills.

A. indisputable

B. indisputably

C. dispute

D. disputation 

Câu hỏi 589 :

Read the following passage and mark the letter A, B, C, or D on your answer sheet to indicate the correct answer to each of the questions from 31 to 35.

On March 15, Dunes View Middle School held a contest for school bands. Student bands tried  out for the opportunity to perform at the school picnic, which will be held at the end of June. The winner of the contest was the band called Four Square. "We're very proud that we won the contest  and are excited to perform at the picnic," says Peter Zandt, who plays the guitar in the band. "And     since we hope to perform someday at other local places, like restaurants and parks, this will be  a  great first step."

The contest was the creation of music teacher Mr. Lopez and drama teacher Ms. Cho. The two thought of the idea while discussing recent years' school picnics. "The picnic is one of the biggest events of the year, but it has become a bit formulaic ," said Ms. Cho. "The activities are the same every year. We thought that a performance by a student band would make the school picnic more interesting and fun." Mr. Lopez, Ms. Cho, and three other teachers judged the contest, which took place  in the gym. Eight student bands signed up to audition. The bands varied in their musical forms: there  were several rock bands, a folk band, and even a jazz band. "I'm disappointed that my band didn't win,

but I think the judges made the right choice," says student Marisol Varga, a member of the folk trio called The Bell Girls. "Four Square is really excellent."

To see if the bands could present a wide range of musical skills, the teachers asked them  each to prepare two songs: one song with original words, and another in which students played instrumental music only. The judges finally chose the band Four Square as the winner of the contest. Four Square is a rock band with an unusual twist: it includes a violin player! The members of Four Square write their own songs and practice three times a week after school. Students and teachers agreed that the band competition was a  big success. All are looking forward  to the school picnic in June.

(Source: https://goo.g1/SZazhm)

Which would be the most appropriate headline for the article?


A. Famous Band Visits School


B. Teachers Give Music Lessons

C. Students Have Fun at School Picnic

D. School Holds Student Band Contest

Câu hỏi 590 :

What is probably true about school picnics in recent years?

A. They were held in March.

B. They were not very exciting.

C. They were very hard to organize.

D. They included musical performances.

Câu hỏi 591 :

The word formulaic is closest in  meaning to________________ _.

A. expensive

B. uncertain

C. long

D. dull

Câu hỏi 592 :

What type of band is NOT mentioned in the article?

A. Pop

B. jazz

C. Folk

D. Rock

Câu hỏi 593 :

Why did the teachers ask each band to play two songs?

A. To decide which songs to play at the picnic

B. To see if the bands could play a variety of songs

C. To make sure the contest did not go on too long

D. To make the concert more enjoyable for students

Câu hỏi 594 :

Read the following passage and mark the letter A, B, C, or D on your answer sheet to indicate the correct answer to each of the questions from 36 to 42.

Translators and interpreters for tech jobs of the future are expected to be one of the fastest growing occupations in the nation, according to a just released survey by Vietnamworks. Almost all positions for programmers, application developers, database and network administrators, engineers, designers, architects, scientists, technicians, and tech support will require bilingual or multilingual fluency. In just the last two years the demand for tech professionals with foreign language skills has increased more than two and one-half fold, said the survey, and the uptick shows no signs of abating anytime soon. Roughly 400,000 jobs are expected to open for interpreters (who focus on spoken language) and translators (who focus on written language) in the tech segment, between 2017 and 2020, says Tran Anh Tuan. Tuan, who works for the Centre for Forecasting Manpower Needs and Labour Market Information in Ho Chi Minh City doesn't include other industries in his prediction,

which are also recruiting ferociously for more people with these same language skills.

While that claim might seem a bit overblown (and amounts to little more than a guess by Tuan), it is clear that innovative technologies like robotics, 3D printing, drones, artificial intelligence   and virtual reality will create major upheavals in all sorts of labor markets, not just technology over     the next few years. In the last month alone, most every job posted on employment websites throughout Vietnam included the word bilingual. Far higher salaries go to people who work in high    tech positions and can speak a foreign language such as English in addition to Vietnamese, says Tran Quang Anh from the Posts and Telecommunications Institute of Technology.

Unfortunately, the surveys show that most graduating Vietnamese students are unable to do more than understand a few basic phrases of foreign languages, and practically none of them can speak any foreign language coherently, The good paying jobs with high salaries and benefits are  only available to  translators and interpreters who specialize in  high tech jobs, says Anh. But it's  not just English— graduates are needed with fluency in  middle eastern languages like Arabic, Farsi and Pashto (Afghani) as well as German, Japanese and Korean to name just a few. Spanish is  also in high demand in Vietnam, primarily because it is the second most common language in the US after English.

A recent tech expo in Hanoi sponsored by Vietnamworks and the Navigos Group attracted nearly 4,000 young tech graduates and recruiters from 14 leading companies looking to fill job  vacancies with skilled bilingual workers. The job applicants were young and industrious, said the recruiters. However, missing were candidates with the requisite language skills and most  lacked  basic 'soft skills' such as written and verbal communication abilities to effectively communicate  even in their native Vietnamese language.

Notably, the recruiters said they considered language abilities and soft skills just as, if not more important, than academic ability. Yet virtually all the prospective academically qualified employees lacked even the most basic of interpersonal communication abilities.

(Source: http://engiiskvov.vni)

Which of the following could be the main idea of the passage?

A. The most popular jobs in Vietnam's job markets.

B. The necessity of foreign languages in most tech jobs.

C. The skills needed in tech jobs nowadays.

D. Vietnamese students are not aware of the importance of learning foreign languages.

Câu hỏi 595 :

According to the passage, interpreters and translators are described as the jobs that               .

A. are decreasing dramatically in the number of employees.

B. there are not enough employees for technology companies to recruit.

C. the requirements have risen considerably and steadily.

D. are expected to experience a downward trend in the near future.

Câu hỏi 596 :

The word "upheavals" in paragraph 3 is closest in meaning to____________________.

A. upward displacements

B. upward disruptions

C. downward changes

D. downward problems

Câu hỏi 597 :

Which of the following is TRUE about employment in Vietnam according to Tran Quang Anh from the Posts and Telecommunications Institute of Technology?

A. People whose major in foreign languages is high tech often earn high salaries.

B. The demand of interpreters and translators in the workforce is not as much as what people believe.

C. Not only technology but also other jobs are putting more pressure on language skills.

D. Foreign languages in Vietnam are only needed in the posts on websites.

Câu hỏi 598 :

What does the word "them" in paragraph 4 refer to?

A. foreign languages

B. a few basic phrases

C. Vietnamese students

D. the surveys

Câu hỏi 599 :

The word "requisite" in paragraph 5 could best be replaced by________________ _.

A. shortage

B. necessity

C. specification

D. measurement

Câu hỏi 600 :

What can be inferred from the passage?

A. Academic ability is much more important than foreign languages and soft skills.

B. To get a good job nowadays, applicants must know more than a language and be good at other soft skills.

C. Written and verbal communication abilities determine the opportunity of applicants to get a desirable job.

D. Translators and interpreters are suitable for any position in the work markets.

Câu hỏi 604 :

Conan said to me, “If I were you, I would read different types of books in different ways.”


A. Conan ordered me to read different types of books in different ways.


B. I said to Conan to read different types of books in different ways to me.

C. I read different types of books in different ways to Conan as he told me.

D. Conan advised me to read different types of books in different ways.

Câu hỏi 605 :

Every student is required to write an essay on the topic.

A. Every student might write an essay on the topic.

B. Every student must write an essay on the topic.

C. They require every student can write an essay on the topic.

D. Every student should write an essay on the topic.

Câu hỏi 606 :

Mark the letter A, B, C, or D on your answer sheet to indicate the sentence that best combines each pair of sentences in the following questions

A. Lan wishes she had applied for the job in the library.

B. Lan wishes she hadn’t applied for the job in the library.

C. Lan wishes she would apply for the job in the library.

D. Lan wishes she applies for the job in the library.

Câu hỏi 607 :

Helen wrote a novel. He made a cowboy film, too

A. Helen wrote not only a novel but also made a cowboy film.

B. Helen both wrote a novel as well as made a cowboy film.

C. Helen either wrote a novel or made a cowboy film.

D. Not only did Helen write a novel but she also made a cowboy film.

Câu hỏi 613 :

Harry Potter and the Goblet of Fire ________ by J K Rowling.

A. written

B. was written

C. wrote

D. were written

Câu hỏi 615 :

The richer you are, _______.

A. you may become more worried

B. you more worried may become

C. the more worried you may become

D. the more worry you may become

Câu hỏi 616 :

She bought a ________ jacket as a present for her daughter.

A. new Italian leather

B. new leather Italian

C. leather new Italian

D. leather Italian new

Câu hỏi 617 :

Jane _______ her homework when her friend asked her to do his as well.

A. did

B. would do

C. was doing

D. had done

Câu hỏi 619 :

_______ , she will go to sleep.

A. By the time she finished the homework

B. After she was finishing the homework

C. When she finished the homework

D. As soon as she has finished the homework

Câu hỏi 620 :

________ repairing the car, he took it out for a road test.

A. Finish

B. To finish

C. Having finished

D. Finished

Câu hỏi 621 :

The Games really became a festival that impressed sports    .

A. enthusiasts

B. enthusiasm

C. enthuse

D. enthusiastic

Câu hỏi 624 :

He applied for a teaching____________ at Bales University with great confidence.

A. employment

B. post

C. work

D. career

Câu hỏi 625 :

Someone is going to have to take responsibility for this disaster. Who is going to                         ?

A. foot the bill

B. carry the can

C. hatch the chicken

D. catch the worms

Câu hỏi 628 :

He decided not to buy the fake watch and wait until he had more money.

A. cheap

B. unattractive

C. counterfeit

D. genuine

Câu hỏi 638 :

Read the following passage and mark the letter A, B, C, or D to indicate the answer to each of the question.

A. the rise in solo living

B. figures about solo dwellers in the US

C. the increase in divorce

D. solos have tendency to live in small houses

Câu hỏi 639 :

Which statement is probably TRUE according to the information in paragraph 1?

A. From the beginning of the last century, people married at young age and only death separated them.

B. Until the second half of the last century, a large number of people married young and parted only at death.

C. From the beginning of last century, people have begun settling down as singletons.

D. Until the second half of the last century, people divorce, and stay single for years or decades

Câu hỏi 641 :

In the 2rd paragraph, the writer suggests that                                                .

A. The elderly take up for a large number of the total solo dwellers

B. In the US, more male choose to live alone than female

C. Solos dwellers have helped to renew cities as they seem to spend money, socialize and take part in public life

D. There are fewer middle-aged solos than the elderly who live alone

Câu hỏi 643 :

Read the following passage and mark the letter A, B, C, or D to indicate the answer to each of the question.

Educating children at home as an alternative to formal education is an option chosen by families in many parts of the world. The homeschooling movement is popular in the United States, where close to one million Children are educated at home. In Canada, 1 percent of school-age children are homeschooled, and the idea also enjoys growing popularity in Australia, where 20,000 families homeschool their children. The movement is not limited to these countries. Homeschooling families can be found all over the world, from Japan to Taiwan to Argentina to South Africa.

Homeschooling is not a novel idea. In fact, the idea of sending children to spend most of their day away from home at a formal school is a relatively new custom. In the United States, for example, it was not until the latter part of the nineteenth century that state governments began making school attendance compulsory. Before that, the concept of a formal education was not so widespread. Children learned the skills they would need for adult life at home from tutors or their parents, through formal instruction or by working side by side with the adults of the family.

In the modern developed world, where the vast majority of children attend school, families choose homeschooling for a variety of reasons. For people who live in remote areas, such as the Australian outback or the Alaskan Wilderness, homeschooling may be their only option. Children who have exceptional talents in the arts or other areas may be homeschooled so that they have more time to devote to their special interests. Much of the homeschooling movement is made up of families who, for various reasons, are dissatisfied with the schools available to them. They may have a differing educational philosophy, they may be concerned about the safety of the school environment, or they may feel that the local schools cannot adequately address their children's educational needs. Although most families continue to choose a traditional classroom education for their children, homeschooling as an alternative educational option is becoming more popular.

Which of the following could be the main idea of the passage?

A.  A new form of school: Homeschooling

B.  Homeschool option: a common form of education all over the world.

C.  The reasons why children should be educated at home.

D.  The origin of Homeschooling.

Câu hỏi 644 :

What does the word “that” in paragraph 2 refer to?

A. the second half of the 19th century

B. the beginning of the 19th century

C. the former part of the 19th century

D. the end of the 19th century

Câu hỏi 645 :

The word "widespread" in paragraph 2 mostly means               .

A. uncommon

B. customary

C. exceptional

D. prevalent

Câu hỏi 646 :

According to the passage, the following are true about the Homeschooling, EXCEPT                  .

A. Many families in both developed and developing countries choose to educate their children at home.

B.  Parents or tutors were the ones who taught the children necessary skills in society.

C.  People got familiar with school attendance before choosing to learn at home.

D.  Before modern times, most students did not attend the school.

Câu hỏi 647 :

As mentioned in the last paragraph, children in rural areas                                          .

A.  have no choice but stay at home to learn.

B.  prefer to improve their extraordinary interests.

C.  are not contented with the philosophy of the schools available.

D.  believe that their needs to study is more than what a normal school can provide.

Câu hỏi 648 :

It can be inferred from the last passage that                                       .

A.  parents’ satisfaction plays an important role in the number of students attending class.

B.  teachers’ qualifications may be one of the reasons why students come to school.

C.  not many children in modern society are allowed to be educated at home.

D.  some schools are unable to provide a safe environment for their students.

Câu hỏi 649 :

The word “adequately” in paragraph 3 is closest in meaning to                            .

A. correctly

B. applicably

C. sufficiently

D. inappropriately

Câu hỏi 651 :

They are having (A) his(B) house painted (C) by a construction (D) company.

A. having

B. his

C. painted

D. construction

Câu hỏi 653 :

Mark the letter A, B, C, or D on your answer sheet to indicate the sentence that is closest in meaning to each of the following questions.
The last time she came back to her hometown was 4 years ago.

A. It's 4 years since she last lived in her hometown.

B. She didn't come back to her hometown 4 years ago.

C. She started coming back to her hometown 4 years ago.

D. She hasn’t come back to her hometown for 4 years.

Câu hỏi 654 :

"Don't forget to submit your assignments by Thursday," said the teacher to the students.

A. The teacher reminded the students to submit their assignments by Thursday.

B. The teacher allowed the students to submit their assignments by Thursday.

C. The teacher ordered the students to submit their assignments by Thursday.

D. The teacher encouraged the students to submit their assignments by Thursday.

Câu hỏi 655 :

I’m sure that they had practiced hard for the games as they won a lot of medals.

A. They couldn’t have practiced hard for the games as they won a lot of medals

B. They must have practiced hard for the games as they won a lot of medals

C. They shouldn’t have practiced hard for the games as they won a lot of medals

D. They might have practiced hard for the games as they won a lot of medals.

Câu hỏi 656 :

Mark the letter A, B, C, or D on your answer sheet to indicate the sentence that best combines each pair of sentences in the following questions

He broke up with her but now he thinks that was a mistake.

A. He wishes he hadn't broken up with her.

B. He wishes not to have broken her up.

C. He regrets having not broken up with her.

D. He was mistaken with breaking up with her.

Câu hỏi 657 :

I had just arrived home. I was called immediately back to the head office 10 kilometers away

A. Hardly had I arrived home when I was called immediately back to the office 10 kilometers away.

B. Because I had just arrived home, I was called immediately back to the office 10 kilometers away.

C. Should I had just arrived home, I was called immediately back to the office 10 kilometers away.

D. Whenever I had just arrived home, I was called immediately back to the office 10 kilometers away.

Câu hỏi 663 :

Most of the e-mail accounts at our company ________ by a virus last week.

A. have been affected

B. were affected

C. affected

D. was affected

Câu hỏi 665 :

The more I concentrate, _______ my answers are.

A. the more accurate

B. the most accurate

C. more accurate

D. the accurater

Câu hỏi 666 :

I bought a _________ car.

A. modern blue American racing

B. blue modern American racing

C. modern racing American blue

D. modern blue racing American

Câu hỏi 667 :

While I _______ at the bus stop, three buses went by in the opposite direction.

A. was waiting

B. waited

C. had waited

D. were waiting

Câu hỏi 669 :

_______, leave it in the hospital for someone else to read.

A. When you have read the book

B. After you had read the book

C. Before you are reading the book

D. While you were reading the book

Câu hỏi 670 :

________ all his money, Daniel couldn't afford a new jacket.

A. Spent

B. Having spent

C. Having been spent

D. To spend

Câu hỏi 671 :

High intelligent machines can be automated to operate without human ____.

A. intervene

B. intervening

C. intervention

D. interventionist

Câu hỏi 672 :

After graduating from university, I want to ____ my father's footsteps.

A. follow in

B. succeed in

C. go after

D. keep up

Câu hỏi 673 :

After marriage, Mrs. Hoa always keeps a good _____ with her mother-in-law.

A. association

B. relation

C. friendship

D. relationship

Câu hỏi 675 :

It was such a sad film that we all were reduced to … …at the end.

A. arms

B. hands

C. tears

D. heads

Câu hỏi 676 :

The baby can't even sit up yet, ______stand and walk!

A. but for

B. let alone

C. all but

D. rather than

Câu hỏi 682 :

Linda is thanking Daniel for his birthday present.
- Linda: "Thanks for the book. I've been looking it for months." 
- Daniel: " ____________”

A. Thank you for looking for it.

B. You can say that again.

C. I'm glad you like it.

D. I like reading books.

Câu hỏi 683 :

Reading the following passage and mark the letter on your answer sheet to indicate the correct word that best fits each of the numbered blank from 34 to 38 

      Father's Day was created to complement Mother's Day. Like Mother's Day (26) ______ honors mothers and motherhood, Father's Day celebrates fatherhood and paternal bonds; it highlights the (27) _______ of fathers in society. Many countries celebrate it on the third Sunday of June, but it is also celebrated widely on other days. Historically, Sonora Smart Dodd was the woman behind the celebration of male parenting. Her father, the Civil War veteran William Jackson Smart, was a single parent who (28) _______ his six children there. After hearing a sermon about Jarvis' Mother's Day in 1909, she told her pastor that fathers should have a similar holiday honoring them. Although she initially suggested June 5, her father's birthday, the pastors did not have enough time to prepare their sermons, and the celebration was deferred to the third Sunday of June. The first celebration was in Spokane. Washington at the YMCA (Young Men's Christian Association) on June 19,1910. Since then it has become a traditional day (29) ________ year. 

      In recognition of what fathers do for their families, on this day people may have a party celebrating male parenting or simply make a phone call or send a greeting card. (30) _________, schools help children prepare handmade gifts for their fathers many days before the celebration.

A. which

B. who

C. where

D. when

Câu hỏi 688 :

Read the following passage and mark the letter A, B, C, or D indicate the correct answer to each of the questions.

      In the near term, the goal of keeping AI’s impact on society beneficial motivates research in many areas, from economics and law to technical topics such as verification, validity, security and control. Whereas it may be little more than a minor nuisance if your laptop crashes or gets hacked, it becomes all the more important that an AI system does what you want it to do if it controls your car, your airplane, your pacemaker, your automated trading system or your power grid. Another short-term challenge is preventing a devastating arms race in lethal autonomous weapons.

       In the long term, an important question is what will happen if the quest for strong AI succeeds and an AI system becomes better than humans at all cognitive tasks. Such a system could potentially undergo recursive self-improvement, triggering an intelligence explosion leaving human intellect far behind. By inventing revolutionary new technologies, such a superintelligence might help us eradicate war, disease, and poverty, and so the creation of strong AI might be the biggest event in human history. Some experts have expressed concern, though, that it might also be the last, unless we learn to align the goals of the AI with ours before it becomes superintelligent.

      There are some who question whether strong AI will ever be achieved, and others who insist that the creation of superintelligent AI is guaranteed to be beneficial. At FLI we recognize both of these possibilities, but also recognize the potential for an artificial intelligence system to intentionally or unintentionally cause great harm. We believe research today will help us better prepare for and prevent such potentially negative consequences in the future, thus enjoying the benefits of AI while avoiding pitfalls.

(source: https://www.beyondteaching.com/)

Which best serves as the title for the passage?

    


A. Transcending complacency on superintelligent machines.


B. No time like the present for AI safety work.

C. The importance of researching AI safety.

D. Challenges to ensuring the safety of AI systems.

Câu hỏi 689 :

According to paragraph 1, what is NOT mentioned as a potential benefit of AI?

A. Facilitating the identification system.

B. Managing the electricity in homes.

C. Controlling vehicles and gadgets.

D. Curing life-threatening diseases.

Câu hỏi 690 :

The word “recursive” in paragraph 2 is closest in meaning to ______.

A. powerful

B. repeated

C. impossible

D. monotonous

Câu hỏi 691 :

The word “it” in paragraph 2 refers to _______.

A. event

B. human

C. history

D. poverty

Câu hỏi 692 :

According to paragraph 3, which information is incorrect about the future of AI?

A. We cannot deny the likelihood of AI turning on its creator – the human.

B. The emergence of superintelligence will yield valuable benefits for human.

C. One of the focus of AI system nowadays should be preparation and preventive measures.

D. The probability of malicious artificial intelligence is an unexpected zero.

Câu hỏi 693 :

Read the following passage and mark the letter A, B, C, or D to indicate the correct answer to each of the questions.

         People are living longer and, in some parts of the world, healthier lives. This represents one of the crowning achievements of the last century, but also a significant challenge. Longer lives must be planned for. Societal aging may affect economic growth and lead to many other issues, including the sustainability of families, the ability of states and communities to provide resources for older citizens, and international relations. The Global Burden of Disease, a study conducted by the World Health Organization, predicts a very large increase in age-related chronic disease in all regions of the world. Dealing with this will be a significant challenge for all countries’ health services.

      Population aging is driven by declines in fertility and improvements in health and longevity. In more developed countries, falling fertility beginning in the early 1900’s has resulted in current levels being below the population replacement rate of two live births per woman. Perhaps the most surprising demographic development of the past 20 years has been the pace of fertility decline in many less developed countries. In 2006, for example, the total fertility rate was at or below the replacement rate in 44 less developed countries.

       One central issue for policymakers in regard to pension funds is the relationship between the official retirement age and actual retirement age. Over several decades in the latter part of the 20th century, many of the more developed nations lowered the official age at which people become fully entitled to public pension benefits. This was propelled by general economic conditions, changes in welfare philosophy, and private pension trends. Despite the recent trend toward increased workforce participation at older ages, a significant gap between official and actual ages of retirement persists. This trend is emerging in rapidly aging developing countries as well. Many countries already have taken steps towards much-needed reform of their old-age social insurance programs. One common reform has been to raise the age at which workers are eligible for full public pension benefits. Another strategy for bolstering economic security for older people has been to increase the contributions by workers. Other measures to enhance income for older people include new financial instruments for private savings, tax incentives for individual retirement savings, and supplemental occupational pension plans.

       As life expectancy increases in most nations, so do the odds of different generations within a family coexisting. In more developed countries, this has manifested itself as the ‘beanpole family,’ a vertical extension of family structure characterized by an increase in the number of living generations within a lineage and a decrease in the number of people within each generation. As mortality rates continue to improve, more people in their 50’s and 60’s will have surviving parents, aunts, and uncles. Consequently, more children will know their grandparents and even their great-grandparents, especially their great-grandmothers. There is no historical precedent for a majority of middle-aged and older adults having living parents.

(Adapted from https://www.ielts-mentor.com)

Which of the following best describes the main purpose of the author in the passage?

A. To present a synopsis of the causes and effects of the aging population.

B. To provide an overview of the drawbacks of the world’s aging population.

C. To suggest some effective solutions to deal with the rapid increase of the aging population.

D. To prove the significant contribution of the aging population to the world economy.

Câu hỏi 694 :

As mentioned in the first paragraph, the following aspects will be influenced by the aging in the society, EXCEPT ___________.

A. the stable progress of families

B. social welfare

C. medical care

D. technology achievements

Câu hỏi 695 :

What can be inferred from the second paragraph?

A. The birthrate in developed countries is less than that in developing nations.

B. The increase in death rate led to the shortage of laborers in developed countries.

C. The fertility of the world population is not enough to substitute the elderly.

D. The improvement in medical care is one of the reasons for falling fertility.

Câu hỏi 696 :

What does the word “this” in paragraph 3 refer to?

A. public pension benefits

B. the reduction of the official age entirely pensioned

C. the relationship between the official retirement age and actual retirement age

D. central issue

Câu hỏi 697 :

The word “eligible” in paragraph 3 is closest in meaning to __________.

A. entitled

B. unqualified

C. mature

D. devoted

Câu hỏi 698 :

According to paragraph 3, which of the following is the measure of old-age social insurance programs?

A. The decrease in the age the elderly can receive pension.

B. The encouragement the elderly to contribute more in economy.

C. Enacting many policies to increase their income before retirement.

D. Widening the gap between official and actual ages of retirement.

Câu hỏi 699 :

The word “manifested” in the last paragraph could be best replaced by ________.

A. illustrated

B. demonstrated

C. proved

D. recognized

Câu hỏi 703 :

Mark the letter A, B, C, or D on your answer sheet to indicate the sentence that is closest in meaning to each of the following questions
He started working as a bank clerk 3 months ago.

A. He last worked as a bank clerk 3 months ago.

B. He didn't work as a bank clerk 3 months ago.

C. He has been working as a bank clerk for 3 months.

D. The last time she worked as a bank clerk was 3 months ago.

Câu hỏi 704 :

"No, I won’t go to work, at the weekend," said Sally.

A. Sally promised to go to work at the weekend.

B. Sally refused to go to work at the weekend.

C. Sally apologized for not going to work at the weekend.

D. Sally regretted not going to work at the weekend

Câu hỏi 705 :

I must prepare the meals for my family everyday.

A. I am required to prepare the meals for my family everyday.

B. I will prepare the meals for my family everyday.

C. I am able to prepare the meals for my family everyday.

D. I don’t have to prepare the meals for my family everyday.

Câu hỏi 706 :

Mark the letter A, B, C, or D on your answer sheet to indicate the sentence that best combines each pair of sentences in the following questions
I regret not going to the airport to see her off.

A. I wish I had gone to the airport to see her off.

B. If only I would go to the airport to see her off.

C. I regret to go to the airport to see her off.

D. If I were you, I would go to the airport to see her off.

Câu hỏi 707 :

We couldn't solve the problem until our teacher arrived.

A. Not until we solved the problem could our teacher arrive.

B. When our teacher arrived, we solved the problem.

C. Until our teacher arrived, we were able to solve the problem.

D. Not until our teacher arrived could we solve the problem.

Câu hỏi 713 :

English ________ at state schools as a compulsory lesson for 20 years until last year.

A. is being taught

B. was being taught

C. had been taught

D. has been taught

Câu hỏi 714 :

They were deeply shocked ______ her rebellious behavior.

A. for

B. with

C. to

D. at

Câu hỏi 715 :

_______ you begin to learn a language, the easier it is.

A. The younger

B. Younger

C. The youngest

D. Youngest

Câu hỏi 716 :

Despite giving birth to three children, Joana is such a _________ lady.

A. beautiful tall young

B. beautiful young tall

C. tall beautiful young

D. young tall beautiful

Câu hỏi 717 :

Linda took great photos of butterflies while she _______ in the forest.

A. was hiking

B. is hiking

C. hiked

D. had hiked

Câu hỏi 718 :

_____ the sky was grey and cloudy, we went to the beach.

A. Because

B. Although

C. Despite

D. Because of

Câu hỏi 719 :

As soon as I _____, I’m going to return to my hometown.

A. have graduated

B. will graduate

C. will have graduated

D. am going to graduate

Câu hỏi 720 :

________ hard all day, I was exhausted.

A. To work

B. Work

C. Having worked

D. worked

Câu hỏi 721 :

These quick and easy _______ can be effective in the short term, but they have a cost.

A. solve

B. solvable

C. solutions

D. solvability

Câu hỏi 722 :

The Japanese market _________ 35 per cent of the company's revenue.

A. lets in

B. accounts for

C. cares for

D. takes in

Câu hỏi 723 :

The biggest fear is that humans might _________ control over robots.

A. lose

B. take

C. keep

D. gain

Câu hỏi 724 :

The talks were meant to break down _________ between the two groups.

A. gates

B. walls

C. barriers

D. fences

Câu hỏi 725 :

Failing to submit the proposal on time was _________ for Tom.

A. a real kick in the pants

B. an open and shut case

C. a shot in the dark

D. a mail in the coffin

Câu hỏi 730 :

He’ll give Joe a red rose and a lovey- dovey poem he wrote.

A. lovesick

B. wild

C. tragic

D. romantic

Câu hỏi 732 :

- John: “Do you think that we should use public transportation to protect our environment?” 

- Linda: “___________________”


A. Well, that’s very surprising


B. Yes, it’s an absurd idea

C. Of course not. You bet

D. There is no doubt about it

Câu hỏi 735 :

Drawbacks include complaints from (28) _______ that wind turbines are ugly and noisy.

A. locals

B. foreigners

C. master

D. levels

Câu hỏi 736 :

Nevertheless, the wind energy industry is (29) _______.

A. relating

B. worrying

C. booming

D. informing

Câu hỏi 738 :

Read the following passage and mark the letter A, B, C, or D to indicate the correct answer to each of the question.

Some doctors think that you should drink a glass of water each morning. You should drink this water first thing, before doing anything else. The temperature of the water should be similar to body temperature; neither too hot nor too cold.

Why should you drink this water? Water helps your body in many ways. It helps clean out your kidneys. It prepares your stomach for digestion. Water can also help your intestines work better. After drinking water, the intestines can more easily take out nutrients from our food. Water also helps us go to the bathroom more easily.

Scientists suggest that people take in 1,600 milliliters of water each day. But don’t drink all of that water in one sitting. If you do, your kidneys will have to work much harder to eliminate it. It’s better to drink some in the morning and some in the afternoon. Some people think it’s better to drink between meals and not during meals. They think water dilutes the juices produced in our stomachs. This can interfere with normal digestion.

Are you drinking enough water every day? Check the color of your urine. If it is light yellow, you are probably drinking enough. If your urine is very dark yellow, you probably need to drink more water. A little more water each day could make you much healthier.

(Adapted from Reading Challenge 1 by Casey Malarcher and Andrea Janzen)

What is the main idea of the passage?

    


A. The importance of water


B. The advice of the doctors

C. How to drink water correctly?

D. The best amount of water to drink

Câu hỏi 740 :

The word “it” in paragraph 2 refers to ___________.

A. your body

B. your kidney

C. water

D. your stomach

Câu hỏi 741 :

The word “eliminate” in paragraph 3 is closest in meaning to ______________.

A. preserve

B. remove

C. absorb

D. process

Câu hỏi 742 :

Which of the following is NOT true?

A. The first thing you should do every morning is to drink water.

B. You shouldn’t drink too much water at the same time.

C. Drinking water while having meals may interfere with normal digestion.

D. You need to drink more water if your urine is light yellow.

Câu hỏi 743 :

Read the following passage and mark the letter A, B, C, or D to indicate the correct answer to each of the question.

Grandparents are becoming the forgotten generation, with youngsters now too busy to listen to their stories from the olden days.

A study of 1,000 five to 18 year-olds reveals just 21 per cent will visit their older relatives to hear about how their lives were different in the past; such as where they worked, how it was living in the war, and how they met the love of their life. More than half of youths have no idea what job their grandparent did before retirement – admitting they’d never thought to ask. Sadly, one in 10 admitted they are simply not interested in their grandmother’s or grandad’s previous job or talents and interests, and a quarter only turn up to see them for pocket money. But 23 per cent claim the reason they don’t know anything about their older relatives is because they don’t really get the chance to talk properly.

Geoff Bates, spokesman for McCarthy & Stone’s Inspirational Generation campaign, said: “We know this generation have lived full lives with heroic tales to tell and so much to offer, but how many of us have actually thought to ask these questions of our older family members? We want to shout about the amazing feats retirees have achieved in their lifetime and put the spotlight on the wonderfully colorful lives of today’s older people. We are calling on parents and children to talk to their grandparents, to find out what they have done in their lives – and continue to do, and tell us all about it so we can give them the credit they deserve.”

Researchers found that although 65 per cent of youngsters do see their grandparents every single week, 37 per cent claim this is only because their parents want them to. And while 39 per cent talk to their grandparents on the phone, Facebook or Skype at least once a week – 16 per cent once a day – conversation is rarely focused on what they are doing or have done in the past. Four in 10 kids have no idea what their grandparents proudest achievements are, while 30 per cent don’t know if they have any special skills or talents. And 42 per cent don’t spend any time talking about their grandparent’s history – and are therefore clueless about what their grandmother or grandad was like when they were younger. Perhaps due to this lack of communication and respect, just six per cent of children say they look up to their grandparents as a role model and inspiration. However, grandchildren are agreed their grandparents are both loving and friendly, while 43 per cent think they’re funny – with 23 per cent admitting they often have more fun with their elderly relatives than their parents.

(Source: https://www.independent.co.uk)

Which of the following could be the main idea of the passage?

A. Grandparents are outdated people in their families.

B. Young people now do not concern much about their grandparents.

C. Grandparents are not interested in telling stories about their life in the past any more.

D. Young people are too busy to take care of their grandparents.

Câu hỏi 744 :

According to the study in paragraph 2, which information is NOT true?

A. Merely over one fifth of people in the survey keep asking about the bygone time of their grandparents.

B. Over 50% of the young don’t know about their older relatives’ professions before superannuation.

C. Most of youths visit their grandparents to ask for money.

D. Nearly a quarter of young people don’t have proper opportunities to converse with their older relatives.

Câu hỏi 745 :

The word “feats” in the third paragraph is closest in meaning to _________________.

A. accomplishments

B. failures

C. difficulties

D. differences

Câu hỏi 746 :

What does the word “they” in paragraph 3 refer to?

A. parents

B. children

C. colorful lives

D. grandparents

Câu hỏi 747 :

It can be inferred from paragraph 3 that McCarthy & Stone’s Inspirational Generation campaign ____________________.

A. encourages people to ask more questions about their grandparents’ jobs.

B. would like to honour the retirees with their remarkable achievements and experienced life.

C. hopes to give recognition to the older family members.

D. intends to retell the heroic tails of the older generation and find out what they have done in the past.

Câu hỏi 749 :

The word “inspiration” in the last paragraph mostly means ________.

A. disincentive

B. encumbrance

C. stimulation

D. hindrance

Câu hỏi 752 :

Until the invention (A) of the telephone, skyscrapers (B) were not considered (C) very practicable (D).

A. invention

B. skyscrapers

C. considered

D. practicable.

Câu hỏi 753 :

Mark the letter A, B, C, or D on your answer sheet to indicate the sentence that is closest in meaning to each of the following questions

It is a long time since we last met each other.

A. We haven't met each other for a long time.

B. The last time we met each other is a long time ago.

C. We last met each other for a long time.

D. We started meeting each other a long time ago.

Câu hỏi 754 :

"If I were you, I would take a break," Tom said to Daisy.

A. Tom wanted to take a break with Daisy.

B. Tom advised Daisy to take a break.

C. Tom suggested not taking a break.

D. Tom wanted to take a break, and so did Daisy.

Câu hỏi 755 :

It is impossible for you to buy a big house with little money

A. You are able to buy a big house with little money.

B. You can’t buy a big house with little money.

C. You will buy a big house with little money.

D. You have to buy a big house with little money.

Câu hỏi 756 :

Mark the letter A, B, C, or D on your answer sheet to indicate the sentence that best combines each pair of sentences in the following questions

You are not tactful; that is why you are always offending other people.

A. You must have offended other people because you are not tactful.

B. If you were tactful, you would satisfy other people.

C. I wish you were tactful so that you don’t offend other people.

D. You should have been tactful so that you didn’t offend other people.

Câu hỏi 757 :

He not only studies well, but also he sings well.

A. Not only does he study well, but also he sings well.

B. Not only he studies well, but also does he sing well.

C. Not only he does study well, but also does he sing well.

D. Not only he studies well, but also he sings well.

Câu hỏi 763 :

My passport ________ last year while I was on vacation.

A. was stolen

B. was being stolen

C. had been stolen

D. stole

Câu hỏi 765 :

_______ you prepare for the exam, the _______ you will be.

A. The earlier / more successfully

B. The more early / the more successful

C. The earliest / the most successful

D. The earlier / the more successful

Câu hỏi 766 :

He made a toy as a birthday gift to his daughter. It is a _________ house.

A. beautiful big pink

B. beautiful pink big

C. pink beautiful big

D. big pink beautiful

Câu hỏi 767 :

A fire-fighter was injuried severely while he _______ to put out a fire.

A. would help

B. helped

C. has helped

D. was helping

Câu hỏi 768 :

People should stop smoking ________ it is extremely detrimental to health.

A. despite

B. although

C. because of

D. because

Câu hỏi 769 :

We will invite all of our friends to come over for a house- warning party _______.

A. as soon as we have moved into the new house.

B. until we moved into the new house

C. after we will move into the new house

D. when we moved into the new house

Câu hỏi 770 :

________ every major judo title, Mark retired from international competition.

A. Having won

B. Winning

C. When he won

D. On winning

Câu hỏi 772 :

He _________ the club on recommendation from his friend Jack.

A. got in

B. got on

C. got by

D. got into

Câu hỏi 774 :

It took him a long time to come to __________ with the fact that he was homeless.

A. tabs

B. agreement

C. terms

D. acceptance

Câu hỏi 778 :

My parents' warnings didn't deter me from choosing the job of my dream.

A. influence

B. inspire

C. reassure

D. discourage

Câu hỏi 780 :

The Covid – 19 pandemic has taken a heavy toll on the economy of many countries.

A. put blame

B. exerted a small pressure

C. had little impact

D. been a slight loss

Câu hỏi 788 :

Read the following passage and mark the letter A, B, C or D to indicate the correct answer to each of the questions from 30 to 34.

When we meet people for the first time, we often make decisions about them based entirely on how they look. And of course, we too are being judged on our appearance. Undoubtedly, it’s what’s inside that’s important but sometimes we can send out the wrong signals and so get a negative reaction, simply by wearing inappropriate clothing.

When selecting your clothes each day, it is therefore important to think about who you’re likely to meet, where you are going to be spending most of your time and what tasks you are likely to perform. Clearly, on a practical level, some outfits will be more appropriate to different sorts of activity and this will dictate your choice to an extent. However, there’s no need to abandon your individual taste completely. After all, if you dress to please somebody else’s idea of what looks good, you may end up feeling uncomfortable and not quite yourself.

Some colours bring your natural colouring to life and others can give you a washed-out appearance. Try out new ones by all means, but remember that dressing in bright colours when you really like subtle neutral tones or vice versa will make you feel self-conscious and uncomfortable. You know deep down where your own taste boundaries lie. It may be fun to cross these sometimes, but do take care not to go too far all at once.

Reappraising your image isn’t selfish because everyone who comes into contact with you will benefit. You’ll look better and you’ll feel a better person all round. And if in doubt, you only need to read Professor Albert Mehrabian’s book Silent Messages, which showed that the impact we make on each other depends 55 percent on how we look and behave, 38 percent on how we speak, and only seven percent on what we actually say.

(Adapter from “Expert First” by Jan Bell and Roger Gower)

Which could be the best title for the passage?

A. Choosing Appropriate Business Suits

B. Making Judgements about People’s Appearance

C. Making Your Image Work for You

D. Creating a Professional Image

Câu hỏi 789 :

The word “outfits” in paragraph 2 mostly means      .

A. sets of clothes.

B. types of signals.

C. types of gestures.

D. sets of equipment.

Câu hỏi 790 :

Which of the following is NOT mentioned in paragraph 2 as a factor to be considered when choosing clothes?

A. Places you spend time in.

B. Other people’s views on beauty.

C. Kinds of tasks you perform.

D. People you meet.

Câu hỏi 791 :

The word “others” in paragraph 3 refers to   .

A. neutral tones.

B. taste boundaries.

C. colours.

D. means.

Câu hỏi 792 :

According to Professor Albert Mehrabian, the impact we make on each other depends mainly on            .

A. how we speak.

B. how we look and behave.

C. what we read.

D. what we actually say.

Câu hỏi 793 :

Read the following passage and mark the letter A, B, C or D to indicate the correct answer to each of the questions from 35 to 42.

Did you ever watch a video on the Internet? Maybe you used YouTube. YouTube is a Website where people can share their video. Today, YouTube is an important part of the Internet. However, that wasn't always true.

YouTube started with a young man named Jawed Karim and two friends. One day, Karim was on the Internet. He wanted information about the 2004 tsunami in Southeast Asia. He found news stories about it, but he couldn't find any videos. This gave Karim an idea. He wanted to help people put video on the Internet. Karim told his friends about this idea. Together, they created a company - YouTube.

YouTube become a global success. Millions of people around the world visited the Website. It was clear to Google, another Internet company, that YouTube had a lot of value. Google made a deal. It bought YouTube for $1.65 billion. As a result, YouTube investors and its employees made a lot of money. The three friends who started YouTube were very big investors. Therefore, they made an enormous amount of money.

Karim became very rich, and he continued to work toward his PhD. There was something else he wanted to do. He wanted to help young people go into business. He used money and experience to start a new company called Youniversity Ventures. This company helps young people who have good business ideas. It gives them advice and money to start Internet businesses. Milo is one business that students started with the help of Youniversity Ventures.

Milo is a shopping Website. It helps people find products in stores near their homes. Another example is AirBob. This Web site helps people find for video conferences. People in different places can use this site to have business meetings.

Karim has some advice for students who want to start business. First, find a successful company. Do a lot of research about the company and the top people in the company. There, copy the way they do things. For students who wants to start Internet business, Karim is probably a very good example to copy.

What can be the best title for the passage?

A. Sharing Success on the Internet.

B. To be successful on the Internet.

C. Support from Youniversity Ventures.

D. The best videos from YouTube.

Câu hỏi 794 :

What does the word "This" in paragraph 2 refers to?

A. the 2004 tsunami in Southeast Asia.

B. that he could find no videos.

C. the news stories he could find.

D. the information of the 2004 tsunami.

Câu hỏi 795 :

What do people use YouTube for?

A. doing shopping.

B. sharing videos.

C. starting business.

D. finding vacation places.

Câu hỏi 796 :

Which of the following in NOT mentioned about Karim?

A. He had the original idea of YouTube.

B. He created a lot of sample videos for YouTube.

C. He earned a lot of money from YouTube.

D. He is the owner of Youniversity Ventures.

Câu hỏi 797 :

YouTube now belongs to ___________

A. Milo

B. Youniversity

C. Google

D. Tokbox

Câu hỏi 798 :

Which of the following can replace the word "enormous" in paragraph 4?

A. increasing

B. giant

C. considerable

D. huge

Câu hỏi 799 :

What does Youniversity Ventures do?

A. It helps people to get PhDs at famous Universities.

B. It buys Internet companies such as Milo.

C. It gives people advice and money to start business.

D. It helps people find inexpensive vacation places worldwide.

Câu hỏi 801 :

His(A) house was (B) far away so (C) I had to live with my relatives (D).

A. His

B. was

C. so

D. with my relatives

Câu hỏi 803 :

Mark the letter A, B, C, or D on your answer sheet to indicate the sentence that is closest in meaning to each of the following questions
I last had my hair cut in November.


A. The last time I had my hair cut was since November.


B. I haven't had my hair cut since November.

C. I didn't have my hair cut in November.

D. I started having my hair cut in November.

Câu hỏi 804 :

The secretary said, "Sorry, I will never work on Sunday.”

A. The secretary promised not to work on Sunday.

B. The secretary refused not to work on Sunday.

C. The secretary reminded her boss to work on Sunday.

D. The secretary refused to work on Sunday.

Câu hỏi 805 :

It isn’t necessary for us to get a visa for Singapore.

A. We needn’t get a visa for Singapore.

B. We mustn’t get a visa for Singapore.

C. We mayn’t get a visa for Singapore.

D. We shouldn’t get a visa for Singapore.

Câu hỏi 806 :

Mark the letter A, B, C, or D on your answer sheet to indicate the sentence that best combines each pair of sentences in the following questions
It is a pity that I can’t speak English as native speaker.

A. I wish I could speak English as a native speaker.

B. I wish I couldn’t speak English as a native speaker.

C. I wish I can speak English as a native speaker.

D. I wish I have spoken English as a native speaker.

Câu hỏi 807 :

Alice registered for the course. She then received the scholarship.

A. Hardly had Alice registered for the course when she received the scholarship.

B. Only after Alice registered for the course, she received the scholarship.

C. Having received the scholarship, Alice registered for the course.

D. Registering for the course helped Alice receive the scholarship.

Câu hỏi 813 :

The origin of April Fool’s Day, the first day of April, is uncertain, but it _______ to arrival of spring in late March, when nature _______ to “fool” humanity with changes in weather.

A. has been related – has said

B. may be related – is said

C. ought to relate – has been said

D. relates – is saying

Câu hỏi 815 :

The more different cultures work together, _______ essential to avoid problems.

A. the more cultural competency training is

B. the more culturally competent training is

C. the more cultural competent training is

D. the more culturally competency training is

Câu hỏi 816 :

There is a _______ table which was given to me by my best friends on my house warming.

A. large beautiful round wooden

B. beautiful large round wooden

C. beautiful round large wooden

D. wooden large round beautiful

Câu hỏi 817 :

They didn’t see anyone while they _______ home because it was raining.

A. were walking

B. hadn’t walked

C. didn’t walk

D. weren’t walked

Câu hỏi 819 :

They _______ our winter house by the time we _______ from our summer house.

A. painted - had returned

B. will have painted - return

C. will be painting - have returned

D. have been painting - have returned

Câu hỏi 820 :

One of the men was lying on the ground after _______ down by a piece of rock.

A. being knocked

B. knocking

C. having been knock

D. having knocked

Câu hỏi 822 :

She _______ her hands in horror at his suggestion that she should marry him.

A. broke up

B. threw up

C. brought up

D. woke up

Câu hỏi 823 :

Could you give me a rough_______ of what the decoration job might cost?

A. estimate

B. correlation

C. account

D. value

Câu hỏi 825 :

I haven’t made up my mind about that issue; I’ll have to sit on the _______.

A. fence

B. edge

C. mountain

D. eggshell

Câu hỏi 826 :

His poor standard of play fully justifies his _______ from the team for the next match.

A. expulsion

B. dismissal

C. rejection

D. exclusion

Câu hỏi 828 :

I think Michael hit the nail on the head when he said that what is lacking in this company is the feeling of confidence.

A. interpreted something indirectly

B. described something unconsciously

C. said something correctly

D. misunderstood something seriously

Câu hỏi 830 :

He is very stubborn but flattery usually works like a charm on him.

A. is completely successful in

B. takes effect

C. turned out to be a disaster

D. has no effect on

Câu hỏi 832 :

Hana and Jenifer are talking about a book they have just read.

     - Hana: “The book is really interesting and educational.”

     - Jenifer: “_______.”


A. Don’t mention it.


B. That’s nice of you to say so.

C. I’d love it.

D. I couldn’t agree more.

Câu hỏi 833 :

Read the following passage and mark the letter A, B, C, or D to indicate the correct word or phrase that best fits each the numbered blanks.

Sir Isaac Newton, the English scientist and mathematician, was one of the most important figures of the 17th century scientific revolution. One of his greatest achievement was the (26) _______ of the three laws of motion, which are still used today. But he also had a very unusual personality. Some people would say he was actually insane.

His father died before he was born, and his mother soon remarried. The young Isaac hated his stepfather so much that he once (27) _______ to burn his house down - when his stepfather and mother were still inside! Fortunately he did not, and he went on to graduate from Cambridge without being thrown into prison.

Isaac's first published work was a theory of light and color. When another scientist wrote a paper criticizing this theory, Isaac flew into an uncontrollable rage. The scientist responsible for the criticism was a man called Robert Hooke. He was head of the Royal Society, and one of the most respected scientists in the country. (28) _______, this made no difference to Isaac, (29) _______ refused to speak to him for over a year.

The simple fact was that Isaac found it impossible to have a calm discussion with anyone. As soon as someone said something that he disagreed with, he would lose his temper. For this reason he lived a large part of his life isolated from (30) _______ scientists. It is unlikely that many of them complained.

                                                             (Source: https://www.biography.com/scientist/isaac-newton)

A. discovery

B. research

C. findings

D. inventions

Câu hỏi 837 :

For this reason he lived a large part of his life isolated from (30) _______ scientists.

A. others

B. the others

C. another

D. other

Câu hỏi 838 :

Read the following passage and mark the letter A, B, C, or D to indicate the answer to each of the question.

Called the ‘Red Planet,’ Mars is roughly half the size of Earth, and one of our closest neighboring planets. Though Mars is the most Earth-like of any other planet, the two are still worlds apart. Living on Mars has been the stuff of science fiction for decades. However, can humans really live on Mars? Will it ever be possible or safe? NASA (the National Aeronautics and Space Administration) hopes to find out. NASA researchers on Earth are conducting several experiments together with the International Space Station (ISS) to study the health and safety issues that may tell us if life on Mars is possible.

Food and oxygen would be the main necessities for travelers living extended periods on Mars. The need to grow plants, which provide both food and oxygen, would be a key. But the decreased gravity and low atmospheric pressure environment of the planet will stress the plants and make them hard to grow. However, space station crews are growing plants in controlled environments in two of the station’s greenhouses. They take care of the plants, photograph them, and collect samples to be sent back to Earth. Researchers then use the data to develop new techniques that will make it possible to grow plants successfully in space.

Another concern for space travelers is the health hazards posed by the effect of space radiation on humans. A spacecraft traveling to Mars would be exposed to large amounts of radiation. Since human exposure to such intense radiation would mean certain death, the spacecraft used for such travel would have to protect the humans on the inside of the craft from exposure. Researchers are using special machines inside the crew areas of the International Space Station to carefully watch radiation levels. NASA scientists, who have maintained radiation data since the beginning of human space flight, continue to learn about the dangers it poses. Researchers use the station to test materials that could be used in making a spacecraft that could successfully travel to Mars.

Will it ever be safe for humans to live on Mars? It is still too early to say. But thanks to the dedicated researchers of NASA and the results of ISS experiments, we are getting closer to knowing every day.

                   (Adapted from “Select Readings –Intermediate Tests” by Linda Lee and Erik Gundersen)

What does the passage mainly discuss?


A. The potential of Mars.


B. Life on Mars.

C. The experiments on Mars.

D. The pressure on Mars.

Câu hỏi 839 :

What does the word “they” in paragraph 2 refer to?

A. space station crews

B. plants

C. environments

D. station’s greenhouses

Câu hỏi 840 :

As mentioned in paragraph 2, why can’t people grow plants successfully in space?

A. Because there is a lack of food and oxygen on Mars.

B. Because there aren’t enough station’s greenhouses to control the environments.

C. Because of the reduced gravity and low atmospheric pressure environment.

D. Because of the shortage of new techniques.

Câu hỏi 841 :

The word “hazards” in paragraph 3 is closest in meaning to _______.

A. dangers

B. problems

C. diseases

D. symptoms

Câu hỏi 842 :

According to paragraph 3, which of the following is the demand for manufacturing spacecrafts travelling to Mars?

A. They must contain special machines inside to watch radiation levels.

B. They have to be made from special materials which are light and safe for travelling.

C. They have to maintain radiation data from the beginning of human space flight inside.

D. They have to protect travelers from radiation exposure effectively.

Câu hỏi 843 :

Read the following passage and mark the letter A, B, C, or D to indicate the answer to each of the question.

The Trump campaign ran on bringing jobs back to American shores, although mechanization has been the biggest reason for manufacturing jobs’ disappearance. Similar losses have led to populist movements in several other countries. But instead of a pro-job growth future, economists across the board predict further losses as AI, robotics, and other technologies continue to be ushered in. What is up for debate is how quickly this is likely to occur.

 Now, an expert at the Wharton School of Business at the University of Pennsylvania is ringing the alarm bells. According to Art Bilger, venture capitalist and board member at the business school, all the developed nations on earth will see job loss rates of up to 47% within the next 25 years, according to a recent Oxford study. “No government is prepared,” The Economist reports. These include blue and white collar jobs. So far, the loss has been restricted to the blue collar variety, particularly in manufacturing.

To combat “structural unemployment” and the terrible blow, it is bound to deal the American people, Bilger has formed a nonprofit called Working Nation, whose mission it is to warn the public and to help make plans to safeguard them from this worrisome trend. Not only is the entire concept of employment about to change in a dramatic fashion, the trend is irreversible. The venture capitalist called on corporations, academia, government, and nonprofits to cooperate in modernizing our workforce.

To be clear, mechanization has always cost us jobs. The mechanical loom, for instance, put weavers out of business. But it also created jobs. Mechanics had to keep the machines going, machinists had to make parts for them, and workers had to attend to them, and so on. A lot of times those in one profession could pivot to another. At the beginning of the 20th century, for instance, automobiles were putting blacksmiths out of business. Who needed horseshoes anymore? But they soon became mechanics. And who was better suited?

Not so with this new trend. Unemployment today is significant in most developed nations and it’s only going to get worse. By 2034, just a few decades, mid-level jobs will be by and large obsolete. So far the benefits have only gone to the ultra-wealthy, the top 1%. This coming technological revolution is set to wipe out what looks to be the entire middle class. Not only will computers be able to perform tasks more cheaply than people, they’ll be more efficient too.

Accountants, doctors, lawyers, teachers, bureaucrats, and financial analysts beware: your jobs are not safe. According to The Economist, computers will be able to analyze and compare reams of data to make financial decisions or medical ones. There will be less of a chance of fraud or misdiagnosis, and the process will be more efficient. Not only are these folks in trouble, such a trend is likely to freeze salaries for those who remain employed, while income gaps only increase in size. You can imagine what this will do to politics and social stability.

                                                                                                               (Source: https://bigthink.com/)

Which of the following could be the main idea of the passage?


A. Many jobs will disappear in the future.


B. AI will replace the workers’ positions in almost jobs.

C. Manufacturing jobs are predicted to be the first ones to disappear.

D. Changing jobs is not a new trend in the future.

Câu hỏi 844 :

The word “irreversible” in paragraph 3 is closest in meaning to _______.

A. impermanent

B. remediable

C. reparable

D. unalterable

Câu hỏi 845 :

According to the passage, which of the following is NOT true about jobs in the future?

A. AI, robots and technologies continuously used will put more labourers out of their jobs.

B. Every country has applied many policies to prepare for the massive loss of jobs in the next 25 years.

C. Many different organizations are called to cooperate in renovating the workforce.

D. Working Nation is an organization founded to warn the public and make plans to save people from job loss.

Câu hỏi 846 :

According to paragraph 4, what is the advantage of mechanization?

A. Although mechanization drives people out of work, it also creates more jobs.

B. People can change their jobs to be more suitable with the society.

C. People will no longer need the useless like horseshoes.

D. Workers will spend less time on manufacturing with the help of machines.

Câu hỏi 847 :

The word “obsolete” in paragraph 5 could be best replaced by _______.

A. outdated

B. modern

C. fashionable

D. adventurous

Câu hỏi 848 :

What does the word “they” in paragraph 5 refer to?

A. people

B. tasks

C. computers

D. the entire middle class

Câu hỏi 849 :

Why does the author mention in the last paragraph that accountants, doctors, lawyers, teachers, bureaucrats, and financial analysts are not safe jobs?

A. Because they are easy to make mistakes or misdiagnosis in doing their jobs.

B. Because the salaries paid for these jobs may be frozen in the future.

C. Because computers are likely to analyze and process a great amount of data with high accuracy.

D. Because these jobs directly influence politics and social stability.

Câu hỏi 853 :

Mark the letter A, B, C, or D to indicate the sentence that is closest in meaning to each of the following questions.

It’s almost nine months since I stopped subscribing to that magazine.


A. I have subscribed to that magazine for almost nine months.


B. I have subscribed to that magazine almost nine months ago.

C. I cancelled my subscription to that magazine almost nine months ago.

D. I have subscribed to that magazine for almost nine months, but now I stopped.

Câu hỏi 854 :

"Good luck in your new job. It will work out well for you," said his mother.

A. His mother wished him good luck and said his new job would work out well for him.

B. His mother wanted him to get good luck in his new job as it would work out well for him.

C. His mother wondered whether he got luck in his job or it would work out well for him.

D. His mother ascertained that his new job would work out well for him and bring him luck.

Câu hỏi 855 :

It was wrong of you not to ask your parents’ permission before deciding to quit your job.

A. You must have asked your parents’ permission before deciding to quit your job.

B. You might have asked your parents’ permission before deciding to quit your job.

C. You had to have asked your parents’ permission before deciding to quit your job.

D. You should have asked your parents’ permission before deciding to quit your job.

Câu hỏi 856 :

Mark the letter A, B, C, or D to indicate the sentence that best combines each pair of sentences in the following questions.

The boss scolded Laura for that minor mistake. He now feels really bad about it.


A. The boss wishes he did not scold Laurafor that minor mistake.


B. The boss wishes Laura had not made that minor mistake.

C. If only the boss had not scolded Laura for that minor mistake.

D. The boss regretted to scold Laurafor that minor mistake.

Câu hỏi 857 :

Olga was about to say something about the end of the movie. He was stopped by his friends right then.

A. Hardly had Olga intended to say something about the end of the movie before he was stopped by his friends.

B. Only after Olgahad said something about the end of the movie was he stopped by his friends.

C. Were it not for Olga’s intension of saying something about the end of the movie, he would not be stopped by his friends.

D. It was not until Olga was stopped by his friends that he started to say something about the end of the movie.

Câu hỏi 863 :

The old woman still recalls clearly_______ by her teacher when she was late on her first day at school.

A. to be criticized

B. to have criticized

C. being criticized

D. criticizing

Câu hỏi 865 :

The higher the pollution becomes, _______ lose their natural habitats.

A. the more animals

B. more animals

C. many animals

D. the most animals

Câu hỏi 866 :

Visitors to the local museum are mostly attracted by _______ rocking chair.

A. an old wooden European beautiful

B. a beautiful old European wooden

C. an old beautiful wooden European

D. a wooden old beautiful European

Câu hỏi 867 :

He_______a terrible accident while he_______along Ben Luc Bridge.

A. see - am walking

B. saw - was walking

C. was seeing - walked

D. have seen - were walking

Câu hỏi 868 :

She had butterflies in her stomach _______ having prepared carefully for the interview.

A. although

B. due to

C. despite

D. because

Câu hỏi 869 :

By the end of this month, I hope I _______ 200 pages of my new novel.

A. had written

B. will have written

C. will have been writing

D. have written

Câu hỏi 870 :

_______, the examinees knew it was time to stop.

A. had written

B. To hear the bell

C. Heard the bell

D. Hearing the bell

Câu hỏi 871 :

Urbanization has resulted in _______ problems besides the benefits.

A. vary

B. various

C. variety

D. variability

Câu hỏi 872 :

The company's advertising campaign was a _______ failure - it didn't attract a single new customer.

A. miraculous

B. spectacular

C. wonderful

D. unprecedented

Câu hỏi 873 :

Beaches were _______ as police searched for canisters of toxic waste from the damaged ship.

A. cut off

B. sealed off

C. washed up

D. kept out

Câu hỏi 874 :

Mr John is held in high _______ for his dedication to the school.

A. respect

B. esteem

C. homage

D. honor

Câu hỏi 875 :

The judge show that the murderer had shown a callous _______ for human life.

A. disregard

B. ignorance

C. omission

D. neglect

Câu hỏi 876 :

Having three sons under the age of five keeps Jana on her _______.

A. knees

B. feet

C. toes

D. Legs

Câu hỏi 878 :

He sank into deep despair when he lost his job.

A. became very enjoyable

B. became very unhappy

C. was delighted

D. burst into tears

Câu hỏi 880 :

I heard it through the grapevine that they were enemies and had never been on good terms.

A. gave it the low-down

B. put it in the picture

C. heard it straight from the horse’s mouth

D. kept it in the loop

Câu hỏi 883 :

Read the following passage and mark the letter A, B, C, or D to indicate the correct word or phrase that best fits each the numbered blanks.

The tradition of gift giving is a worldwide (26) ______ that is said to have been around since the beginning of human beings. Over time, different cultures have developed their own gift giving customs and traditions.

In France, the gift of wine for the hostess of a dinner party is not an appropriate gift as the hostess would prefer to choose the vintage for the night. In Sweden, a bottle of wine or flowers are an appropriate gift for the hostess. In Viet Nam, a gift of whisky is appropriate for the host, and some fruit or small gifts for the hostess, children or elders of the home. Besides, gifts should never be wrapped in black paper because this color is unlucky and associated with funerals in this country. Gifts (27) ______ symbolize cutting such as scissors, knives and other sharp objects should be avoided because they mean the cutting of the relationship. Also, in some countries you should not open the gift in front of the giver and in (28) ______ it would be an insult if you did not open the gift.

Beyond the gift itself, give careful consideration to the manner in which it is presented. Different cultures have different customs regarding how a gift should be offered - using only your right hand or using both hands, (29) ______. Others have strong traditions related to the appropriate way to accept a gift. In Singapore, for instance, it is the standard to graciously refuse a gift several times before finally accepting it. The recipient would never unwrap a gift in front of the giver for fear of appearing greedy.

Understanding these traditions and customs, as well as taking time to choose an appropriate gift, will help you to avoid any awkwardness or (30) ______ as you seek to build a better cross-cultural relationship.

                                                           (Source: http://www.giftypedia.com/International_Gift_Customs)


A. exercise


B. practice

C. work

D. task

Câu hỏi 888 :

The word “it” in the first paragraph refers to _______.

A. organisation

B. machinery

C. knowledge

D. company

Câu hỏi 889 :

Read the following passage and mark the letter A, B, C, or D to indicate the answer to each of the question.

One way of training for your future occupation in Germany is by pursuing a dual vocational training programme. Such programmes offer plenty of opportunity for on-the-job training and work experience. Programmes usually last between two and three and a half years and comprise theoretical as well as practical elements. You will spend one or two days a week, or several weeks at once, at a vocational school where you will acquire the theoretical knowledge that you will need in your future occupation. The rest of the time will be spent at a company. There you get to apply your newly acquired knowledge in practice, for example by learning to operate machinery. You will get to know what your company does, learn how it operates and find out if you can see yourself working there after completing your training.

This combination of theory and practice gives you a real head start into your job: by the time you have completed your training, you will not only have the required technical knowledge, but you will also have hands-on experience in your job. There are around 350 officially recognised training programmes in Germany, so chances are good that one of them will suit your interests and talents. You can find out which one that might be by visiting one of the jobs and vocational training fairs which are organised in many German cities at different times in the year.

Employment prospects for students who have completed a dual vocational training programme are very good. This is one of the reasons why this kind of training is very popular with young Germans: around two thirds of all students leaving school go on to start a vocational training programme.

                                                                              (Source: http ://www. make-it-in-germany. com)

How many German school leavers choose this vocational training programme?

A.around one out of five

B. about 70%


C. less than a third


D. well over 75%

Câu hỏi 890 :

Which of the following statements best describes the dual vocational training programmes?

A. These programmes provide you with both theoretical knowledge and practical working experience.


B. These programmes consist of an intensive theoretical course of two and a half years at a vocational school.


C. These programmes require you to have only practical working time at a certain company.

D. These programmes offer you some necessary technical skills to do your future job.

Câu hỏi 891 :

The word “hands-on” in the second paragraph is closest in meaning to _______.

A. practical

B. technical

C. theoretical

D. integral

Câu hỏi 892 :

Which of the following is probably the best title of the passage?

A. Employment Opportunities and Prospects in Germany

B. Dual Vocational Training System in Germany

C. Combination of Theory and Practice in Studying in Germany

D. Higher Education System in Germany

Câu hỏi 893 :

Read the following passage and mark the letter A, B, C, or D to indicate the answer to each of the question.

      The Trump campaign ran on bringing jobs back to American shores, although mechanization has been the biggest reason for manufacturing jobs’ disappearance. Similar losses have led to populist movements in several other countries. But instead of a pro-job growth future, economists across the board predict further losses as AI, robotics, and other technologies continue to be ushered in. What is up for debate is how quickly this is likely to occur.

      Now, an expert at the Wharton School of Business at the University of Pennsylvania is ringing the alarm bells. According to Art Bilger, venture capitalist and board member at the business school, all the developed nations on earth will see job loss rates of up to 47% within the next 25 years, according to a recent Oxford study. “No government is prepared,” The Economist reports. These include blue and white collar jobs. So far, the loss has been restricted to the blue collar variety, particularly in manufacturing.

       To combat “structural unemployment” and the terrible blow, it is bound to deal the American people, Bilger has formed a nonprofit called Working Nation, whose mission it is to warn the public and to help make plans to safeguard them from this worrisome trend. Not only is the entire concept of employment about to change in a dramatic fashion, the trend is irreversible. The venture capitalist called on corporations, academia, government, and nonprofits to cooperate in modernizing our workforce.

       To be clear, mechanization has always cost us jobs. The mechanical loom, for instance, put weavers out of business. But it also created jobs. Mechanics had to keep the machines going, machinists had to make parts for them, and workers had to attend to them, and so on. A lot of times those in one profession could pivot to another. At the beginning of the 20th century, for instance, automobiles were putting blacksmiths out of business. Who needed horseshoes anymore? But they soon became mechanics. And who was better suited?

         Not so with this new trend. Unemployment today is significant in most developed nations and it’s only going to get worse. By 2034, just a few decades, mid-level jobs will be by and large obsolete. So far the benefits have only gone to the ultra-wealthy, the top 1%. This coming technological revolution is set to wipe out what looks to be the entire middle class. Not only will computers be able to perform tasks more cheaply than people, they’ll be more efficient too.

       Accountants, doctors, lawyers, teachers, bureaucrats, and financial analysts beware: your jobs are not safe. According to The Economist, computers will be able to analyze and compare reams of data to make financial decisions or medical ones. There will be less of a chance of fraud or misdiagnosis, and the process will be more efficient. Not only are these folks in trouble, such a trend is likely to freeze salaries for those who remain employed, while income gaps only increase in size. You can imagine what this will do to politics and social stability.

                                                                                                            (Source: https://bigthink.com/)

Which of the following could be the main idea of the passage?



A. Many jobs will disappear in the future.




B. AI will replace the workers’ positions in almost jobs.

C. Manufacturing jobs are predicted to be the first ones to disappear.

D. Changing jobs is not a new trend in the future.

Câu hỏi 894 :

It can be inferred from paragraph 2 that “blue and white collar jobs” are related to ____________.

A. people whose uniforms’ colors are blue and white.

B. people who are distinguished by the colors of their collars.

C. people who do physical work in industry and who work in an office.

D. people (mainly women) who do low-paid jobs, for example in offices and restaurants.

Câu hỏi 895 :

The word “irreversible” in paragraph 3 is closest in meaning to ___________.

A. impermanent

B. remediable

C. reparable

D. unalterable

Câu hỏi 896 :

According to the passage, which of the following is NOT true about jobs in the future?

A. AI, robots and technologies continuously used will put more labourers out of their jobs.

B. Every country has applied many policies to prepare for the massive loss of jobs in the next 25 years.

C. Many different organizations are called to cooperate in renovating the workforce.

D. Working Nation is an organization founded to warn the public and make plans to save people from job loss.

Câu hỏi 897 :

The word “obsolete” in paragraph 5 could be best replaced by __________.

A. outdated

B. modern

C. fashionable

D. adventurous

Câu hỏi 898 :

What does the word “they” in paragraph 5 refer to?

A. people

B. tasks

C. computers

D. the entire middle class

Câu hỏi 899 :

Why does the author mention in the last paragraph that accountants, doctors, lawyers, teachers, bureaucrats, and financial analysts are not safe jobs?

A. Because they are easy to make mistakes or misdiagnosis in doing their jobs.

B. Because the salaries paid for these jobs may be frozen in the future.

C. Because computers are likely to analyze and process a great amount of data with high accuracy.

D. Because these jobs directly influence politics and social stability.

Câu hỏi 903 :

Mark the letter A, B, C, or D to indicate the sentence that is closest in meaning to each of the following questions.

It is over twenty years since I last got in touch with them.


A. I can’t help keeping getting in touch with them for over 20 years.


B. I haven’t gotten in touch with them for over 20 years.

C. I used to get in touch with them for over 20 years.

D. I have been getting in touch with them for over 20 years.

Câu hỏi 904 :

Do homework now or I'll cut your pocket money! shouted the mum to her son.

A. The mum threatened to cut his pocket money if her son didn't do homework.

B. The mum said that she would cut his pocket money if her son didn't do homework.

C. The mum told her son that she would cut his pocket money if he didn't do homework.

D. The mum informed her son that she would cut his pocket money if he didn't do homework.

Câu hỏi 905 :

It was wrong of you not to show any concern for her mom's health.

A. You should show concern for her mom's health.

B. You should have shown concern for her mom's health.

C. It was wrong that you showed concern for her mom's health.

D. You shouldn't have shown any concern for her mom's health.

Câu hỏi 906 :

Mark the letter A, B, C, or D to indicate the sentence that best combines each pair of sentences in the following questions.

Maria says she'd like to have been put in a higher class.


A. Maria wishes that she will be put in a higher class.


B. Maria wishes that she is put in a higher class.

C. Maria wishes that she had been put in a higher class.

D. Maria wishes that she were put in a higher class.

Câu hỏi 907 :

As soon as James started working, he realized that his decision had not been a good one.

A. Just before James took up his new post, he realized that he was not suited for it.

B. No sooner had James begun his new job than he knew his decision was wrong.

C. Had James not begun his new job, he would have gone looking for a better one.

D. Since James did not like his new job, he began looking for a better one.

Câu hỏi 913 :

We'll go for a walk in the park ___________________.

A. as soon as it stops raining

B. whenever it rains

C. until it has stopped raining

D. after it had stopped raining

Câu hỏi 915 :

The kids _______ in the garden when it suddenly began to rain.

A. played

B. were being played

C. has played

D. were playing

Câu hỏi 916 :

Three books on the top of the shelf _______regularly in the class.

A. is used

B. are used

C. used

D. have been used

Câu hỏi 917 :

My grandmother has knitted a _________ pullover for me.

A. woolen nice new

B. new nice woolen

C. woolen new nice

D. nice new woolen

Câu hỏi 918 :

______ from your school you live, the earlier you have to get up.

A. The farer

B. Farer

C. The farther

D. Farther

Câu hỏi 919 :

She still pursues her dream job as a firefighter __________ many obstacles.

A. In spite of

B. Because

C. Although

D. Owing to

Câu hỏi 921 :

____________ the instructions twice, I still couldn’t understand how to use it.

A. Having read

B. Reading

C. To rescue

D. To be rescued

Câu hỏi 922 :

A student may want to know how many stages insects ____ before they become butterflies.

A. made up of

B. become of

C. go through

D. put through

Câu hỏi 923 :

He didn’t have to go to prison but he was placed under house ___ for six months.

A. arrest

B. management

C. investigation

D. record

Câu hỏi 926 :

I don’t use rat ______ in my house because it might harm my cat.

A. form

B. autograph

C. source

D. poison

Câu hỏi 928 :

She did not know whether to say yes or no to the proposal, she was sitting on the fence.

A. be able to make a decision

B. hesitating about making a choice

C. sitting in front of her house

D. finding solutions

Câu hỏi 930 :

The team had partied late into the night, but on match day, each of them was as fit as a fiddle.

A. be in a good physical condition

B. too tired to play well

C. good enough to play with a fiddle

D. be good fitness

Câu hỏi 938 :

Read the following passage and mark the letter A, B, C, or D on your answer sheet to indicate the correct answer to each of the questions from 31 to 35.

ARE SPORTS BAD FOR KIDS?

       People think children should play sports. Sports are fun, and children stay healthy while playing with others. However, playing sports can have negative effects on children. It may produce feelings of poor self-esteem or aggressive behavior in some children. According to research on kids and sports, 40 million kids play sports in the US. Of these, 18 million say they have been yelled at or called names while playing sports. This leaves many children with a bad impression of sports. They think sports are just too aggressive.

       Many researchers believe adults, especially parents and coaches, are the main cause of too much aggression in children’s sports. They believe children copy aggressive adult behavior. This behavior is then further reinforced through both positive and negative feedback. Parents and coaches are powerful teachers because children usually look up to them. Often these adults behave aggressively themselves, sending children the message that winning is everything. At children’s sporting events, parents may yell insults at other players or cheer when their child behaves aggressively. As well, children may be taught that hurting other players is acceptable, or they may be pushed to continue playing even when they are injured. In addition, the media makes violence seem exciting. Children watch adult sports games and see violent behavior replayed over and over on

television.

       As a society, we really need to face up to this problem and do something about it. Parents and coaches

should act as better examples for children. They also need to teach children better values. They should teach children to enjoy themselves whether they win or not. It is not necessary to knock yourself out to enjoy sports. Winning is not everything. In addition, children should not be allowed to continue to play when they are injured. Sending a child with an injury into a game gives the child the message that health is not as important as winning. If we make some basic changes, children might learn to enjoy sports again.

What is the main idea of the reading?

A. Children often become like their parents.

B. Children need to play sports in school.

C. Playing sports may have negative results.

D. Some sports can cause health problems.

Câu hỏi 939 :

Which is NOT TRUE according to the passage?

A. Adults are the main cause of children’s aggression in sports.

B. Children usually respect parents and coaches so they are powerful teachers.

C. Adults should teach children to win than to enjoy themselves.

D. Children easily imitate adults’ aggressive behaviors.

Câu hỏi 940 :

The word “self-esteem” in the passage is closest in meaning to ______.

A. confidence

B. education

C. independence

D. communication

Câu hỏi 941 :

The word “they” in the last paragraph refers to ______.

A. children

B. examples

C. sports

D. parents and coaches

Câu hỏi 942 :

What would probably NOT be done when “facing up to a problem”?

A. Ignoring the problem

B. Finding the reason.

C. Admitting there is a problem.

D. Looking for a solution.

Câu hỏi 943 :

Read the following passage and mark the letter A, B, C, or D on your answer sheet to indicate the correct answer to each of the questions from 36 to 42.

THE RING OF FIRE

     The Ring of Fire is an enormous chain of volcanoes all around the Pacific Ocean. The ring goes from New Zealand up to Asia and across the ocean to Alaska. From Alaska, the ring continues southward along the coast of both North and South America. More than seventy-five percent of the world’s volcanoes are situated in this ring.

      Scientists are interested in studying the Ring of Fire because they can observe plate tectonics at work there. In 1912, a German scientist, Alfred Wegener, came up with the first theory of land movement. Wegener said continents are made up of lighter rocks resting on heavier material. Similar to the way large things move while floating on water, Wegener suggested that the positions of the continents were not fixed, but that they moved slightly. Later, scientists discovered most of Wegener’s ideas were right on the mark. They then developed the theory called plate tectonics.

      According to plate tectonics, the surface of the Earth consists of a number of enormous plates or sections of rock, each about eighty kilometers thick. The plates float and slowly move at speeds between one and ten centimeters every year. That is about the rate your fingernails grow! Within the Ring of Fire, new material for the Earth’s plates is constantly being created as hot liquid rock called magma flows from the center of the Earth up to the ocean floor. All the existing plates on the Earth’s surface have to move slightly to make room for the new material.

      As plates move both away from and toward each other, they run into each other. When they hit each other, one plate might move under another. This process is called subduction. Subduction frequently causes earthquakes. It may also result in the bottom plate melting due to the extreme temperatures under the top plate. The magma created in this process can rise to the Earth’s surface and come out through volcanoes, as can be seen along the Ring of Fire.

What does the passage mainly discuss?

A. The most active volcanoes found in the Ring of Fire

B. The location of the Ring of Fire

C. How the plates on the Earth's surface move in different ways.

D. How plate tectonics, volcanoes, and earthquakes are related

Câu hỏi 944 :

The word “situated” in paragraph 1 is closest in meaning to __________.

A. exploded

B. located

C. examined

D. displaced

Câu hỏi 945 :

According to the passage, which is TRUE about the Ring of Fire?

A. All of the volcanoes along the ring are active.

B. Most of the volcanoes on Earth are part of the ring.

C. The ring gets bigger each year.

D. The ring was discovered in the 20th century.

Câu hỏi 946 :

The word “They” in paragraph 2 refers to _________.

A. plate tectonics

B. Wegener’s ideas

C. scientists


D. theories

Câu hỏi 947 :

What does the word "subduction" in paragraph 4 mean?

A. Rocks moving under volcanoes.

B. Erupting liquid rock.

C. The theory of moving plates

D. Movement of a plate under another.

Câu hỏi 948 :

What is NOT a result of shifting tectonic plates?

A. Earthquakes.

B. Extreme temperatures inside the Earth.

C. Volcanoes.

D. Subduction.

Câu hỏi 949 :

Which question is NOT answered in this passage?

A. How fast do tectonic plates move?

B. How thick are the plates in tectonic theory?

C. What is the most active volcano today?

D. Where is the Ring of Fire?

Câu hỏi 953 :

Mark the letter A, B, C, or D on your answer sheet to indicate the sentence that is closest in meaning to each of the following questions.
It’s a long time since I last saw my faraway grandparents.

A. I last saw my faraway grandparents for a long time

B. My grandparents was living faraway a long time ago.

C. I haven’t seen my faraway grandparents for a long time.

D. I haven’t seen my faraway grandparents since a long time.

Câu hỏi 954 :

“I won’t help you with your homework.” Jane said to me.



A. Jane promised to help me with my homework.


B. Jane refused to help me with my homework.

C. Jane asked me to help her with her homework.

D. Jane denied helping me with my homework.

Câu hỏi 955 :

Attending classes is compulsory for children from 5 in Britain.

A. Children from 5 in Britain mustn’t attend classes.

B. Children from 5 in Britain shouldn’t attend classes.

C. Children from 5 in Britain must attend classes.

D. Children from 5 in Britain may attend classes.

Câu hỏi 956 :

Mark the letter A, B, C, or D on your answer sheet to indicate the sentence that best combines each pair of sentences in the following questions.

Beatrix is not here. She is the only person who can tell me what to do now.


A. I wish Beatrix were here and told me what to do now.

B. Supposing Beatrix is here, she could tell me what to do now.

C. If only Beatrix had been here and told me what to do now.

D. If Beatrix were here, she couldn’t help me now.

Câu hỏi 957 :

Yuri Gagarin made a successful space flight in 1961. Human’s uncertainties about universe were enlightened only then.

A. Not until human’s uncertainties about universe were enlightened did Yuri Gagarin make a successful space flight in 1961.

B. Without human’s uncertainties about universe, Yuri Gagarin wouldn’t have made a successful space flight in 1961.

C. Only after Yuri Gagarin had made a successful space flight in 1961 were human’s uncertainties about universe enlightened.

D. Hardly had Yuri Gagarin made a successful space flight in 1961, human’s uncertainties about universe were enlightened.

Câu hỏi 963 :

Today many serious childhood diseases ________by early immunization.

A. are preventing

B. can prevent

C. prevent

D. can be prevented

Câu hỏi 964 :

I am well _________with the problem encountered in starting a business.

A. aware

B. informed

C. acquainted

D. knowledgeable

Câu hỏi 965 :

_______ in Stevenson’s landscape, the more vitality and character the paintings seem to possess.

A.The brushwork is loose

B. The loose brushwork

C. The loose brushwork is

D. The looser the brushwork is

Câu hỏi 966 :

My mother is a _____________lady who holds me in high esteem.

A. young tall beautiful

B. tall young beautiful

C. beautiful young tall

D. beautiful tall young

Câu hỏi 967 :

I saw an accident while I ________ my friend yesterday.

A. waited

B.  had waited

C. was being waited

D.  was waiting

Câu hỏi 968 :

__________hardship, the firemen managed to save many people who were caught in the fire.

A. In spite of

B. Although

C. due to

D. because

Câu hỏi 969 :

I will call you as soon as I ________in Paris.

A. arrive

B. will arrive

C. am arriving

D. will have arrived

Câu hỏi 970 :

_________help from many kind people , these poor children have got over difficulty.

A. Received

B. Having been received

C. To receive

D. Receiving

Câu hỏi 971 :

_________, There are black holes in space.

A. theorize

B. Theoretical

C. Theoretically

D. Theorist

Câu hỏi 972 :

You will have to _________ your holiday if you are too ill to travel.

A. call off

B. cut down

C. back out

D. put aside

Câu hỏi 975 :

He will have to __________and work harder or he’ll fail the exam.

A. pull his socks up

B. polish his head

C. empty his washing basket

D. stick his neck out

Câu hỏi 976 :

Switzerland is well –known for its impressive mountainous __________.

A. views

B. scenes

C. scenery

D. sights

Câu hỏi 980 :

Danny put the cat among the pigeons by suggesting that the company might have to make some redundancies.

A. made other people disappointed

B. made other people nervous

C. made a lot of people satisfied

D. made a lot of people annoyed

Câu hỏi 982 :

Peter : “ Need a hand with your suitcase , Jane ? ”

                          Jane :  “_________________”

A. Not a chance

B. That’s very kind of you

C. I don’t believe it

D. Well done !

Câu hỏi 983 :

Read the following passage and mark the letter A, B, C, or D on your answer sheet to indicate the correct word or phrase that best fits each of the numbered blanks from 26 to 30.

Almost 90 percent of American students below the college level attend public elementary and secondary schools, which do not charge tuition but rely on local and state taxes for funding. Traditionally, elementary school includes kindergarten through the eighth grade. In some places (26) ________, elementary school ends after the sixth grade, and students attend middle school, or junior high school, from grades seven through nine. Similarly, secondary school, or high school, traditionally comprises grades nine through twelve, but in ( 27 )_______places begins at the tenth grade. 

Most of the students who do not attend public elementary and secondary schools attend private schools, for which their families pay (28) ________. Four out of five private schools are run by religious groups. In these schools, religious instruction is part of the curriculum, (29) _________ also includes the traditional academic courses. There is also a small but growing number of parents who educate their children themselves, a practice known as home schooling. 

The United States does not have a national school system. Nor, with the exception of the military academies, are there schools run by the federal government. But the government (30) _______ guidance and funding for federal educational programs in which both public and private schools take part, and the U.S. Department of Education oversees these programs. 


A. whereas


B. moreover

C. therefore

D. however

Câu hỏi 988 :

Read the following passage and mark the letter A, B, C, or D on your answer sheet to indicate the correct answer to each of the questions from 31 to 35.

     Flirting is a fundamental fixture in the sexual behavior repertoire, a time-honored way of signaling interest and attraction. It is a kind of silent language spoken by men and women around the world. The ways people communicate interest are so deeply rooted in human nature that the signals are automatically understood by all: from ways of glancing to movements such as licking one's lips, to meet nature's most basic command—find a good mate and multiply.

Flirting is not a trivial activity; it requires many skills: intellect, body language, creativity, empathy. At its best, flirting can be high art, whether the flirter is vying for a soul mate, manipulating a potential customer, or just being playful.

The process of flirting allows the signaling of interest to another in small increments, which is especially appealing to a partner. Flirting is driven by emotions and instinct rather than logical thought. Yet the gestures and movements used in flirting also provide reliable clues to a person's biological and psychological health.

Flirting has many parallels in the animal world, seen in the behavioral displays many animals engage in to signal not only their availability but their suitability. For example, penguins search for small pebbles to deliver to their partner of interest. Seahorses lock their tails together for a romantic swim. Bower birds use leaves, grass, and twigs to construct elaborate nests.

Across the animal kingdom, such actions are closely tied to seasonal reproductive readiness and signal reproductive fitness by one creature to another. Among humans, the repertoire of flirting behaviors can be deployed at will and the exact meaning of any gesture is usually a matter of interpretation.

(Source: https://www.psychologytoday.com/)

What is the main topic of the passage?


A. Flirting


B. Charisma

C. Nonverbal cues

D. Romantic feelings

Câu hỏi 989 :

The word “vying” in paragraph 2 mostly means _______.

A. seeking

B. challenging

C. competing

D. clashing

Câu hỏi 990 :

The word “their” in paragraph 4 refers to _____.

A. nests’

B. pebbles’

C. penguins’

D. seahorses’

Câu hỏi 991 :

According to the passage, what can be suggested about the flirting of animals?

A. Animal courtship varies tremendously between species.

B. Birds prefer subtle movements to lavish displays.

C. Aquatic animals do not flirt due to their breeding habits.

D. Animals cannot feel the excitement of flirting like humans.

Câu hỏi 992 :

Which of the following statements is NOT mentioned in the passage?


A. The behavior of flirting is not restricted to humans.


B. Humans generally flirt with thinly veiled innuendoes.

C. All humans are equipped with the language of flirtation.

D. There is profound information transmitted in flirting.

Câu hỏi 993 :

Read the following passage and mark the letter A, B, C, or D on your answer sheet to indicate the correct answer to each of the questions from 36 to 42.

The Socialist Republic of Vietnam has stood out as one such example of how to effectively respond to the pandemic. Vietnam, which shares a border with China and is about 1,200 miles from where the outbreak was first reported in Wuhan, has overcome steep odds in the global fight against COVID-19. As of April 6, the Vietnamese government has reported 245 confirmed cases with 95 recoveries and no fatalities.

The country’s response to the outbreak has received international recognition, including from the World Health Organization and World Economic Forum, for its comprehensive, low-cost model of disease prevention. WHO representative in Vietnam, Dr. Ki Dong Park, attests the government “has always been proactive and prepared for necessary actions”.

The Communist Party of Vietnam has strengthened its anti-pandemic measures by implementing nationwide social distancing rules, such as banning outside gatherings of more than two people while keeping a distance of 6.5 feet, and temporary shutdowns of “non-essential” businesses, including restaurants, entertainment centers and tourist sites.

Supermarkets and other essential services remain open, but are instructed to safeguard customers’ health by checking their temperatures before entering the building and providing them with hand sanitizers. In addition, the government has warned against panic buying and has taken action against businesses engaged in price gouging. To ensure social security for affected workers, Vietnam has approved a 111.55 million dollar financial support package that includes covering all costs for workers in quarantine or are recovering from the disease.

Unlike the U.S. capitalist class and the Trump administration, the Vietnamese government took early measures to combat the current coronavirus epidemic. Officials began preparing strategies to combat the outbreak immediately after the first cases emerged in China.

On February 1, Prime Minister Nguyen Xuan Phuc signed Decision No.173.QD-TTG, categorizing the virus as a Class A contagious disease “that can transmit very rapidly and spread widely with high mortality rates.” This declaration of a national emergency came after the sixth case of coronavirus in the country was reported. In contrast, the Trump administration only declared a national emergency over the global pandemic on March 13, when there were at least 1,920 confirmed cases across 46 states.

(Adapted from https://vietnaminsider.vn/)

Which of the following best serves as the title for the passage?

A. Vietnam and the U.S in the campaign to combat the pandemic.

B. Why has no one in Vietnam died of COVID 19?

C. How does the pandemic affect Vietnam’s society?

D. The international recognition for Vietnam’s response to COVID 19.

Câu hỏi 994 :

Vietnam’s reaction to the epidemic was acknowledged as a ____________by the WHO and WEF.

A. comprehensible and economical model of disease prevention

B. perfect and expensive model of disease prevention

C. complete and inexpensive example of disease prevention

D. incomplete and luxurious example of disease prevention

Câu hỏi 995 :

Which of the following is NOT mentioned as a measure to the pandemic in Vietnam?

A. prohibiting outside gatherings

B. keeping a distance of 6.5 feet

C. closing unnecessary businesses permanently

D. warning inhabitants against panic buying

Câu hỏi 996 :

The word “their” in paragraph 4 refers to ___________.

A. supermarkets’

B. customers’

C. governments’

D. essential services’

Câu hỏi 997 :

What has the Vietnamese government done to help affected workers?


A. Check their temperatures and provide them with hand sanitizers.



B. Keep opening some essential services for their daily needs.


C. Support them in finance to help them ensure their daily lives.

D. Pay for their costs in quarantine or are getting over the disease.

Câu hỏi 998 :

The word “contagious” in the last paragraph is closest in meaning to ___________.

A. dangerous

B. fatal

C. hazardous

D. infectious

Câu hỏi 999 :

Which of the following is TRUE, according to the passage?

A. Vietnam implemented the measures to push back the outbreak after the first cases emerged in China.

B. The Trump administration declared a national emergency over the global pandemic earlier than the Vietnamese government did.


C. Vietnam has become the country that combats the epidemic most effectively.


D. The strategies to deal with the disease of Vietnam and the U.S are familiar.

Câu hỏi 1001 :

These students have been studying very hard for his coming examination.

A. These

B. have been studying

C. his

D. coming

Câu hỏi 1002 :

Kazakova's performance made her the heroin of the Moscow Film Festival.

A. Kazakova's

B. made

C. the heroin of

D. Film Festival

Câu hỏi 1003 :

Mark the letter A, B, C, or D on your answer sheet to indicate the sentence that is closest in meaning to each of the following questions.
The last time we went to the cinema was five years ago.

A. We didn’t go to the cinema five years ago.

B. We started going to the cinema five years ago.

C. We have never gone to the cinema for five years.

D. We haven’t gone to the cinema for five years.

Câu hỏi 1004 :

"Why don’t you get your hair cut , Gavin ? " said Adam.

A. Adam advised Gavin to cut his hair.

B. Gavin was suggested to have a haircut.

C. It was suggestible that Gavin should have his haircut.

D. Adam suggested that Gavin should have his haircut.

Câu hỏi 1005 :

It was a mistake for her to marry Peter.

A. She shouldn’t have married Peter.

B. She and Peter weren’t married in the right way.

C. Peter didn’t want to get married, so it was his mistake.

D. She ought to think again before she marries Peter.

Câu hỏi 1006 :

Mark the letter A, B, C, or D on your answer sheet to indicate the sentence that best combines each pair of sentences in the following questions

I regret agreeing to do the extra work.


A. I am sorry to say that I agreed to do the extra work.


B. I wish I didn’t take on the extra work.

C. I feel regret  now because I have done the extra work.

D. If only I hadn’t taken on the extra work.

Câu hỏi 1007 :

The play started as soon as we arrived at the theater.

A. No sooner we had arrived at the theater than the play started.

B. Hardly had we arrived at the theater when the play started.

C. Scarely had we arrived at the theater than the play started.

D. Not until we arrived at the theater did the play start.

Câu hỏi 1013 :

Do you believe that such a problem can ________?

A.  solve

B. be solving

C. is solved

D. be solved

Câu hỏi 1014 :

It is unusual ___________him to go out on a Monday night.

A. of

B. for

C. to

D. with

Câu hỏi 1015 :

The more she practices, __________she becomes.

A. the greater confidence

B. more confidently

C. the more confident

D. the most confident

Câu hỏi 1016 :

I was given __________book on my last birthday.

A. a new interesting English

B. an English new interesting

C. an interesting English new

D. an interesting new English

Câu hỏi 1017 :

The phone rang suddenly while Joanna _________her homework.

A. did

B. was doing

C. had done

D. is doing

Câu hỏi 1019 :

Your father will be furious ____________.

A. When he will see what you do

B. when he sees what you have done

C. when he sees what you had done

D. when he will see what you will do

Câu hỏi 1020 :

One of the ___________related to weddings is that the bride should wear something blue.

A. superstition

B. superstitious

C. superstitiously

D. superstitions

Câu hỏi 1021 :

__________10 years ago, This car is still my favorite.

A. Having bought

B. Buying

C. bought

D. Having been bought

Câu hỏi 1022 :

Although his first book was a roaring success, his later ones never really ______________.

A. took in

B. took on

C. took up

D. took off

Câu hỏi 1024 :

Thanks to __________ provision, many women who wish to work are capable of doing so.

A. childbearing

B. childrearing

C. childcare

D. child-centred

Câu hỏi 1025 :

I ____________ the impression that he was lying to all of us.

A. made

B. created

C. got

D. did

Câu hỏi 1026 :

Be honest! Don't let success go to your ____________.

A. head

B. hairs

C. eyes

D. mouth

Câu hỏi 1030 :

I am just a cog in the machine in this organization as my contribution to it is still minor.

A. negligible

B. pivotal

C. of little importance

D. of complete indifference

Câu hỏi 1033 :

“Have you seen the movie Joker?” – “________________.”

A. Of course! I’m going to see it on Monday!

B. I haven’t had a chance to see it.

C. The cinema is closed today.

D. Yes, the tickets were too expensive

Câu hỏi 1038 :

Read the following passage and mark the letter A, B, C, or D on your answer sheet to indicate the correct answer to each of the questions from 31 to 35.

In the early 1800s, less than 3% of the world's population lived in cities; today, more than half of the global population is urban and by 2050, the proportion will rise to three quarters. There are thousands of small and medium-sized cities along with more than 30 megacities and sprawling, networked metropolitan areas — conurbations — with 15 million residents or more. Yet despite these massive transformations in how people live and interact, our international affairs are still largely dictated by nation states, not cities.

     Cities are beginning to flex their muscles on the international stage. They are already displacing nation states as the central nodes of the global economy, generating close to 80% of global GDP. Cities like New York and Tokyo are bigger in GDP terms than many G-20 countries.

     Metropolitan regions and special economic zones are linking global cities through transnational supply chains. A growing number of mega-regions, such as those linking cities in Mexico and the US, transcend borders. In the process, cities are collectively forging common regional plans, trading partnerships, and infrastructure corridors.

The spectacular rise of cities did not happen by accident. Cities channel creativity, connect human capital, and when well governed, they drive growth. That many cities and their residents are rolling up their sleeves and getting things done — where nations have failed — are grounds for optimism. In the future, we hope that it is our proximate, accountable, and empowered city leaders who will define our fates.

What is the passage mainly about?


A. History of cities all over the world


B. Types of cities in the world

C. Contributions of cities to the world

D. The not-to-distant future of cities

Câu hỏi 1039 :

Which of the following is NOT true according to the writer?

A. Humanity transitioned from a rural to a primarily urban species at breathtaking speed.

B. Cities are the dominant form of human civilization in the 21st century.

C. There is a tendency for cities to connect to form large urban regions.

D. People should not be too optimistic about the future of cities.

Câu hỏi 1040 :

The phrase "flex their muscles" in paragraph 2 is closest in meaning to ____.

A. show their strength

B. change their attitudes

C. leave their side

D. find their ways

Câu hỏi 1041 :

Would the following sentence best be placed at the end of which paragraph? This is neither fair nor rational.

A. Paragraph 1

B. Paragraph 2

C. Paragraph 3

D. Paragraph 4

Câu hỏi 1042 :

The word "they” in paragraph 4 refers to ___________.

A. plans

B. partnerships

C. cities

D. residents

Câu hỏi 1043 :

Read the following passage and mark the letter A, B, C, or D on your answer sheet to indicate the correct answer to each of the questions from 36 to 42.

For hundreds of years, giving flowers have been a social means of communication. In the United States, flowers are often given during rites of passage, for commemorating special occasions or as a heartfelt gift between loved ones and friends. Flower gifting also occurs in most countries around the world. However, the meanings and traditions often vary.

While students traditionally gave their favorite teacher an apple in past years, in China, teachers are given flowers. Peonies are by far the flower most often given in China. They are also quite popularly used for weddings. Strangely, potted plants are not considered a pleasant gift among Asian cultures. The people believe that like a plant confined by a pot, the gift symbolizes a binding or restriction.

In Russia, in lieu of giving birthday presents, the guest of honor receives a single flower or an unwrapped bouquet. Floral arrangements or baskets are not given. Russians celebrate a holiday known as Woman’s Day. Traditional gifts include red roses, hyacinths or tulips. When there is a funeral or other occasion where someone wishes to express sympathy, carnations, lilies or roses are given in circular configurations, which signify the transition of birth, life and death to rebirth. In this instance, the color of choice is commonly yellow. For joyous occasions, arrangements and bouquets generally contain an odd number of flowers.

In the times of ancient Rome, brides carried flowers to scare away evil spirits and encourage fertility. The Dutch believed that flowers were food for the soul. When invited to someone’s home in Great Britain, it is tradition to bring a gift of flowers. All types are acceptable except white lilies, which are usually seen at funerals. Not unlike the United States, red roses are a symbol of love. Flowers are generally gifted in odd numbered increments regardless of the occasion. However, the Brits also have superstitions regarding the number 13, so the number is avoided.

In the southern region of the continent, flowers are traditionally given during Christmas. Egyptians are much more conservative and restrict flower gifting to funerals and weddings. While certain flowers may have significant meanings for some, flowers in Las Vegas and across the United States flowers are an accepted gift for any reason desired.

(Source: http://www.flowersofthefieldlv.com/ )

What does the topic mainly discuss?


A. The fascinating tradition of giving flowers.


B. The different meaning of flowers in different cultures.

C. The comparison of giving flowers between Asian and European cultures.

D. The kinds of flowers people often give others in different cultures.

Câu hỏi 1044 :

What does the word “They” in paragraph 2 refer to?

A. Students

B. Teachers

C. Flowers

D. Peonies

Câu hỏi 1045 :

Why should not you give a potted plant to an Asian?

A. Because the Asian prefer to be given flowers.

B. As this gift is often given at weddings in Asia.

C. Since this gift is believed to symbolize a binding and limitation in Asia.

D. Because the Asian students like to give an apple or flowers to others.

Câu hỏi 1046 :

According to the passage, the following flowers are given at Woman’s Day in Russia, EXCEPT ________.

A. red roses

B. hyacinths

C. tulips

D. yellow roses

Câu hỏi 1047 :

What could the word “fertility” in paragraph 4 best be replaced by?

A. fecundity

B. good spirit

C. happiness

D. loyalty

Câu hỏi 1048 :

The word “superstitions” in paragraph 4 is closest in meaning to __________.

A. deep-seated belief

B. unfounded belief

C. religious belief

D. traditional belief

Câu hỏi 1049 :

It can be inferred from the passage that _______________.

A. People can give flowers to the American in any occasion.

B. Egyptians are rather comfortable when receiving flowers at funerals and weddings.

C. Flowers given in Britain are in even numbers in any case.

D. At the funerals in any cultures, flowers are gifted in circular configurations.

Câu hỏi 1053 :

Mark the letter A, B, C, or D on your answer sheet to indicate the sentence that is closest in meaning to each of the following questions
I have not played tennis for two years.

A. I have never played tennis before

B. About two years ago, I last played tennis

C. The last time I played tennis was two years ago

D. I started played tennis two years ago

Câu hỏi 1054 :

“I will fix your bike tomorrow” Paul said to his son.

A. Paul said that he would fix his son’s bike tomorrow.

B. Paul said that he will fix his son’s bike the following day.

C. Paul said that he would fix his son’s bike next day.

D. Paul promised to fix his son’s bike the following day.

Câu hỏi 1055 :

In most developed countries, it is not necessary for people to boil water before they drink it.

A. In most developed countries, people ought to boil water before they drink it.

B. In most developed countries, people don’t need to boil water before they drink it.

C. In most developed countries, people should have boiled water before they drink it.

D. In most developed countries, people mustn’t boil water before they drink it.

Câu hỏi 1056 :

Mark the letter A, B, C, or D on your answer sheet to indicate the sentence that best combines each pair of sentences in the following questions
I gave her my phone number . I regret it now.

A. I should have given her my phone number.

B. If only I had given her my phone number.

C. If only I had not given her my phone number.

D. I wish I could give her my phone number.

Câu hỏi 1057 :

She is the winner of the beauty pageant. That is the only reason people pay attention to her.

A. Were she the winner of the beauty pageant, people would pay attention to her.

B. Should she be the winner of the beauty pageant, people will pay attention to her.

C. If it hadn’t been for the fact that she was the winner of the beauty pageant, people wouldn’t have paid attention to her.

D. If it weren't for the fact that she is the winner of the beauty pageant, people wouldn’t pay attention to her.

Câu hỏi 1063 :

My cousin will put me up here ___________________.


A. after I will have found reasonable accommodation to stay

B. by the time I found reasonable accommodation to stay

C. when I had found reasonable accommodation to stay

D. until I find reasonable accommodation to stay

Câu hỏi 1065 :

I can’t find my wallet anywhere. It ___________.

A. were stolen

B. stole

C. have been stolen

D. must have been stolen

Câu hỏi 1066 :

No one even noticed when I got home because they _________ the big game on TV.

A. are watching

B. watched

C. were watching

D. had watched

Câu hỏi 1067 :

My grandfather is really into collecting unique things. He bought a __________ chair and put it in the living room.

A. red strange plastic

B. red plastic strange

C. strange red plastic

D. plastic red strange

Câu hỏi 1068 :

___________, the more boring the debate becomes.

A. The complicated are the questions

B. The questions are more complicated

C. The complicated questions are

D. The more complicated the questions are

Câu hỏi 1069 :

_________ economic boom last year, the business was successful.

A. Due to

B. Because

C. Although

D. Despite

Câu hỏi 1070 :

Holidays where people carry out voluntary charity work are becoming ________ popular.

A. increasing

B. increased

C. increasingly

D. increase

Câu hỏi 1071 :

Because of this economic downturn, we may have to _________ up to 100 people.

A. take on

B. put off

C. lay off

D. turn down

Câu hỏi 1072 :

__________ a secret island, they started searching for the treasure.

A. To find

B. Finding

C. Having found

D. Being found

Câu hỏi 1073 :

Phillip was born into poverty, but made an absolute ___ by the time he was thirty.

A. fortune

B. profit

C. progress

D. failure

Câu hỏi 1074 :

I don’t want to argue with him again. It’s better to let sleeping ________ lie.

A. elephants

B. crocodiles

C. dogs

D. cats

Câu hỏi 1075 :

He ___________ guilty to the crime so he got a lighter sentence.

A. pled

B. read

C. claimed

D. announced

Câu hỏi 1078 :

You’re flogging a dead horse trying to persuade him to come with us. He hates going out at night.

A. beat the horse that isn’t alive

B. finish something before deadline

C. do something without effort

D. try to do something impossible

Câu hỏi 1080 :

I really made a hash of that project at work. I'm going to get fired for sure!

A. spoiled

B. ruined

C. revealed

D. improved

Câu hỏi 1081 :

Mark the letter A, B, C, or D on your answer sheet to indicate the option that best completes each of the following exchanges.
William and Laura are talking about Covid-19 pandemic.

A. We can do nothing for it.

B. Yes, everyone should be aware of this epidemic

C. Stay 6 feet away from others.

D. No problem.

Câu hỏi 1082 :

Charles and Adam are talking about learning online.

- Charles: “There is no problem with online learning.

- Adam: “_____. Looking at the screen most of the time can make students’ eyes tired.”

A. There is no doubt about it

B. You’re dead wrong

C. Well, that's very surprising

D. That’s exactly what I was thinking.

Câu hỏi 1083 :

Read the following passage and mark the letter A, B, C, or D on your answer sheet to indicate the correct word or phrase that best fits each of the numbered blanks.

     Because of mobile phones’ explosion in popularity, many schools have opposed, or even banned, their use in the classroom. Yet an increasing ___(26) ___ of educators are now turning to cell phones in order to bolster student engagement and learning. ___(27) ___ mobile-phone use should be limited to situations (28) _______ the technology genuinely enhances instruction, the advanced functionality of today's cell phones makes them ideal for 2 1st century lessons. For example, many schools use the devices to poll students in class via text message.

     Some schools encourage students to use their phones Cameras to snap pictures for use as inspiration in art classes. Others allow students to capture photographs of a post-lecture whiteboard. In seconds, kids car grab a detailed image of a teacher's notes on the board and email it themselves, resulting in incredible study tool.

     On field ___(29) ___, students can participate in scavenger hunts by snapping pictures of items on a teacher-supplied list. Mobile phones can also help facilitate class discussion. Teachers might ask students to run a Google Search for information on a particular topic. An alternative to presenting the information via lecture or having students read it from a textbook, discussion-integrated Web searchers are ___(30)___ more engaging, increasing the adds that students will retain the information.

A. number

B. amount

C. lot

D. few

Câu hỏi 1088 :

Read the following passage and mark the letter A, B, C, or D on your answer sheet to indicate the correct answer to each of the questions from 31 to 35.

    Have you ever heard some superstitions about birds? Usually, birds are considered bad luck, or even a sign of imminent death. For example, all over the world, both crows and ravens have some connection to war, and death. In early times, crows and ravens were thought to accompany the gods of war, or be signs of the gods’ approaching arrival. This idea later changed. Crows in particular were thought to be harbingers of ill fortune or, in some cases, guides to the afterlife. Woe be it to the person who saw a single crow or raven flying overhead, for this was most certainly a portent of death in the near future.

       Interestingly, though potentially bad luck for people individually, the raven is considered to be good luck for the crown of England. So much so, in fact, that a “raven master” is, even today, an actual government position in London. He takes care of the ravens there and also clips their wings, ensuring that these birds can never fly far from the seat of the British government. This way, the kingdom will never fall to ill fortune.

      Another bird that is thought to play a part in forecasting the fortunes of people is the swallow. Depending on how and when it is seen, the swallow can be a harbinger of either good or ill fortune. Perhaps inspired by the swallow’s red-brown breast, Christian people initially related the swallow to the death of Jesus Christ. Thus, people who saw a swallow fly through their house considered it a portent of death. Later, however, farmers began to consider swallows signs of good fortune. Any barn that has swallows living in it is sure to be blessed in the following year. Farmers also have to beware of killing a swallow; that would be certain to end any good luck they might have had.

      Though many people think these superstitions are old wives’ tales, there is actually some evidence to support them. For example, crows and ravens, being scavengers, appear at the aftermath of battles. Thus, large numbers of crows and ravens could be good indications of war in an area. As well, swallows feed on insects that can cause infections in cattle. Thus, a farmer who has many swallows in his barn may actually have healthier animals on his farm. Therefore, the next time you feel inclined to laugh at an old wives’ tale, maybe you had better find out if there is any truth to it first!

What is the main idea of this reading?

A. Birds bring bad luck.

B. Don’t look a crow in the eye.

C. There are many superstitions surrounding birds.

D. Birds are important to English people.

Câu hỏi 1089 :

The word "portent" in the first paragraph is closest in meaning to ___________.

A. sign

B. coming

C. symbol

D. destruction

Câu hỏi 1090 :

Which is NOT true, according to the reading?

A. Some superstitions are based on reality

B. Seeing a swallow is a sign of war and death in the future.

C. Ravens bring good luck for the country of England.

D. Crows eat dead animals.

Câu hỏi 1091 :

The word "old wives’ tales" in paragraph 4 is closest in meaning to ___________.

A. stories told by housewives in the past.

B. old beliefs which have been proved to be scientifically true.

C. folk beliefs passed down from generation to generation without any reason of science.

D. folk tales about wives that were passed on to people in a spoken form.

Câu hỏi 1092 :

Read the following passage and mark the letter A, B, C, or D on your answer sheet to indicate the correct answer to each of the questions from 36 to 42.

        Most people currently realize the danger of using credit card numbers online. However, from time to time, we all use passwords and government ID numbers on the Internet. We think we are safe, but that may not be true! A new kind of attack is being used by dishonest people to steal IDs and credit card numbers from innocent web surfers. This new kind of attack is called “phishing”.

       Phishing sounds the same as the word “fishing”, and it implies that a thief is trying to lure people into giving away valuable information. Like real fishermen, phishers use bait in the form of great online deals or services. For example, phishers might use fake emails and false websites to con people into revealing credit card numbers, account usernames, and passwords. They imitate well-known banks, online sellers, and credit card companies. Successful phishers may convince as many as five percent of the people they contact to respond and give away their personal financial information.

       Is this really a big problem? Actually, tricking five percent of the online population is huge! Currently, more than 350 million people have access to the Internet, and seventy-five percent of those Internet users live in the wealthiest countries on Earth. It has been estimated that phishers send more than three billion scam messages each year. Even by tricking only five percent of the people, phishers can make a lot of money.

        Since there is so much money to make through this kind of scam, it has caught the interest of more than just small-time crooks. Recently, police tracked down members of an organized phishing group in Eastern Europe, who had stolen hundreds of thousands of dollars from people online. The group created official-looking email messages requesting people to update their personal information at an international bank’s website. However, the link to the bank in the message actually sent people to the phishers’ fake website. To make matters worse, further investigation revealed that this group had connections to a major crime gang in Russia.

       How can innocent people protect themselves? Above all, they have to learn to recognize email that has been sent by a phisher. Always be wary of any email with urgent requests for personal financial information. Phishers typically write upsetting or exciting, but fake, statements in their emails so that people will reply right away.

       Also, messages from phishers will not address recipients by name because they really don’t know who the recipients are yet. On the other hand, valid messages from your bank or other companies you normally deal with will typically include your personal name.

Which of the following could best serve as the topic of the passage?

A. Innocent web surfers and online risks.

B. A new type of internet attack.

C. Valuable information shared on the internet.

D. Password and government ID numbers on the Internet.

Câu hỏi 1093 :

How might a farmer attempt to prevent bad luck?

A. Catch a raven.

B. Keep his windows open.

C. Follow a crow.

D. Avoid harming a swallow.

Câu hỏi 1094 :

What does the word "lure" in paragraph 2 mostly means _________?

A. attract

B. entice

C. avert

D. obviate

Câu hỏi 1095 :

The word “they” in paragraph 2 refers to ___________.

A. fake emails and false websites

B. phisher

C. credit card number

D. people

Câu hỏi 1096 :

According to paragraph 3, why are only 5% of online users tricked a big problem?

A. Because these 5% of internet users are the wealthiest people among online population.

B. Because the personal information these people are tricked is financial.

C. Because the number of online population is myriad and three fourths of them live in rich nation.

D. Because the number of scam messages sent to these users is more than three billion.

Câu hỏi 1097 :

The word “revealed” in paragraph 4 mostly means ___________.

A. determined

B. concealed

C. suppressed

D. showed

Câu hỏi 1098 :

What suggestion is TRUE according to the last paragraph?

A. Ask your bank or company if you receive any upsetting or exciting email.

B. You shouldn't answer any message relate to your personal financial information.

C. Be cautious with any email without the name sender.

D. Keep calm and be careful with urgent financial messages without your name.

Câu hỏi 1099 :

What can be inferred from the passage?

A. Successful phishers may steal people's personal information from false international banks, online sellers and credit card companies.

B. Phishers mostly steal personal information of the wealthiest people.

C. Recognizing the differences between the true and Take messages will help innocent people protect their information.

D. It's not easy to track down phishers since their website are false.

Câu hỏi 1103 :

‘Can I give you a hand with this report?’ said my colleague.


A. My colleague asked me if I could give him a hand with that report.

B. My colleague told me to help him with that report.

C. My colleague refuse to give me a hand with that report.

D. My colleague offered to help me with that report.

Câu hỏi 1104 :

Mark the letter A, B, C, or D on your answer sheet to indicate the sentence that is closest in meaning to each

The last time my friend wrote a letter to me was in February.

A. I haven't never received a letter from my friend since February.

B. I last received a letter from my friend in February.

C. It was in February since I first received a letter from my friend.

D. My friend last wrote a letter to me when in February.

Câu hỏi 1105 :

I’m sure he worked his socks off to get to the conference in time.

A. He must work his socks off to get to the conference in time.

B. He needn’t work his socks off to get to the conference in time.

C. He should have worked his socks off to get to the conference in time.

D. He must have worked his socks off to get to the conference in time.

Câu hỏi 1106 :

Mark the letter A, B, C, or D on your answer sheet to indicate the sentence that best combines each pair of sentences in the following questions.

My two younger brothers ate so much chocolate after lunch. They’re feeling very sick now.

A. If my two younger brothers hadn’t eaten so much chocolate after lunch, they wouldn’t have been very sick now.

B. If only my two younger brothers hadn’t eaten so much chocolate after lunch.

C. I wish my two younger brothers weren’t feeling very sick because of eating so much chocolate after lunch.

D. But for eating so much chocolate after lunch, my two younger brothers wouldn’t have been very sick now.

Câu hỏi 1107 :

Pollution is a significant problem all around the world. Many people are debating possible solutions.


A. Such is pollution a significant problem all around the world that many people are debating possible solutions.

B. Not until pollution is a significant problem all around the world are many people debating possible solutions.

C. It is only when pollution is a significant problem all around the world are many people debating possible solutions.

D. Significant as problem is all around the world, many people are debating possible solutions.

Câu hỏi 1113 :

I locked the door of my room all day yesterday to avoid _______.

A. disturbing

B. being disturbed

C. to disturb

D. to be disturbed

Câu hỏi 1115 :

The more polite you appear to be, _______ your partner will be.

A. the happiest

B. the happier

C. the most happily

D. the more happily

Câu hỏi 1116 :

At first sight I met her, I was impressed with her______.

A. big beautiful round black eyes

B. beautiful black big round eyes

C. beautiful big round black eyes

D. beautiful round big black eyes

Câu hỏi 1117 :

She hurt herself while she _______ hide-and-seek with her friends.

A. is playing

B. had played

C. played

D. was playing

Câu hỏi 1119 :

They will have suffered from coldness and hunger for 6 hours _______ them on the mountain.

A. by the time we find

B. when we found

C. as soon as we had found

D. after we had found

Câu hỏi 1120 :

_______ for about 4 weeks, we now can perform most of the tasks confidently.

A. Being trained

B. Training

C. Having trained

D. Having been trained

Câu hỏi 1122 :

You shouldn’t lose heart; success often comes to those who are not _______ by failures

A. put off

B. turned on

C. left out

D. switched off

Câu hỏi 1124 :

I was left out in the _______ in the annual promotions in the company.

A. rain

B. sun

C. snow

D. cold

Câu hỏi 1128 :

I'm all in favor of ambition but I think when he says he'll be a millionaire by the time he's 25, he's simply crying for the moon.

A. longing for what is beyond the reach

B. asking for what is attainable

C. doing something with vigor or intensity

D. crying a lot and for a long time

Câu hỏi 1130 :

He is very stubborn but flattery usually works like a charm on him.

A. is completely successful in

B. takes effect

C. turned out to be a disaster

D. has no effect on

Câu hỏi 1133 :

Read the following passage and mark the letter A, B, C, or D to indicate the correct word or phrase that best fits each the numbered blanks.

A scientist said robots will be more intelligent than humans by 2029. The scientist's name is Ray Kurzweil. He works for Google as Director of Engineering. He is one of the world’s leading experts on (26) _______ intelligence (A.I). Mr Kurzweil believes computers will be able to learn from experiences, just like humans. He also thinks they will be able to tell jokes and stories, and even flirt. Kurzweil‘s 2029 prediction is a lot sooner than many people thought. The scientist said that in 1999, many A.I. experts said it would be hundreds of years (27) _______ a computer was more intelligent than a human. He said that it would not be long before computer (28) _______ is one billion times more powerful than the human brain.

Mr Kurzweil joked that many years ago, people thought he was a little crazy for predicting computers would be as intelligent as humans. His thinking has stayed the same but everyone else has changed the way they think. He said: “My views are not radical any more. I've actually stayed (29) _______. It's the rest of the world that's changing its View.” He highlighted examples of high-tech things we use, see or read about every day. These things make us believe that computers have intelligence. He said people think differently now: "Because the public has seen things like Siri (the iPhone’s voice-recognition technology) (30) _______ you talk to a computer, they've seen the Google self-driving cars."

                                                                                         (Source: https://breakingnewsenglish.com)


A. artificial


B. false

C. handmade

D. fake

Câu hỏi 1135 :

He said that it would not be long before computer (28) _______ is one billion times more powerful than the human brain.

A. intelligent

B. intelligently

C. intelligence

D. intelligences

Câu hỏi 1138 :

Read the following passage and mark the letter A, B, C, or D to indicate the answer to each of the question.

      Anthropogenic global warming is a theory explaining today's long-term increase in the average temperature of Earth's atmosphere as an effect of human industry and agriculture.

      Since the latter half of the 20th century, growing banks of data and improved climate models have convinced most climate scientists that rising trends in greenhouse gas emissions are directly responsible for a rising trend in atmospheric temperature. The source of these emissions varies, consisting of a mix of gases that include methane and carbon dioxide. While some sources - such as volcanoes - are natural, their overall emissions compared with those produced by human industries, transport, and livestock have been regarded as insignificant over recent centuries.

       Greenhouse gases are made of molecules that absorb electromagnetic radiation, such as the light reflecting from the planet's surface, and re-emit it as heat. These gases include methane, carbon dioxide, water, and nitrous oxide. Despite making up only a small percentage of the atmosphere's mix of gases they are very important. If we had no naturally occurring greenhouse gases in the atmosphere at all, the average global temperature on Earth would be a much lower -18 degrees instead of the roughly 15 degrees Celsius we have enjoyed most of human history.

        Carbon dioxide levels have steadily risen over the past two centuries, thanks largely to the burning of fossil fuels for electricity general, transportation, and smelting. Current levels are approximately 415 parts per million (ppm), up from pre-industrial levels of around 280 ppm. Models vary in their predictions of further temperature increases, and depend heavily on future trends in greenhouse gas emissions. Conservative estimates by the Intergovernmental Panel on Climate Change predict an even chance of 4 degrees Celsius rise by the end of the century if current emissions trends continue.

                                                                                                     (Source: https://www.sciencealert.com/)

Where is the passage possibly taken from?


A. An official business letter.


B. A brochure for ecotourism.

C. An environmental magazine.

D. A questionnaire at the mall.

Câu hỏi 1139 :

The word “livestock” in paragraph 2 is closest in meaning to _______.

A. beast

B. animal

C. cattle

D. poultry

Câu hỏi 1140 :

According to paragraph 3, what would happen if the Earth were bereft of natural greenhouse gases?

A. The Earth’s temperature would be too cold for the habitation of any living things.

B. The sunny weather would be a permanent setting for the Earth.

C. Most of the Earth’s surface would crack due to the scorching heat.

D. No noticeable changes in the climate would arise.

Câu hỏi 1141 :

According to paragraph 4, what are the CO2-producing fossil fuels utilized for?

A. Fossil fuels, as the main energy source, generate a significant share of electricity.

B. The oil and natural gas industry supply a massive number of jobs.

C. Coals provide the right amount of heat for extracting metal from its ore.

D. Petroleum and other derivatives power various means of transport.

Câu hỏi 1142 :

The word “those” in paragraph refers to _______.

A. levels

B. models

C. sources

D. emissions

Câu hỏi 1143 :

Read the following passage and mark the letter A, B, C, or D to indicate the answer to each of the question.

While watching sports on TV, the chances are children will see professional players cheating, having tantrums, fighting, or abusing officials. In addition, it’s highly likely that children will be aware of well-known cases of sportspeople being caught using drugs to improve their performance. The danger of all this is that it could give children the idea that winning is all that counts and you should win at all costs. Good behavior and fair play aren’t the message that comes across. Instead, it looks as if cheating and bad behavior are reasonable ways of getting what you want. This message is further bolstered by the fact that some of these sportspeople acquire enormous fame and wealth, making it seem they are being handsomely rewarded either despite or because of their bad behavior.

What can parents do about this? They can regard sport on television as an opportunity to discuss attitudes and behavior with their children. When watching sports together, if parents see a player swearing at the referee, they can get the child’s opinion on that behavior and discuss whether a player’s skill is more important than their behavior. Ask what the child thinks the player’s contribution to the team is. Point out that no player can win a team game on their own, so it’s important for members to work well together.

Another thing to focus on is what the commentators say. Do they frown on bad behavior from players, think it’s amusing or even consider it’s a good thing? What about the officials? If they let players get away with a clear foul, parents can discuss with children whether this is right and what effect it has on the game. Look too at the reactions of coaches and managers. Do they accept losing with good grace or scowl and show a bad attitude? Parents can use this to talk about attitudes to winning and losing and to remind children that both are part of sport.

However, what children learn from watching sports is by no means all negative and parents should make sure they accentuate the positives too. They should emphasise to children the high reputation that well-behaved players have, not just with their teammates but also with spectators and the media. They can focus on the contribution made by such players during a game, discussing how valuable they are in the team. In the interviews after a game, point out to a child that the well-behaved sportspeople don’t gloat when they win or sulk when they lose. And parents can stress how well these people conduct themselves in their personal lives and the good work they do for others when not playing. In other words, parents should get their children to focus on the positive role models, rather than the antics of the badly behaved but often more publicized players.

                                                              (Adapter from “New English File – Advanced” by Will Maddox)

Which of the following does the passage mainly discuss?


A. The importance of team spirit in sport


B. The influence of model sportspeople on children

C. Moral lessons for children from watching sports

D. Different attitudes toward bad behavior in sport

Câu hỏi 1144 :

The word “bolstered” in paragraph 1 is closest in meaning to _______.

A. inspired.

B. represented.

C. energized.

D. reinforced.

Câu hỏi 1145 :

According to paragraph 2, what should parents teach their children through watching sports?

A. Cheating is frowned upon by the majority of players.

B. A team with badly-behaved players will not win a game.

C. A player’s performance is of greater value than his behavior.

D. Collaboration is fundamental to any team’s success.

Câu hỏi 1146 :

The word “accentuate” in paragraph 4 can be best replaced by _______.

A. highlight.

B. embolden.

C. consolidate.

D. actualize.

Câu hỏi 1147 :

The word “They” in paragraph 4 refers to _______.

A. children.

B. spectators.

C. teammates.

D. parents.

Câu hỏi 1148 :

Which of the following about sport is NOT mentioned in the passage?

A. Misconduct from sportspeople may go unpunished despite the presence of officials.

B. A well-behaved player enjoys a good reputation among his teammates, spectators and the media.

C. Reactions of coaches and managers when their teams lose a game may be of educational value.

D. Many sportspeople help others so as to project good images of themselves.

Câu hỏi 1149 :

Which of the following can be inferred from the passage?

A. The media tend to turn the spotlight more on sportspeople’s wrongdoings than on their good deeds.

B. The well-behaved players in a game invariably display desirable conducts when not playing.

C. Players with good attitudes make a greater contribution to their teams’ budgets than others.

D. Well-mannered players sometimes display strong emotions after winning or losing a game.

Câu hỏi 1153 :

Mark the letter A, B, C, or D to indicate the sentence that is closest in meaning to each of the following questions.

She has taught the children in this remote village for five months.


A. She started to teach the children in this remote village five months ago.


B. She was the teacher in this remote village for months ago.

C.  She started to teach the children in this remote village for five months

D. The last time she taught the children in this remote village for five months ago

Câu hỏi 1154 :

Tom told his girlfriend, "Let me tell your parents about your health if you can't."

A. Tom advised his girlfriend to tell her parents about her health if you couldn't.

B. Tom suggested that I should tell her parents about her health if his girlfriend couldn't.

C. Tom volunteered to tell his girlfriend’s parents about her health if she couldn't.

D. Tom asked his girlfriend to tell her parents about her health if you couldn't.

Câu hỏi 1155 :

I’m certain Luis was on top of the world when his wife gave birth to their first child.

A. Luisa must be on top of the world when his wife gave birth to their first child.

B. Luisa must have been on top of the world when his wife gave birth to their first child.

C. Luisa could have been on top of the world when his wife gave birth to their first child.

D. Luisa may be very on top of the world when his wife gave birth to their first child.

Câu hỏi 1156 :

Mark the letter A, B, C, or D to indicate the sentence that best combines each pair of sentences in the following questions.

My neighbours often sing karaoke in the evening. It is noisy.


A. If only my neighbours hadn’t sung karaoke in the evening.


B. If my neighbours hadn’t sung karaoke in the evening, it wouldn’t have been noisy.

C. I wish my neighbours didn’t often sing karaoke in the evening.

D. As long as my neighbours don’t sing karaoke in the evening, it won’t be noisy. 

Câu hỏi 1157 :

Smoking is unhealthy. It also costs governments billions of dollars in health-care costs.

A. Not only is smoking unhealthy but it also costs government billions of dollars in health-care costs.

B. Whereas smoking is unhealthy, it costs government billions of dollars in health-care costs.

C. If smoking is unhealthy, it will cost government billions of dollars in health-care costs.

D. Smoking is so unhealthy that it costs government billions of dollars in health-care costs.

Câu hỏi 1163 :

___________________, he will take a gap year to do some voluntary work at mountainous areas.

A. Until he graduates from high school

B. When he had graduated from high school

C. As soon as he has graduated from high school

D. After he had graduated from high school

Câu hỏi 1165 :

We ________ at the breakfast table when the doorbell rang.

A. were sat

B. be sitting

C. sat

D. were sitting

Câu hỏi 1166 :

When we reached the airport, we found that all the flights____ due to the storm.

A. postponed

B. had postponed

C. was postponed

D. had been postponed

Câu hỏi 1167 :

I am going to wear my ________ tie to the wedding.

A. big cotton blue

B. blue big cotton

C. big blue cotton

D. cotton big blue

Câu hỏi 1168 :

The richer a person is, ______________.

A. the more they enjoy privilege

B. the more privilege they enjoy

C. the more privilege did they enjoy

D. they enjoy more privilege

Câu hỏi 1169 :

The Alien films are frightening ______ the terrible monsters.

A. in spite of

B. because

C. although

D. because of

Câu hỏi 1171 :

______________ around on the meadow all day, the sheep were tired.

A. Having ran

B. Running

C. To run

D. Having run

Câu hỏi 1172 :

This theory is quite difficult to take in. Could you __________for me?

A. break it down

B. get it across

C. put it up

D. turn it off

Câu hỏi 1173 :

This message is ______. Please reply immediately.

A. dishonest

B. duplicate

C. rustling

D. urgent

Câu hỏi 1180 :

He was jumping the gun when he gave up his job to start university. Now he has found out he can't get a grant.

A. shooting someone with his gun

B. finishing something too soon

C. doing something at the right time

D. doing something with enthusiasm

Câu hỏi 1183 :

Read the following passage and mark the letter A, B, C, or D on your answer sheet to indicate the correct word or phrase that best fits each of the numbered blanks.

        A university professor in Singapore gave a two-hour online lecture (26) _______ he didn't realize he was on mute. Professor Wang, who teaches Maths, was not aware that throughout his online presentation, the microphone on his computer was switched off. This meant that (27) _______ of the students attending his online class heard what professor Wang was talking about. His university switched his classes online to help reduce the (28) _______of COVID-19. His lecture started well but then it froze. The video came back but the mic was off and professor Wang did not notice. One of his students says it might have been because the professor was using an iPad and not a computer.

      Students tried many times to contact professor Wang during the lesson but could not get through to him. They waved their arms and even tried calling him on his personal phone. A video of the moment (29) _________he understood his mic was off has gone viral on the Internet. It has received more than 653,000 views. After realizing what happened, professor Wang appeared shocked and stressed. He said he was upset at the thought of wasting two hours of his students' time. He has offered to redo the lecture at a different time so his students do not miss that class. One of his students posted on social media about the professor's (30) _______. She wrote: "I took a class under him before and he teaches well. I feel bad for him.".

A. because

B. however

C. but

D. although

Câu hỏi 1188 :

Read the following passage and mark the letter A, B, C, or D on your answer sheet to indicate the correct answer to each of the questions from 31 to 35.

FOR A QUICK PICKUP

       It used to be that people would drink coffee or tea in the morning to pick them up and get them going for the day. Then cola drinks hit the market. With lots of caffeine and sugar, these beverages soon became the pick-me-up of choice for many adults and teenagers. Now drink companies are putting out so-called “energy drinks.” These

beverages have the specific aim of giving tired consumers more energy.

       One example of a popular energy drink is Red Bull. The company that puts out this beverage has stated in interviews that Red Bull is not a thirst quencher. Nor is it meant to be a fluid replacement drink for athletes. Instead, the beverage is meant to revitalize a tired consumer’s body and mind. In order to do this, the makers of Red Bull, and other energy drinks, typically add vitamins and certain chemicals to their beverages. The added chemicals are like chemicals that the body naturally produces for energy. The vitamins, chemicals, caffeine, and sugar found in these beverages all seem like a sure bet to give a person energy.

       Health professionals are not so sure, though. For one thing, there is not enough evidence to show that all of the vitamins added to energy drinks actually raise a person’s energy level. Another problem is that there are so many things in the beverages. Nobody knows for sure how all of the ingredients in energy drinks work together.

       Dr. Brent Bauer, one of the directors at the Mayo Clinic in the US, cautions people about believing all the claims energy drinks make. He says, “It is plausible if you put these twelve things together, you will get a good result.” However, Dr. Bauer adds the mix of ingredients could also have a negative impact on the body. “We just don’t know at this

point,” he says.

What is the main idea of the reading?

A. Caffeine is bad for people to drink.

B. Red Bull is a good energy drink.

C. Energy drinks may or may not work.

D. Teenagers should not drink energy drinks.

Câu hỏi 1189 :

According to the reading, what makes it difficult for researchers to know if an energy drink gives people energy?


A. Natural chemicals in a person’s body.

B. The age of the consumer.

C. The company that makes the beverage.

D. The number of ingredients.

Câu hỏi 1190 :

The word “it” in the second paragraph refers to________.

A. one example

B. Red Bull

C. the company

D. thirst quencher

Câu hỏi 1191 :

The word “plausible” in the passage is closest in meaning to________.

A. reasonable

B. unlikely

C. typical

D. impossible

Câu hỏi 1192 :

Which of the following is NOT true according to the reading?

A. Bauer does not believe the claims of energy drink makers.

B. Colas have been on the market longer than energy drinks.

C. It is impossible to ever prove that energy drinks work.

D. The makers of Red Bull say that it can revitalize a person.

Câu hỏi 1193 :

Read the following passage and mark the letter A, B, C, or D on your answer sheet to indicate the correct answer to each of the questions from 36 to 42.

        People who daydream are often thought of in negative terms, such as being lazy or not doing what they should be doing. However, scientists who study the brain have learned many interesting things, especially from studying the brains of daydreamers. In fact, far from being a waste of time, some scientists believe that daydreaming is a healthy and useful activity for all of our brains.

        In order to study the brain, scientists use special devices that scan the brain and show pictures of which parts of the brain are active at certain times. By using this technique, they proved that when a person is daydreaming, the device will show a distinct pattern of activity in the brain called the “default” mode of thinking. In the default mode, the top or outside part of the brain is very active. Actually, several regions of the brain are interacting in this mode. Some scientists describe this mode as a time when the brain focuses on itself rather than focusing on the environment around the person. Typically, this occurs when a person is doing simple, tedious work or performing routine actions that don’t need much attention, like walking to school or cooking simple foods. People tend to daydream during such activities.

        The importance of daydreaming lies in developing both creative and social skills. When the mind is not engaged in dealing with one’s immediate situation or problem, then it is free to wonder about things. A time of wandering allows the mind to create things. New inventions may be imagined, or possible solutions to a problem can be planned. For example, solutions for problems in relationships with other people may come to mind. In fact, most daydreams involve situations with others. Perhaps these are daydreams based on memories of the past, or daydreams of what might be in the future. In either case, daydreams help us develop the appropriate skills we can use in real interactions with others.

         As neurologist Dr. Marcus Raichle of Washington University explains: “When you don’t use a muscle, that muscle really isn’t doing much. But when your brain is supposedly doing nothing and daydreaming, it’s really doing a tremendous amount.” During the so-called “resting state” the brain isn’t resting at all!

What does the passage mainly discuss?

A. Can daydreams reduce the hours we sleep?

B. How is daydreaming different from sleeping?

C. Who daydreams more, men or women?

D. Why is daydreaming good for people?

Câu hỏi 1194 :

The word “they” in the second paragraph refers to________.

A. parts

B. pictures

C. devices

D. scientists

Câu hỏi 1195 :

Which of the following is NOT involved in daydreaming?


A. One’s immediate environment.

B. Several regions of the brain.

C. The “default” mode of thinking.

D. The outside part of the brain.

Câu hỏi 1196 :

What does the brain focus on in the “default” mode?

A. A person’s immediate situation.

B. Appropriate relationships.

C. Itself.

D. Routine actions.

Câu hỏi 1197 :

The word “wonder” in the passage is closest in meaning to________.

A. consume

B. question

C. drag

D. celebrate

Câu hỏi 1198 :

According to the reading, which of the following would NOT be a result of letting the mind wonder about things?


A. A job that needs close attention.

B. A new invention.

C. A solution to a problem.

D. A way to act in a relationship.

Câu hỏi 1199 :

What does Raichle mean by the quote used in the reading?

A. The brain develops one’s muscles.

B. The brain and our muscles both need rest.

C. The brain is a lot like a muscle.

D. The brain is different than a muscle.

Câu hỏi 1203 :

Mark the letter A, B, C, or D on your answer sheet to indicate the sentence that is closest in meaning to each of the following questions.
Paul had his last drink 10 years ago.

A. Paul has never had his last drink for 10 years.

B. Paul has never had his last drink before.

C. Paul hasn’t drunk since 10 years.

D. It’s 10 year since Paul last had his drink.

Câu hỏi 1204 :

“Don't sit in front of the computer for too long.”, our teacher told us.


A. Our teacher wanted us to sit in front of the computer for too long.

B. Our teacher refused us to sit in front of the computer for too long.

C. Our teacher warned us not to sit in front of the computer for too long.

D. Our teacher asked us to sit in front of the computer for too long.

Câu hỏi 1205 :

In the end, it wasn’t necessary to buy as much bread as I did.

A. In the end, I needn’t buy as much bread as I did.

B. In the end, I needn’t have bought as much bread as I did.

C. In the end, I mustn’t have bought as much bread as I did.

D. In the end, I mustn’t buy as much bread as I did.

Câu hỏi 1206 :

Mark the letter A, B, C, or D on your answer sheet to indicate the sentence that best combines each pair of sentences in the following questions.

She had an argument with her father last night. She now realizes her father was right.

A. Without her argument with her father last night, she wouldn’t realize her father was right.

B. She wishes she hadn’t had an argument with her father last night.

C. If only she realized her father was right now.

D. If she didn’t have an argument with her father last night, she wouldn’t realize her father was right.

Câu hỏi 1207 :

Jimmy took up rock climbing. He overcame his fear of heights only then.

A. Hardly had Jimmy took up rock climbing when he overcame his fear of heights.

B. But for taking up rock climbing, Jimmy would have overcome his fear of heights.

C. Despite his fear of heights, Jimmy took up rock climbing.

D. Only when Jimmy took up rock climbing did he overcome his fear of heights.

Câu hỏi 1213 :

We will send you a brochure______________.

A. until new stock will arrive

B. by the time new stock arrives

C. when new stock arrived

D. until new stock arrives

Câu hỏi 1215 :

One day, hopefully in the not so distant future, all housework __________ by robots.

A. will be done

B. is being done

C. will have done

D. will do

Câu hỏi 1216 :

What _____the two women ____about when we started to quarrel ?

A. did/ talk

B. are/ talking

C. were/ talking

D. had/ talked

Câu hỏi 1217 :

We are going on a _______ boat for our anniversary next Sunday.

A. red sailing beautiful

B. beautiful red sailing

C. sailing red beautiful

D. red beautiful sailing

Câu hỏi 1218 :

The older dogs get, ______________.

A. the aggressive they get more

B. the less aggressive they get

C. they get more and more aggressive

D. they get less aggressive

Câu hỏi 1220 :

I was afraid the ________ was going to turn into a fight.

A. argue

B. argument

C. arguable

D. arguably

Câu hỏi 1221 :

David was determined to see the performance through after _____ his admission fee.

A. having been paid

B. paying

C. having paid

D. being paid

Câu hỏi 1222 :

Attendance at that new play is so low that the producers are trying to __________ by advertising heavily.

A. build it up

B. stick it out

C. fade it away

D. come it about

Câu hỏi 1225 :

I feel as fresh as a(n) __________ after having had a good workout and a shower.

A. cave

B. daisy

C. fish

D. orange

Câu hỏi 1226 :

Jack is a great manager. However, he is very hot-tempered. That’s his only __________.

A. act of god

B. acid test

C. Achilles’ heel

D. Dutch treat

Câu hỏi 1228 :

The Rev. Dr. Martin Luther King fought to put an end to racial segregation in the United States.

A. integration

B. separation

C. education

D. torture

Câu hỏi 1230 :

I think having a beer during a meeting with your boss is clearly overstepping the mark.

A. doing a completely acceptable thing.

B. making your boss satisfied.

C. do something over.

D. behaving in an improper way.

Câu hỏi 1232 :

Meghan and Kate are discussing the cancer.

- Meghan: “I believe that worrying about cancer can give you cancer.

- Kate: “______________. Worrying makes things worsen.”

A. No, I'm not so sure about that.

B. No way.

C. I couldn't agree with you more.

D. Really? Are you sure?

Câu hỏi 1238 :

Read the following passage and mark the letter A, B, C, or D on your answer sheet to indicate the correct answer to each of the questions from 31 to 35.

    We can find bees in almost every part of the world except the northernmost and southernmost regions. They are insects and classified into over 10,000 species. One commonly known species is the honeybee, the only bee that produces honey and wax. Humans use the wax in making candles, lipsticks, and other products, and they use the honey as a food. While gathering the nectar and pollen with which they make honey, bees are simultaneously helping to fertilize the flowers on which they land. Many fruits and vegetables would not survive if bees did not carry the pollen from blossom to blossom.

      Bees live in a structured environment and social structure within a hive, which is a nest with storage space for the honey. The different types of bees each perform a unique function. The worker bee carries nectar to the hive in a special stomach called a honey stomach. Other workers make beeswax and shape it into a honeycomb, which is a waterproof mass of six-sided compartments, or cells. The queen lays eggs in completed cells. As the workers build more cells, the queen lays more eggs.

      All workers, like the queen, are female, but the workers are smaller than the queen. The male honeybees are called drones; they do no work and cannot sting. They are developed from unfertilized eggs, and their only job is to impregnate a queen. The queen must be fertilized in order to lay worker eggs. During the season when less honey is available and the drone is of no further use, the workers block the drones from eating the honey so that they will starve to death.

Which of the following is the best title for this reading?

A. Making Honey.

B. The Honey Bee - Its Characteristics and Usefulness.

C. The Useless Drone.

D. The Many Species of Bees.

Câu hỏi 1239 :

The word “simultaneously” in the fifth sentence is closest in meaning to __________.

A. stubbornly

B. concurrently

C. skillfully

D. diligently

Câu hỏi 1240 :

The word “They” in the last paragraph refers to_________.

A. workers

B. queens

C. drones

D. honeybees

Câu hỏi 1241 :

According to the passage, the drone_________.


A. mates with the queen and has no other purpose.

B. comes from eggs fertilized by other drones.

C. can be male or female.

D. collects less honey than workers.

Câu hỏi 1242 :

The author implies that _______________________.

A. bees are unnecessary in the food chain.

B. drones are completely dispensable.

C. the queen can be a worker.

D. drones are never females.

Câu hỏi 1243 :

Read the following passage and mark the letter A, B, C, or D on your answer sheet to indicate the correct answer to each of the questions from 36 to 42.

        There are two basic types of glaciers, those that flow outward in all directions with little regard for any underlying terrain and those that are confined by terrain to a particular path.

        The first category of glaciers includes those massive blankets that cover whole continents, appropriately called ice sheets. There must be over 50,000 square kilometers of land covered with ice for the glacier to qualify as an ice sheet, when portions of an ice sheet spread out over the ocean, they form ice shelves.

        About 20,000 years ago the Cordilleran Ice sheet covered nearly all the mountains in southern Alaska, western Canada, and the western United States. It was about 3 kilometers deep at its thickest point in northern Alberta. Now there are only two sheets left on Earth, those covering Greenland and Antarctica.

        Any domelike body of ice that also flows out in all directions but covers less than 50,000 square kilometers is called an ice cap. Although ice caps are rare nowadays, there are a number in northeastern Canada, on Baffin Island, and on the Queen Elizabeth Islands.

        The second category of glaciers includes those of a variety of shapes and sizes generally called mountain or alpine glaciers. Mountain glaciers are typically identified by the landform that controls their flow. One form of mountain glacier that resembles an ice cap in that it flows outward in several directions is called an ice field. The difference between an ice field and an ice cap is subtle. Essentially, the flow of an ice field is somewhat controlled by surrounding terrain and thus does not have the domelike shape of a cap. There are several ice fields in the Wrangell. St. Elias, and Chugach mountains of Alaska and northern British Columbia.

        Less spectacular than large ice fields are the most common types of mountain glaciers: the cirque and valley glaciers. Cirque glaciers are found in depressions in the surface of the land and have a characteristic circular shape. The ice of valley glaciers, bound by terrain, flows down valleys, curves around their corners, and falls over cliffs.

Which of the following could best serve as the topic of the passage?

A. Where major glaciers are located

B. How glaciers shape the land

C. The different kinds of glaciers

D. How glaciers are formed

Câu hỏi 1244 :

The word “terrain” in paragraph 1 could best be replaced by____________.

A. the seabed

B. area of land

C. countryside

D. prairie

Câu hỏi 1245 :

It can be inferred from paragraph 2 that ice sheets are so named because____________

A. they are thicker in some areas than the others

B. they are identified by the landform that controls their flow

C. they cover large areas of land

D. they are confined to cirque glaciers

Câu hỏi 1246 :

According to the passage, where was the Cordilleran Ice Sheet thickest?

A. Alaska

B. Antarctica

C. Greenland

D. Alberta

Câu hỏi 1247 :

The word “subtle” in paragraph 5 could best be replaced by____________.

A. slight

B. substantial

C. regional

D. obvious

Câu hỏi 1248 :

The word “their” in last paragraph refers to__________.

A. ice fields

B. cirque glaciers

C. valley glaciers

D. valleys

Câu hỏi 1249 :

According to the passage, ice shelves can be found_________.

A. covering an entire continent

B. buried within the mountains

C. spreading into the ocean

D. filling deep valleys

Câu hỏi 1253 :

Mark the letter A, B, C, or D on your answer sheet to indicate the sentence that is closest in meaning to each of the following questions.
It’s a long time since men last went to the Moon.

A. The last time men went to the Moon was since a long time.


B. Men haven’t gone to the Moon for a long time

C. Men went to the Moon a long time ago.

D. Men didn’t go to the Moon since a long time.

Câu hỏi 1254 :

‘If you’re good, I’ll buy you an ice cream later,’ said the boy’s mother


A. His mother told him to buy her an ice cream later if he was good.

B. His mother promised to buy him an ice cream later if he was good.

C. His mother asked him whether he liked an ice cream or not.

D. His mother threatened to buy him an ice cream later if he was good.

Câu hỏi 1255 :

There is no obligation to get an international driving licence for this country.

A. You must get an international driving licence for this country.

B. You shouldn’t get an international driving licence for this country.

C. You don’t have to get an international driving licence for this country.

D. You may not get an international driving licence for this country.

Câu hỏi 1256 :

Mark the letter A, B, C, or D on your answer sheet to indicate the sentence that best combines each pair of sentences in the following questions.

Flora couldn’t get a scholarship. She didn’t get a chance to study abroad.

A. If only Flora had got a scholarship and got a chance to study abroad.

B. Without a scholarship, Flora could have got a chance to study abroad.

C. Flora wishes she could get a chance to study abroad without a scholarship.

D. Supposed Flora got a scholarship, she could get a chance to study abroad.

Câu hỏi 1257 :

The researchers developed the first flu vaccination recently. There was a reliable protection against flu in this country.

A. Not until the researchers developed the first flu vaccination recently was there a reliable protection against flu in this country.

B. Scarcely the researchers developed the first flu vaccination recently when there was a reliable protection against flu in this country.

C. But for the first flu vaccination, there wouldn’t be a reliable protection against flu in this country.

D. So reliable was the protection against flu in this country that the researchers developed the first flu vaccination recently.

Câu hỏi 1265 :

__________ are becoming fed up with the corruption in the government.

A. More and more

B. More people and more people

C. More and less people

D. More and more people

Câu hỏi 1266 :

She has just bought a __________ watch.

A. Swiss expensive brand-new

B. Swiss brand-new expensive

C. brand-new expensive  Swiss

D. expensive brand-new Swiss

Câu hỏi 1267 :

When I last came into him, he________in London.

A. is living

B. has been living

C. was living

D. lived

Câu hỏi 1268 :

Spider monkeys are the best climbers in the jungle, ________  not having thumbs.

A. because of

B. because

C. despite

D. although

Câu hỏi 1269 :

Mrs Nam will have lived here for 30 years ________.

A. by the time she retired

B. when she retired

C. by the time she retires

D. after she had retired

Câu hỏi 1270 :

________, he began to make friends more easily.

A. Having entered  the new school

B. After entered the new school

C. On having been entered the new school

D. Upon entering the new school

Câu hỏi 1271 :

We thanked the hosts for their ___________ hospitality and got under way.

A. generous

B. generosity

C. generously

D. ungenerosity

Câu hỏi 1272 :

He has eaten so much sweet food , which __________ his obesity.

A. taken on

B. brought about

C. caught out

D. kept in

Câu hỏi 1273 :

I took this new job because I felt that the ___________ prospects were much better.

A. work

B. career

C. job

D. profession

Câu hỏi 1274 :

The president ________ tribute to all the people who had supported him.

A. made

B. gave

C. paid

D. returned

Câu hỏi 1278 :

Students are required to abide by the rules of the school and mind their manners.

A. obey

B. memorize

C. review

D. compose

Câu hỏi 1280 :

Because of rain and greasy surfaces, driving conditions became treacherous.

A. dangerous

B. serious

C. poor

D. favorable

Câu hỏi 1282 :

Paul and Daisy are discussing about life in the future.

Paul: “I believe space travel will become more affordable for many people in the future.”

Daisy:“_________________”


A. It doesn’t matter at all.


B. I am sorry to hear that.

C. It is very kind of you to say so.

D. There’s no doubt about that.

Câu hỏi 1283 :

Read the following passage and mark the letter A, B, C, or D on your answer sheet to indicate the correct word or phrase that best fits each of the numbered blanks.

Artificial Intelligence (AI) in Viet Nam is developing rapidly and gradually affirming that it is a pillar and (26) ________ technology in the fourth industrial revolution. Technological powers have long built their own AI development strategies, taking this technology as the core for (27) ________ economic development.” OpenGov reports. 

      Over the last few years, more organizations have started developing and applying AI in various fields including education, telecommunications, retail, healthcare and (28) ________ which have not only gradually dominated the market but also earned huge profits. 

      Currently, AI is the focus of the global technology circle and governments around the world. Many countries have spent billions of US dollars on AI development strategies with the ambition to be the frontrunners of AI. 

      Viet Nam is not outside this development trend either. In recent years, Viet Nam's AI industry has made visible strides with increasing AI content in various products, the release noted. Large domestic technology corporations (29) ________ are interested in investing in and building AI sources implement AI projects.  Many domestic companies as well as innovative start-ups, also tend to invest in AI and carry out many new applications in new business models. (30) ________, like many countries that prioritize AI investment ,  Viet Nam lacks large databases, infrastructure, resources, and a solid foundation from AI businesses. 


A. break-in


B. outbreak

C. breakdown

D. breakthrough

Câu hỏi 1288 :

Read the following passage and mark the letter A, B, C, or D on your answer sheet to indicate the correct answer to each of the questions

      Many of the major supermarket chains have come under fire with accusations of various unethical acts over the past decade. They've wasted tons of food, they've underpaid their suppliers and they've contributed to excessive plastic waste in their packaging, which has had its impact on our environment. 

      But supermarkets and grocers are starting to sit up and take notice. In response to growing consumer backlash against the huge amounts of plastic waste generated by plastic packaging, some of the largest UK supermarkets have signed up to a pact promising to transform packaging and cut plastic wastage. In a pledge to reuse, recycle or compost all plastic wastage by 2025, supermarkets are now beginning to take some responsibility for the part they play in contributing to the damage to our environment with one major supermarket announcing their plan to eliminate all plastic packaging in their own-brand products by 2023. 

      In response to criticisms over food waste, some supermarkets are donating some of their food surplus. However, charities estimate that they are only accessing two per cent of supermarkets' total food surplus, so this hardly seems to be solving the problem. Some say that supermarkets are simply not doing enough. Most supermarkets operate under a veil of secrecy when asked for exact figures of food wastage, and without more transparency it is hard to come up with a systematic approach to avoiding waste and to redistributing surplus food. 

      Some smaller companies are now taking matters into their own hands and offering consumers a greener, more environmentally friendly option. Shops like Berlin's Original Unverpakt and London's Bulk Market are plastic- tree shops that have opened in recent years, encouraging customers to use their own containers or compostable bags. Online grocer Farmdrop eliminates the need for large warehouses and the risk of huge food surplus by delivering fresh produce from local farmers to its customers on a daily basis via electric cars, offering farmers the lion's share of the retail price. 

      There is no doubt that we still have a long way to go in reducing food waste and plastic waste. But perhaps the major supermarkets might take inspiration from these smaller grocers and gradually move towards a more sustainable future for us all. 

(Adapted from https://learnenglish.britishcouncil.org) 

Which is the most suitable title for the passage?


A. Major Supermarket Chains


B. Grocers vs. Supermarkets

C. Sustainable Supermarkets

D. Friendly Supermarkets

Câu hỏi 1289 :

According to paragraph 2, more and more people want supermarkets to _______.

A. compost all plastic wastage

B. donate some of their food surplus

C. reduce their plastic waste

D. lower their prices for local farmers

Câu hỏi 1290 :

The word "backlash” in paragraph 2 is closest in meaning to _______.

A. agreement

B. request

C. reaction

D. benefit

Câu hỏi 1291 :

The phrase "the lion's share" in paragraph 4 is closest in meaning to _______.

A. the largest part

B. the best choice

C. the animal's food

D. the royal dish

Câu hỏi 1292 :

Which statement is TRUE, according to the passage?

A. Supermarkets are not telling people how much food they are actually wasting.

B. There is a grocer in Berlin that doesn't allow customers to use their own containers.

C. Supermarkets are still denying that plastic packaging can cause damage to our environment.

D. Farmdrop stores large amounts of food and produces unnecessary waste.

Câu hỏi 1293 :

Read the following passage and mark the letter A, B, C, or D to indicate the correct answer to each of the questions.

         The emergence of modern humans are explained on the foundation of  two main hypotheses. The "Out of Africa" theory holds that homo sapiens burst onto the scene as a new species around 150,000 to 200,000 years ago in Africa and subsequently replaced archaic humans such as the Neanderthals. The other model, known as multi-regional evolution or regional continuity, posits far more ancient and diverse roots for our kind. Proponents of this view believe that homo sapiens arose in Africa some 2 million years ago and evolved as a single species spreading across the Old World, with populations in different regions linked through genetic and cultural exchange. 

      Of these two models, Out of Africa, which was originally developed based on fossil evidence, and supported by much genetic research, has been favored by the majority of evolution scholars. The vast majority of these genetic studies have focused on DNA from living populations, and although some small progress has been made in recovering DNA from Neanderthal that appears to support multi-regionalism, the chance of recovering nuclear DNA from early human fossils is quite slim at present. Fossils thus remain very much a part of the human origins debate. 

      Another means of gathering theoretical evidence is through bones. Examinations of early modern human skulls from Central Europe and Australia dated to between 20,000 and 30,000 years old have suggested that both groups apparently exhibit traits seen in their Middle Eastern and African predecessors. But the early modern specimens from Central Europe also display Neanderthal traits, and the early modern Australians showed affinities to archaic Homo from Indonesia. Meanwhile, the debate among paleoanthropologists continues, as supporters of the two hypotheses challenge the evidence and conclusions of each other.

All of the following statements are true EXCEPT _________


A. DNA studies offer one of the best ways in future to provide clear evidence.


B. three methods of gathering evidence are mentioned in the passage.

C. the multi-regional model goes back further in history.

D. the Out of Africa model has had more support from scholars.

Câu hỏi 1294 :

It can be inferred from the passage that ___________

A. the debate will interest historians to take part in.

B. there is likely to be an end to the debate in the near future.

C. there is little likelihood that the debate will die down.

D. the debate is likely to be less important in future.

Câu hỏi 1295 :

According to the passage, which of the following information is NOT true?

A. The vast majority of genetic studies have focused on living populations.

B. Examinations of early modem human skulls all support the same conclusions.

C. Both hypotheses focus on Africa as a location for the new species.

D. Early modem Australian skulls have similarities to those from Indonesia.

Câu hỏi 1296 :

Which of the following statements is NOT true about the two hypotheses?

A. Both hypotheses cite Africa as an originating location.

B. Genetic studies have supported both hypotheses.

C. One hypothesis dates the emergence of homo sapiens much earlier than the other.

D. Both hypotheses regard Neanderthals to be the predecessors of modern humans.

Câu hỏi 1297 :

The passage primarily discusses which of the following ________

A. two hypotheses and some evidence on the human origins debate.

B. evidence that supports the "Out of Africa" theory.

C. that fossils remain very much a part of the human origins debate.

D. the difficulties in obtaining agreement among theorists on the human origins debate.

Câu hỏi 1298 :

In paragraph 3, the word “their” refers to __________


A. Central Europeans and Australians


B. traits

C. skulls

D. Middle Easterners and Africans

Câu hỏi 1299 :

The word "predecessors" in paragraph 3 is closest in meaning to __________

A. off-spring

B. ancestors

C. pioneers

D. juniors

Câu hỏi 1301 :

I bought an interesting book last week, but I cannot find them now.

A. bought

B. interesting

C. cannot

D. them

Câu hỏi 1302 :

Doctors fear possible epidemics of cholera, malaria, and other deadthy diseases.

A. fear

B. epidemic

C. malaria

D. deadthy

Câu hỏi 1303 :

Mark the letter A, B, C, or D on your answer sheet to indicate the sentence that is closest in meaning to each of the following questions.
She began to play the piano three years ago.

A. She has played the piano since three years.

B. She has played the piano for three years.

C. She doesn’t play the piano now.

D. She stops playing the piano now.

Câu hỏi 1304 :

“I’ll help you repair this washing machine, Jenny”, he said.

A. He admitted helping Jenny repair that washing machine.

B. He denied helping Jenny repair that washing machine.

C. He promised to help Jenny repair that washing machine.

D. He refused to help Jenny repair that washing machine.

Câu hỏi 1305 :

You are supposed to be here at 9 a.m, but it is 11 a.m now.

A. You needn’t have been here two hours ago.

B. You must have been here two hours ago.

C. You should have been here two hours ago.

D. You oughtn’t to have been here two hours ago.

Câu hỏi 1306 :

Mark the letter A, B, C, or D on your answer sheet to indicate the sentence that best combines each pair of sentences in the following questions.

A. As long as you helped me, I could overcome the problem.

B. If you didn’t help me, I couldn’t overcome the problem.

C. If only you helped me to overcome the problem.

D. I wish you had helped me to overcome the problem.

Câu hỏi 1307 :

I posted the letter. I realized that I had forgotten to put on a stamp.

A. Only after posting the letter did I realize that I had forgotten to put on a stamp.

B. Hardly had I realized that I had forgotten to put on a stamp when I posted the letter.

C.Not only did I realize that I had forgotten to put a stamp on the letter but I posted it as  well.

D. No sooner had I posted the letter than I forgot to put on a stamp.

Câu hỏi 1313 :

Only one of the workers_______ to do this difficult task.


A. has been chosen


B. chosen

C. have chosen

D. has chosen

Câu hỏi 1314 :

We must avoid  war _______ any cost

A. In

B. on

C. at

D. for

Câu hỏi 1315 :

My neighbor is driving me mad! It seems that _______it is at night,_______he plays his music!

A. the more late / the more loudlier

B. the less / the more loud

C. the latter / the louder

D. the later / the louder

Câu hỏi 1316 :

Barry pulled out the sword, which was made of a __________ material.

A. strange green metallic

B. green strange metallic

C. strange metallic green

D. metallic strange green

Câu hỏi 1317 :

I was playing sports when my mom ______ home.

A. came

B. was coming

C. comes

D. has come

Câu hỏi 1318 :

They have tried their best ....................... they can’t reach their goal.

A. though

B. but

C. because

D. in spite of

Câu hỏi 1319 :

You will receive an automatic email notification ________

A. after your profile will be available online.

B. when you finished your online profile.

C. as soon as you completed our online profile.

D. once your profile is available online.

Câu hỏi 1320 :

_______contaminated water from a nearby river, lots of people in this village suffer from cancers.

A. Drinking

B. Having drunk

C. Having been drunk

D. Drunk

Câu hỏi 1321 :

A. components

B. materials

C. ingredients

D. supplements

Câu hỏi 1322 :

After the economic crisis, some industry analysts are convinced that things are _____ for the business.

A. looking up

B. going on

C. taking up

D. turning on

Câu hỏi 1323 :

In the past, the ____ and engagement ceremonies took place one or two years before the wedding.

A. propose

B. proposing

C. proposal

D. proposed

Câu hỏi 1325 :

We realized we weren't going to get the job finished, so we decided to ______.

A. pull my leg

B. call it a day

C. put your back up

D. push your luck

Câu hỏi 1326 :

The campaign is trying to stop the _____ use of chemicals in farming.


A. redundant


B. intensive

C. marginal

D. excessive

Câu hỏi 1329 :

To receive truthful answers from respondents is an expectation upon which all interviews are based.

A. exactly as things really happen

B. with a negative attitude

C. in a harmful way

D. as trustingly as you can

Câu hỏi 1330 :

The Internet provides an infinite pool of buyers from around the world with whom freelancers can contract their services.

A. quite a few

B. a limited number of

C. a wide variety of

D. many a

Câu hỏi 1331 :

Mark the letter A, B, C, or D on your answer sheet to indicate the option that best completes each of the fol­lowing exchanges.

A. Don’t mention it

B. That’s nice of you to say so.

C. I’d love it.

D. I couldn’t agree more.

Câu hỏi 1332 :

Hanh and David have just had a delicious meal in a restaurant.

Hanh: “It's very generous of you to offer to pay!”

David: “_________________.”


A. I'm glad you like it.


B. Thanks a million.

C. That was the least I could do.

D. You can say that again.

Câu hỏi 1337 :

Not (30)_____ skin markings are permanent, though.


A. all


B. any

C. other

D. another

Câu hỏi 1338 :

The word confiscated in paragraph 2 is closest in meaning to ___________.

A. robbed

B. detained

C. arrested

D. seized

Câu hỏi 1339 :

Read the following passage and mark the letter A, B, C, or D on your answer sheet to indicate the correct answer to each of the questions.

            The Chinese government has removed pangolin scales from its 2020 list of approved ingredients used in traditional Chinese medicine, a move campaigners describe as a "critical step" towards saving the world's most trafficked mammal.

Pangolins are highly valued in Asia for scales, which many people believe can improve blood circulation and reduce inflammation. Last year alone, authorities confiscated more than 130 tons of illegal pangolin related products. There are eight species of pangolin found in Asia and Africa. To date, three species are listed as critically endangered by the International Union for Conservation of Nature, while the remaining five are listed as either vulnerable or endangered.

According to Chinese media, the latest edition of Chinese Pharmacopoeia - an official government list of drugs covering approved traditional Chinese and Western medicines - no longer includes pangolin scales. The decision to remove the pangolin from the official list comes just days after China's State Forestry and Grassland Bureau announced that the Chinese pangolin would be upgraded to a "first-level protected wild animal," the highest possible protection status alongside pandas and tigers.

While conservationists from China and other countries have praised the actions of Chinese authorities, some still felt that they came a bit too late. "Many years have passed. How many pangolins have already been hunted and killed?" said Sophia Zhang, director of the Pangolin Working Group at the China Biodiversity Conservation and Green Development Foundation.

                                      (Adapted from edition.cnn.com)

What is the best title for this passage?


A. What We Can All Do To Protect Pangolins


B. Why Are Chinese Pangolins Under Threat?

C. Pangolin Scales: How Valuable Are They?

D. A Big Step In The Protection Of Pangolins

Câu hỏi 1340 :

How have conservationists in China and in other countries react to the Chinese government?

A. They are pleased but some felt that actions should have been taken sooner

B. They believe these actions are useless, given the number of pangolins killed

C. They doubt whether these actions will prevent people from hunting pangolins.

D. They are all glad that Chinese authorities have taken measures at the right time.

Câu hỏi 1341 :

Which of the following is NOT TRUE, according to the passage?

A. It is widely believed that pangolin scales have medicinal powers.

B. The Chinese government has a list of approved medicines.

C. There are five species of pangolins that are not endangered.

D. Pandas and tigers are first-level protected wild animals in China.

Câu hỏi 1342 :

The word they in paragraph 4 refers to __________.

A. conservationists

B. countries

C. actions

D. authorities

Câu hỏi 1343 :

Read the following passage and mark the letter A, B, C, or D on your answer sheet to indicate the correct answer to each of the questions from 36 to 42.

      Throughout history, various people have demonstrated a high degree of confidence in the ability of certain animals to predict the weather. It may seem surprising today in view of the complex equipment now involved in weather forecasting to understand that in certain cases, the behavior of animals does indeed provide an indication of inclement weather. Sensitivity of certain animals to falling air pressure or to low-frequency sound waves that humans cannot hear, which are indicators of approaching storms, causes behaviors in animals that certain societies have come to recognize as predictors of storms.

A number of animals are remarkably sensitive to variations in air pressure, and some of these animals show consistent, noticeable, and predictable behaviors as air pressure drops before a storm hits. Where the air pressure drops before a storm, some animals move closer to the ground to equalize the pressure in their ears: some birds such as swallows tend to stay on the ground or roost in trees instead of soaring in the skies when a storm is imminent because of the decreasing air pressure. Other animals make more noise than usual as air pressure drops: an unusual amount of quacking by ducks and a high volume of croaking by frogs are both indicators that are believed to occur because of the high degree of sensitivity of ducks and frogs to the change in pressure. Finally, still other animals become more active before storms as a reaction to the falling air pressure: dolphins and porpoises seem to be taking part in a frenzied sort of play, and bees and ants become more active prior to storms, most likely because of their sensitivity to lower pressure.

There is good reason to believe that the fact that these animal behaviors seem to occur regularly prior to storms may have a scientific basis and that the animals demonstrating these behaviors may actually be good short-range weather forecasters. However, their ability to predict long-range weather patterns is rather suspect. Certain proverbs, for example, are based on what is most likely the idea that squirrels are good indicators of long-range weather patterns. One proverb indicates that, if a squirrel seems busier than usual in gathering nuts, then a long and cold winter is on its way; however, this behavior in squirrels is more likely due to a large supply of nuts available for gathering, which occurs because of earlier good weather, and is not an indicator of cold weather to come. Another proverb about squirrels indicates that if a squirrel grows a long and bushy tail in the fall, then a particularly harsh winter is on its way; in this case too, the squirrel develops a long and bushy tail because of earlier good weather and not as a warning of bad weather to come.

(Adapted from TOEFL Reading Practice by Deborah Philips)

What is the main idea of the passage?


A. Animals as weather forecasters.


B. Signs of a weather condition.

C. Proverbs of weather.

D. Study of animal behaviors.

Câu hỏi 1344 :

Which of the following happens when air pressure drops before a storm?

A. Swallows soar in the sky.

B. Ducks suddenly fall silent.

C. Frogs gather together in large groups.

D. Ants move around more frantically than usual.

Câu hỏi 1345 :

The word inclement in paragraph 1 is closest in meaning to _____

A. local

B. stormy

C. favored

D. stable

Câu hỏi 1346 :

The word that in paragraph 2 refers to ____

A. a high volume

B. croaking frogs

C. indicators

D. other animals

Câu hỏi 1347 :

The word roost in paragraph 2 is closest in meaning to _____

A. feed

B. communicate

C. grow

D. settle

Câu hỏi 1348 :

Which of the following statements is NOT true?

A. Some animals can predict weather for a season.

B. Proverbs relating to squirrels are not good weather predictors.

C. Many animals change their behaviors shortly before a storm.

D. Low-frequency sounds are inaudible to the human ear.

Câu hỏi 1350 :

Which of the following can be inferred from paragraph 3?

A. There is a scientific basis for the belief that certain behaviors in animals influence the paths taken by thunderstorms.

B. Short-range weather forecasts have become increasingly more successful in predicting thunderstorms.

C. Scientists are studying the possibility that certain animal behaviors may occur during storms.

D. Quite possibly there are scientific reasons why animals act in certain ways before storms.

Câu hỏi 1351 :

Every country has their own traditions, some of which have existed for centuries.

A. country

B. Their own

C. Some of which

D. existed

Câu hỏi 1353 :

Mark the letter A, B, C, or D on your answer sheet to indicate the sentence that is closest in meaning to each of the following question
last saw him when I was a student.

A. I haven’t seen him since I was a student.

B.  I haven’t seen him when I was a student.

C. I last saw him since I was a student.

D. I have seen him since I was a student.

Câu hỏi 1354 :

"I will help you with your CV, Mary", Peter said.

A. Peter advised Mary to write the CV

B. Peter promised to help Mary with her CV.

C. Peter advised Mary not to write her CV

D. Peter wanted Mary to help with the CV.

Câu hỏi 1355 :

It isn't necessary for you to go to the meeting today.

A. You won’t go to the meeting today.

B. You should go the meeting today

C. You don’t have to go to the meeting today.

D. You must go to the meeting today.

Câu hỏi 1356 :

Mark the letter A, B, C, or D on your answer sheet to indicate the sentence that best combines each pair of sentences in the following questions

Marie didn’t turn up at John’s birthday party. I feel so sorry for that.

A. If only Marie turn up at John’s birthday party.

B. I wish Marie had turned up at Johns birthday party.

C. I wished Marie wouldn’t turn up at John’s birthday party.

D. It’s a shame Marie had turned up at John’s birthday party.

Câu hỏi 1357 :

The office phoned me. I found out about the meeting

A. Not until did the office phone me did I find out about the meeting

B. Not until the office phoned me did I find out about the meeting

C. Only the office phoned me did I find out about the meeting

D. Only after did the office phone me that  I found out about the meeting

Câu hỏi 1363 :

___________________, we will have moved in a new house.

A. When you came next time

B. After you come next time

C. After you had come next time

D. When you come next time

Câu hỏi 1365 :

It’s true that happiness ________ with money, but some money makes life easier.

A. can buy

B. are bought

C. bought

D. can be bought

Câu hỏi 1366 :

Last night, I __________ in bed when I suddenly heard a scream.

A. were watching

B. read

C. reads

D. was reading

Câu hỏi 1367 :

We bought a(n) _________ rug on our holiday to Iran last year.

A. Persian beautiful old

B. beautiful Persian old

C. old beautiful Persian

D. beautiful old Persian

Câu hỏi 1368 :

_________________________, the more likely he or she is to die young.

A. The more dangerous a person lives

B. More and more dangerous a person lives

C. More and more dangerously a person lives

D. The more dangerously a person lives

Câu hỏi 1370 :

Customer reviews indicate that many modern mobile devices are often unnecessarily______.

A. complication

B. complicates

C. complicate

D. complicated

Câu hỏi 1371 :

__________ his work successfully, he went to collect his payment.

A. Finishing

B. Having finished

C. Being finished


D. Having been finished

Câu hỏi 1372 :

The chairman ______ a very controversial idea which had little chance of being accepted.

A. got on

B. came up

C. made for

D. put forward

Câu hỏi 1373 :

The proposal to save the environment by collecting waste was _______ by the majority of the public.

A. reduced

B. advocated

C. required

D. predicted

Câu hỏi 1374 :

Maxwell was in _________ heaven when he learned that he had been promoted.

A. ninth

B. seventh

C. fifth

D. first

Câu hỏi 1375 :

Mr. Doan Ngoc Hai is held in high __________ for his dedication to the society.

A. respect

B. esteem

C. homage

D. honor

Câu hỏi 1378 :

You should pat yourself on the back for having achieved such a high score in the graduation exam.

A. praise yourself

B. criticize yourself

C. check up your back

D. wear a backpack

Câu hỏi 1380 :

He bent over backwards to please his new girlfriend but she never seemed satisfied.

A. gave someone a nice surprise

B. tried to do something which is easy

C. did some physical exercise

D. tried to do something impossible

Câu hỏi 1388 :

Read the following passage and mark the letter A, B, C, or D on your answer sheet to indicate the correct answer to each of the questions from 31 to 35.

    Humans generally spent more time working than do other creatures, but there is greater variability in industriousness from one human culture to the next than is seen in subgroups of any other species. For instance, the average French worker toils for 1,646 hours a year; the average American for 1,957 hours; and the average Japanese for 2,088.

    One reason for human diligence is that people, unlike animals, can often override the impulses they may feel to slow down. They can drink coffee when they might prefer a nap or flick on the air-conditioning when the heat might otherwise demand torpor. Many humans are driven to work hard by a singular desire to gather resources far beyond what is required for survival. Squirrels may collect what they need to make it through one winter but only humans worry about collect bills, retirement, or replacing their old record albums with compact discs.

    Among other primates, if you don't need to travel around to get food for that day, you sit down and relax, said Dr.Frans de Waal of Emory University in Atlarita. "it's typically human to try to accumulate wealth and get more and more.”

     Much of the acquisitiveness is likely to be the result of cultural training. Anthropologists have found that most hunter-gatherer groups, who live day to day on the resources they can kill or forage and who stash very little away for the future generally work only three to five hours daily.

     Indeed, an inborn temptation to reduce may lurk beneath even the most work-obsessed people, which could explain why sloth ranks with lust and gluttony as one of the seven deadly sins.

What is the main idea of this reading?

A. Work among Humans and Animals.

B. The Accumulation of Wealth.

C. Sloth: One of the Seven Deadly Sins.

D. Human Cultural Training.

Câu hỏi 1389 :

According to the author, humans are so industrious because ___________.

A. they have an inborn temptation to take it easy so that they can survive.

B. they are stronger and better protected than animals, so they can easily gain more to survive.

C. they need much more than animals need in order to survive.

D. they can overcome the impulse to slow down and they work for gains beyond survival.

Câu hỏi 1390 :

The word “They” in paragraph 2 refers to ___________.

A. impulses

B. animals

C. resources

D. people

Câu hỏi 1391 :

The word “override” in paragraph 2 is closest in meaning to ___________.

A. ignore

B. count

C. consider

D. rank

Câu hỏi 1392 :

The passage is a combination of contrast and ________________.

A. steps in a process of working for human survival.

B. definitions and examples of human and animals’ work.

C. a series of events in human and animals’ life.

D. reasons for human’s diligence and acquisitiveness.

Câu hỏi 1393 :

Read the following passage and mark the letter A, B, C, or D on your answer sheet to indicate the correct answer to each of the questions from 36 to 42.

     What picture do you have of the future? Will life in the future be better, worse or the same as now? What do you hope about the future?

      Futurologists predict that life will probably be very different in 2050 in all the fields, from entertainment to technology. First of all, it seems that TV channels will have disappeared. Instead, people will choose a program from a 'menu' and a computer will send the program directly to the television.

Today, we can use the World Wide Web to read newspaper stories and see pictures on a computer thousands of kilometers away. By 2050, music, films, programs, newspapers and books will come to us by computer.

      In what concerns the environment, water will have become one of our most serious problems. In many places, agriculture is changing and farmers are growing fruit and vegetables to export.  This uses a lot of water. Demand for water will increase ten times between now and 2050 and there could be serious shortages. Some futurologists predict that water could be the cause of war if we don't act now.

      In transport, cars running on new, clean fuels will have computers to control the speed and there won't be any accidents. Today, many cars have computers that tell drivers exactly where they are. By 2050, the computer will control the car and drive it to your destination. On the other hand, space planes will take people halfway around the world in two hours. Nowadays, the United States Shuttle can go into space and land on Earth again. By 2050, space planes will fly all over the world and people will fly from Los Angeles to Tokyo in just two hours.

      In the domain of technology, robots will have replaced people in factories. Many factories already use robots. Big companies prefer robots - they do not ask for pay rises or go on strike, and they work 24 hours a day. By 2050, we will see robots everywhere - in factories, schools, offices, hospitals, shops and homes.

       Last but not least, medicine technology will have conquered many diseases. Today, there are electronic devices that connect directly to the brain to help people hear. By 2050, we will be able to help blind and deaf people see and hear again. Scientists have discovered how to control genes. They have already produced clones of animals. By 2050, scientists will be able to produce clones of people and decide how they look, how they behave and how much intelligence they have.

The passage mainly discusses ______________.

A. the effect of the futurologists' prediction on our lives.

B. the effect of telecommunication on our future life.

C. the changes in our future life compared with our life today.

D. the importance of cloning in the future.

Câu hỏi 1394 :

What can be inferred about the life in 2050 according to the passage?

A. Life in 2050 will be much better than that of today.

B. TV will be an indispensable means of communication and business.

C. People will not suffer from the shortage of water due to the polar melting.

D. The deaf will not have to depend any longer on the electronic hearing devices.

Câu hỏi 1395 :

Why does the author use "prefer robots" in paragraph 5?

A. To show the importance of robots in production.

B. To encourage the workers to resign from work.

C. To compare the robots with the workers.

D. To emphasize the scientist's role in inventing robots.

Câu hỏi 1396 :

The word "This" in paragraph 3 refers to __________________.

A. the possible war.

B. the cash crop planting.

C. the serious shortage.

D. the demand for water.

Câu hỏi 1397 :

What may NOT be true about life in 2050?

A. Many forms of entertainment will come to us thanks to computer.

B. Blind and deaf people will be helped by scientists to see and hear again.

C. Scientists will be able to produce clones of people and decide how intelligent they are.

D. Our sources of water for agriculture will increase ten times.

Câu hỏi 1398 :

The word “domain” in paragraph 5 mostly means ___________.

A. way of life

B. interest

C. challenge

D. field of expertise

Câu hỏi 1399 :

The word “conquered” in the last paragraph mostly means ___________.

A. controlled

B. diagnosed

C. transmitted

D. caused

Câu hỏi 1400 :

Mark the letter A, B, C, or D on your answer sheet to indicate the underlined part that needs correction in each of the following questions.
In 1619, the first African slaves arrived in Virginia, USA, and by 1790, their numbers reached nearly 700,000.


 


A.    the                                                                                


  

B. arrived

C. their

D. reached

Câu hỏi 1403 :

Mark the letter A, B, C, or D on your answer sheet to indicate the sentence that is closest in meaning to each of the following questions.

The last time I went to the gym was five months ago.

A. It’s five months since I last went to the gym.

B. I haven’t gone to the gym for five months ago.

C. I last went to the gym for 5 months.

D. I started going to the gym five months ago.

Câu hỏi 1404 :

“It’s very kind of you to help me with my project.”, said Andrea.


A. Andrea thanked me for helping her with her project.

B. Andrea wanted me to help her with her project

C. Andrea denied helping me with my project.

D. Andrea blamed me for not helping her with her project.

Câu hỏi 1405 :

It was impossible for him to commit the crime, according to the report.

A. He must have committed the crime, according to the report.

B. He needn’t have committed the crime, according to the report.

C. He can’t have committed the crime, according to the report.

D. He shouldn’t have committed the crime, according to the report.

Câu hỏi 1406 :

Mark the letter A, B, C, or D on your answer sheet to indicate the sentence that best combines each pair of sentences in the following questions.

The manager didn’t know about the change of the schedule. He was late for the meeting yesterday.

A. The manager wishes he had known about the change of the schedule so that he wasnn’t late for the meeting yesterday.

B. If the manager hadn’t been late for the meeting yesterday, he would have known about the change of the schedule.

C. Without knowing about the change of the schedule, the manager wouldn’t be late for the meeting yesterday.

D. If only the manager knew about the change of the schedule and came the meeting on time yesterday.

Câu hỏi 1407 :

The “chasing cheese” race this year is becoming a farce. Many people decide not to participate.

A. Were the “chasing cheese” race this year not a farce, many people wouldn’t decide to participate.

B. It is only when the “chasing cheese” race this year is becoming a farce that many people decide not to participate.

C. Although the “chasing cheese” race this year is becoming a farce, many people decide to participate.

D. Such is the “chasing cheese” race this year becoming a farce that many people decide not to participate.

Câu hỏi 1413 :

This house _______ in 1970 by my grandfather.

A built

B. was built

C. was build

D. has built

Câu hỏi 1414 :

John would like to specialize _______ computer science.

A. of

B. to

C. in

D. at

Câu hỏi 1415 :

The more I tried my best to help her, ________  she became.

A. less lazy

B. the lazier

C. the more lazy

D. lazier

Câu hỏi 1416 :

She has just bought a/ an _______ painting.

A. interesting French old

B. old interesting French

C. French interesting old

D. interesting old French

Câu hỏi 1417 :

When the boss walked into the office, his secretary ________.

A.has been typing

B.was typing

C. is typing

D. had typed

Câu hỏi 1418 :

He was offered the job_______ his qualifications were poor.

A. despite

B. in spite of

C. even though

D. Whereas

Câu hỏi 1419 :

________ to help, we will have finished  the work.

A. By the time John comes

B. Since John comes

C. When John comes

D. Until John comes

Câu hỏi 1420 :

_______ the distance was too far and the time was short, we decided to fly there instead of going there by train.

A. To discover

B. Discovered

C. To have discovered

D. Discovering

Câu hỏi 1421 :

We haven’t reached the final _______ on the funding for scientific research yet.

A. decides

B. decision

C. deciding

D. decisive

Câu hỏi 1422 :

Please help me with this math problem. I can’t_______ the answer.

A. end up

B. face up to

C. come up with

D. run into

Câu hỏi 1423 :

Peter_______ a better understanding of Algebra than we do.

A. makes

B. has

C. takes

D. gives

Câu hỏi 1424 :

The committee is _______ of well-known mountaineers.

A. contained

B. comprised

C. included

D. consisted

Câu hỏi 1425 :

This ticket _______ you to a free meal in our new restaurant.

A. allows

B. grants

C. entitles

D. credits

Câu hỏi 1426 :

We were all in___________of the fact that the new manager was our oldfriend Duncan.

A. surprise

B. shock

C. awe

D. amazement

Câu hỏi 1428 :

The factory is fined for discharging dangerous chemicals into the river.

A. releasing

B. increasing

C. decreasing

D. keeping

Câu hỏi 1430 :

It’s quite disappointing that some people still turn a blind eye to acts of injustice they witness in the street.

A. take no notice of

B. have no feeling for

C. show respect for

D. pay attention to

Câu hỏi 1432 :

Janet wants to invite Susan to go to the cinema.

     - Janet: “Do you feel like going to the cinema this evening?”

     - Susan: “_______.”


A. I don’t agree, I’m afraid


B. You’re welcome

C. That would be great

D. I feel very bored

Câu hỏi 1433 :

Read the following passage and mark the letter A, B, C, or D to indicate the correct word or phrase that best fits each the numbered blanks.

Wind is a clean source of renewable energy that produces no air or water pollution. And since the wind is free, operational costs are nearly zero (26) ______ a turbine is erected. Mass production and technology advances are making turbines cheaper, and (27) ______ governments offer tax incentives to spur wind-energy development.

Drawbacks include complaints from (28) ______ that wind turbines are ugly and noisy. The slowly rotating blades can also kill birds and bats, but not nearly as many as cars, power lines, and high-rise buildings do. The wind is also variable: If it's not blowing, there's no electricity generated.

Nevertheless, the wind energy industry is (29) ______. Thanks to global efforts to combat climate change, such as the Paris Agreement, renewable energy is seeing a boom in growth, in (30) ______ wind energy has led the way. From 2000 to 2015, cumulative wind capacity around the world increased from 17,000 megawatts to more than 430,000 megawatts. In 2015, China also surpassed the EU in the number of installed wind turbines and continues to lead installation efforts.

                                                                             (Adapted from https://www.nationalgeographic.com/)


A. so that


B. though

C. therefore

D. once

Câu hỏi 1435 :

Drawbacks include complaints from (28) ______ that wind turbines are ugly and noisy.

A. foreigners

B. masters

C. locals

D. levels

Câu hỏi 1436 :

Nevertheless, the wind energy industry is (29) ______.

A. worrying

B. booming

C. informing

D. relating

Câu hỏi 1438 :

Read the following passage and mark the letter A, B, C, or D to indicate the answer to each of the question.

There was a man who had four sons. He wanted his sons to learn not to judge things too quickly. So he sent them each on a quest, in turn, to go and look at a pear tree that was a great distance away. The first son went in the winter, the second in the spring, the third in summer, and the youngest son in the fall. When they had all gone and come back, he called them together to describe what they had seen.

The first son said that the tree was ugly, bent, and twisted. The second son said no – it was covered with green buds and full of promise. The third son disagreed, he said it was laden with blossoms that smelled so sweet and looked so beautiful, it was the most graceful thing he had ever seen. The last son disagreed with all of them; he said it was ripe and drooping with fruit, full of life and fulfilment.

The man then explained to his sons that they were all right, because they had each seen but one season in the tree’s life. He told them that you cannot judge a tree, or a person, by only one season, and that the essence of whom they are – and the pleasure, joy, and love that come from that life – can only be measured at the end, when all the seasons are up. If you give up when it’s winter, you will miss the promise of your spring, the beauty of your summer, fulfilment of your fall.

Don’t judge a life by one difficult season. Don’t let the pain of one season destroy the joy of all the rest.

(source: https://www.beliefnet.com)

Which best serves as the title for the passage?


A. The Seasons of Life


B. The Observation of a Tree

C. Father and Four Sons

D. Love all the Seasons in a Year

Câu hỏi 1439 :

According to the paragraph 2, what did the second son see in his turn?

A. The tree was gloomy, withered and crooked.

B. The tree was in buds and teeming with vigor.

C. The tree was blossoming and gave off a sweet scent.

D. The tree was bountifully fruitful, brimming with life force.

Câu hỏi 1440 :

The word “laden” in paragraph 2 is closest in meaning to _______.

A. loaded

B. decorated

C. enhanced

D. given

Câu hỏi 1441 :

The word “they” in paragraph 3 refers to _______.

A. the four sons

B. green buds

C. trees, people

D. the pleasure, joy and love

Câu hỏi 1442 :

According to the paragraph 4, what is the lesson the father wanted to impart to his children?

A. Moral lessons can come from the most unexpected and ordinary things.

B. No matter what season it is outside, you always have to cherish it.

C. The old age of humans is similar to the winter of nature.

D. Persevere through the difficulties and better times are sure to come sometime sooner or later.

Câu hỏi 1443 :

Read the following passage and mark the letter A, B, C, or D to indicate the answer to each of the question.

More than 200 reindeer have died of starvation on the Norwegian archipelago of Svalbard, with scientists blaming their deaths on climate change. The wild deer carcasses were found on the Arctic islands this summer by researchers from the Norwegian Polar Institute (NPI), which said it had never logged so many deaths at once in 40 years of monitoring the animals’ population level. “It’s scary to find so many dead animals,” project leader Ashild Onvik Pedersen told state broadcaster NRK. “This is an example of how climate change affects nature. It is just sad.”

Svalbard’s capital Longyearbyen, the northernmost town on earth, is thought to be warming quicker than any other settlement on the planet, climate scientists warned earlier this year. The milder temperatures in the region led to unusually heavy rainfall in December, leaving a thick layer of ice when the precipitation froze. This meant the reindeer could not dig through the hardened tundra to reach the vegetation they graze on in their usual pastures, the NPI said. Svalbard’s reindeer have been observed eating seaweed and kelp when food is scarce, but these are less nutritious and cause them stomach problems.

A relatively high number of calves born last year increased the death toll, as the youngest and weakest are often the first to die in harsh conditions. “Some of the mortality is natural because there were so many calves last year. But the large number we see now is due to heavy rain, which is due to global warming,” said Ms Onvik Pedersen.

A team of three scientists spent 10 weeks investigating population of the Svalbard reindeer earlier this year. Researchers warned the decline of reindeer would cause unwanted plant species, currently kept in check by the animals’ grazing, to spread across Arctic ecosystems in Europe, Asia and North America.

Arctic reindeer and caribou populations have declined 56 per cent in the last two decades, a report by the National Oceanic and Atmospheric Administration said last year. The report said food security was partly to blame for falling herd numbers, while warmer summers could also put the animals at greater risk of diseases spread by flies and parasites. The average temperature in Longyearbyen has risen by 3.7C since 1900, more than three times the global average increase of about 1C. In 2016, the entrance to the town’s “Doomsday” seed vault – which stores specimens of almost all the world’s seeds – was flooded following heavy rainfall.

(Adapted from https://www.independent.co.uk/)

Which could best serve as the title of the passage?


A. Climate change – The main cause for the death of hundreds of reindeer.


B. Global warming – What are the effects on nature?

C. Reindeer – The most vulnerable animals on the Arctic islands.

D. Climate change – What are the reasons?

Câu hỏi 1444 :

The word “logged” in paragraph 1 is closest in meaning to _______.

A. cut down

B. damaged

C. recorded

D. discovered

Câu hỏi 1445 :

The following are true about capital Longyearbyen, EXCEPT _______.

A. It is believed to be the most quickly warming settlement on earth.

B. People in Longyearbyen suffered unusually heavy rain at the end of the year.

C. It is the northernmost town on our planet.

D. The reindeer here couldn’t stand the low temperature when the precipitation froze.

Câu hỏi 1446 :

The word “scarce” in paragraph 2 could be best replaced by _______.

A. inappropriate

B. insufficient

C. abundant

D. unlimited

Câu hỏi 1447 :

What does the word “these” in paragraph 2 refer to?

A. seaweed and kelp

B. Svalbard’s reindeer

C. their usual pastures

D. milder temperatures

Câu hỏi 1448 :

According to Ms. Onvik Pedersen, why is the death rate of reindeer so high this year?

A. Because of the high number of calves born.

B. Because of heavy rain.

C. Because of natural selection.

D. Because of the shortage of vegetation.

Câu hỏi 1449 :

It can be inferred from the passage that _______.

A. Arctic reindeer play the most important role in the Arctic ecosystems.

B. the Arctic ecosystems are altering worse because of the global warming.

C. the clearest effect of climate change is the limit of food chain in the nature.

D. the harsh weather in Arctic islands only damages the new-born calves.

Câu hỏi 1451 :

Her weight has increased remarkably since they began receiving treatment.

A. Her weight

B. remarkably

C. they

D. receiving

Câu hỏi 1452 :

The sign says that we should read the constructions carefully before proceeding.

A. says

B. should

C. the constructions

D. proceeding

Câu hỏi 1453 :

Mark the letter A, B, C, or D to indicate the sentence that is closest in meaning to each of the following questions.

She began to play the piano three years ago.


A. She has played the piano since three years.


B. She has played the piano for three years

C. She doesn’t play the piano now.

D. She stops playing the piano now.

Câu hỏi 1454 :

"Don't forget to submit your assignments by Thursday," said the teacher to the students.

A. The teacher reminded the students to submit their assignments by Thursday.

B. The teacher allowed the students to submit their assignments by Thursday.

C. The teacher ordered the students to submit their assignments by Thursday.

D. The teacher encouraged the students to submit their assignments by Thursday.

Câu hỏi 1455 :

It’s very likely that the company will accept his application.

A. The company needs accept his application.

B. The company might accept his application.

C. The company must accept his application.

D. The company should accept his application.

Câu hỏi 1456 :

Mark the letter A, B, C, or D to indicate the sentence that best combines each pair of sentences in the following questions.

Marie didn’t turn up at John’s birthday party. I feel so sorry for that.


A. If only Marie turn up at John’s birthday party.


B. I wish Marie had turned up at John’s birthday party.

C. I wished Marie wouldn’t turn up at John’s birthday party.

D. It’s a shame Marie had turned up at John’s birthday party.

Câu hỏi 1457 :

The government does not know what to do with household rubbish in large cities.

A. Little does the government know what to do with household rubbish in large cities.

B. It is unknown what to do with household rubbish in large cities by the government.

C. Rarely the government knows what to do with household rubbish in large cities.

D. Hardly any government knows what to do with household rubbish in large cities.

Câu hỏi 1463 :

This house is going _______ by my mother

A. sold

B. to be sold

C. to sold

D. to sell

Câu hỏi 1464 :

Today, women are increasingly involved _______ the politics.


A. of


B. in

C. with

D. from

Câu hỏi 1465 :

_______ he worked, the more he earned.

A. The more hard

B. The hard

C. The harder

D. The hardest

Câu hỏi 1466 :

Indiana University, one of the largest in the nation, is located in a _______ town.

A. small beautiful Midwestern

B. beautiful Midwestern small

C. Midwestern beautiful small

D. beautiful small Midwestern

Câu hỏi 1467 :

When I entered the room, everyone _______.

A. has been dancing

B. was dancing

C. had danced

D. danced

Câu hỏi 1468 :

Children are encouraged to read books _______ they are a wonderful source of knowledge.

A. in spite of

B. although

C. because of

D. because

Câu hỏi 1469 :

By the time you finish cooking they _______ their homework.

A. will have finished

B. will finish

C. had finished

D. have finished

Câu hỏi 1471 :

I think mobile phones are _______ for people of all ages.

A. usage

B. usefully

C. useful

D. use

Câu hỏi 1472 :

I _______  an old friend of mine in the street this morning. We haven't seen each other for ages.

A. ran into

B. ran out

C. came over

D. came round

Câu hỏi 1474 :

His poor standard of play fully justifies his _______ from the team for the next match.

A. expulsion

B. dismissal

C. rejection

D. exclusion

Câu hỏi 1475 :

She took a course in fine arts _______ startingher own business in interior design.

A. with a view to

B. in terms of

C. in order to

D. with reterence to

Câu hỏi 1476 :

Like everyone else, Sue has her _______ of course but on the whole, she's quite satisfied with life.

A. ups and downs

B. ins and outs

C. safe and sound

D. odds and ends

Câu hỏi 1482 :

Claudia is being interviewed by the manager of the company she's applied for.

     - Manager: “_______”

     - Claudia: "I work hard and I enjoy working with other people."


A. Can you do jobs on your own?


B. Would you describe yourself as ambitious?

C. What are some of your main strengths?

D. Why have you applied for this position?

Câu hỏi 1483 :

Read the following passage and mark the letter A, B, C, or D to indicate the correct word or phrase that best fits each the numbered blanks.

The first thing that is included in the "living together” (26) _______ is the expected good relations with your family. This also involves sharing equally the housework. (27) _______ of people think that everyone should share the housework equally, but in many homes parents do most of it. To certain minds, many families can't share the housework whereas they should try it. In fact, sharing the housework equally is not very possible because of the families' timetable. So, it is somehow believed that children and parents must do things together. For this they can establish a housework planning.

(28) _______, housework's contributions of the teenager make him more responsible. He will think that he has an important role in his family. According to researchers, teenagers should share the housework because (29) _______ will help them when they have to establish their own family in the future. Too many teenagers and young adults leave home without knowing how to cook or clean, but if parents delegate basic housework to teens as they are old enough to do it, they won't be destabilized by doing the housework in their new grown-up life.

It can be (30) _______ concluded that many parents don't really prepare their children for future, because they don't stimulate them to learn how to run a house. If parents get them responsible, teens will be more responsible and that will improve family's life.

                                                                    (Source: https://www.sciencedaily.com/releases/2013)


A. custom


B. tradition

C. notion

D. trend

Câu hỏi 1485 :

(28) _______, housework's contributions of the teenager make him more responsible.


A. In addition


B. However

C. In contrast

D. In case

Câu hỏi 1488 :

Read the following passage and mark the letter A, B, C, or D to indicate the answer to each of the question.

CDC strives to safeguard the health and improve the quality of life of all people with diabetes. Central to that effort is helping them prevent or reduce the severity of diabetes complications, including heart disease (the leading cause of early death among people with diabetes), kidney disease, blindness, and nerve damage that can lead to lower-limb amputations.

Diabetes self-management education and support (DSMES) programs help people meet the challenges of self-care by providing them with the knowledge and skills to deal with daily diabetes management: eating healthy food, being active, checking their blood sugar, and managing stress. These programs have been shown to reduce A1C levels (average blood sugar over the last 2 to 3 months), reduce the onset and severity of diabetes complications, improve quality of life, and lower health care costs.

Diabetes is about 17% more prevalent in rural areas than urban ones, but 62% of rural counties do not have a DSMES program. The use of telehealth (delivery of the program by phone, Internet, or videoconference) may allow more patients in rural areas to benefit from DSMES and the National DPP lifestyle change program. CDC funds state and local health departments to improve access to, participation in, and health benefit coverage for DSMES, with emphasis on programs that achieve American Association of Diabetes Educators accreditation or American Diabetes Association recognition. These programs meet national quality standards and may be more sustainable because of reimbursement eligibility.

                                                                                                     (Adapted from https://www.cdc.gov/)

Which best serves as the title for the passage?


A. Diabetes Complications and CDC’s Response


B. Diabetes self-management education and support programs

C. Knowledge of A1C levels

D. American Association of Diabetes Educators

Câu hỏi 1489 :

The word “them” in paragraph 2 refers to ______.

A. programs

B. people

C. challenges

D. skills

Câu hỏi 1490 :

Which is NOT mentioned in paragraph 2 as a skill to deal with daily diabetes management?

A. eating healthy food

B. checking blood sugar

C. working effectively

D. managing stress

Câu hỏi 1491 :

The word “lower” in paragraph 2 is closest in meaning to _______.

A. borrow

B. prepare

C. approach

D. reduce

Câu hỏi 1493 :

Read the following passage and mark the letter A, B, C, or D to indicate the answer to each of the question.

There are many mistakes that people make when writing their resume (CV) or completing a job application. Here are some of the most common and most serious.

The biggest problem is perhaps listing the duties for which you were responsible in a past position: all this tells your potential employers is what you were supposed to do. They do not necessarily know the specific skills you used in executing them, nor do they know what results you achieved - both of which are essential. In short, they won’t know if you were the best, the worst or just average in your position.

The more concrete information you can include, the better. As far as possible, provide measurements of what you accomplished. If any innovations you introduced saved the organization money, how much did they save? If you found a way of increasing productivity, by what percentage did you increase it?

Writing what you are trying to achieve in life - your objective - is a waste of space. It tells the employer what you are interested in. Do you really think that employers care what you want? No, they are interested in what they want! Instead, use that space for a career summary. A good one is brief - three to four sentences long. A good one will make the person reviewing your application want to read further.

Many resumes list ‘hard’ job-specific skills, almost to the exclusion of transferable, or ‘soft’, skills. However, your ability to negotiate effectively, for example, can be just as important as your technical skills.

All information you give should be relevant, so carefully consider the job for which you are applying. If you are applying for a job that is somewhat different than your current job, it is up to you to draw a connection for the resume reviewer, so that they will understand how your skills will fit in their organization. The person who reviews your paperwork will not be a mind reader.

If you are modest about the skills you can offer, or the results you have achieved, a resume reader may take what you write literally, and be left with a low opinion of your ability: you need to say exactly how good you are. On the other hand, of course, never stretch the truth or lie.

                                                                                                              (Source: www.ielts-mentor.com)

What topic does the passage mainly discuss?


A. The way how to write the resume for job application.


B. The mistakes people make when applying for a job.

C. The common way to make impression in a job interview.

D. The necessary skills for job application.

Câu hỏi 1494 :

The word “executing” in paragraph 2 is closest in meaning to _______.

A. enumerating

B. determining

C. completing

D. implementing

Câu hỏi 1495 :

The word “concrete” in paragraph 3 could be best replaced by _______.

A. indeterminate

B. specific

C. substantial

D. important

Câu hỏi 1496 :

What does the word “it” in paragraph 3 refer to?

A. organization money

B. information

C. productivity

D. percentage

Câu hỏi 1497 :

According to the passage, what information should candidates include in their resume?

A. specific skills for previous jobs

B. the past achievements

C. previous positions

D. future objective

Câu hỏi 1498 :

According to the passage, which of the following is NOT true?

A. The ability to negotiate effectively is as significant as technical skills.

B. Candidates must study the job they are applying carefully before writing the CV.

C. Applicants should not apply for a distinct job from what they are doing.

D. The information interviewees present should be related to the job they are applying.

Câu hỏi 1499 :

It can be inferred from the last paragraph that ______.

A. you should write accurately about your ability for the vacant position.

B. you should be modest about what you can do.

C. a resume reader is good enough to understand what you imply about your ability in the CV.

D. you are allowed to exaggerate the truth of your competence if possible.

Câu hỏi 1502 :

A public spending programme is out of the question in the current economical climate.


A. spending programme


B. out of

C. the question

D. economical climate

Câu hỏi 1503 :

Mark the letter A, B, C, or D to indicate the sentence that is closest in meaning to each of the following questions.

The last time I saw David was when I ran into him at the station on my way to New York.


A. I haven’t seen David since I ran into him at the station on my way to New York.


B. When I last went to New York, I happened to meet David at the station.

C. When I last saw David at the station in New York, I ran after him.

D. I finally saw David at the station when I was on my way to New York.

Câu hỏi 1504 :

Conan said to me, “If I were you, I would read different types of books in different ways.”

A. Conan ordered me to read different types of books in different ways.

B. I said to Conan to read different types of books in different ways to me.

C. I read different types of books in different ways to Conan as he told me.

D. Conan advised me to read different types of books in different ways.

Câu hỏi 1505 :

Every student is required to write an essay on the topic.

A. Every student might write an essay on the topic.

B. Every student must write an essay on the topic.

C. They require every student can write an essay on the topic.

D. Every student should write an essay on the topic.

Câu hỏi 1506 :

Mark the letter A, B, C, or D to indicate the sentence that best combines each pair of sentences in the following questions.

They didn’t have breakfast this morning.That’s why they are hungry now.


A. They wish they had had breakfast this morning


B. They wish they had breakfast this morning

C. They wish they have breakfast this morning

D. They wish they would have breakfast this morning

Câu hỏi 1507 :

He was very surprised to be addressed by the Queen. He didn’t answer at once.

A. Not until he was so surprised to answer did the Queen address him.

B. Only when he was addressed by the Queen could he answer the surprising questions at once.

C. So surprised was he to be addressed by the Queen that he didn’t answer at once.

D. But for such a nice surprise, he would have been addressed by the Queen.

Câu hỏi 1512 :

The road to our village _______ next year.

A. is widened

B. will widen

C. can widen

D. will be widened

Câu hỏi 1514 :

Try to keep calm _______ the sake of your children.

A. with

B. for

C. by

D. in

Câu hỏi 1515 :

The more he slept, _______ irritable he became.

A. the most

B. the vey more

C. much more

D. the more

Câu hỏi 1516 :

My aunt gave me a _______ cotton hat on my 16th birthday.

A. nice yellow new

B. new nice yellow

C. yellow new nice

D. nice new yellow

Câu hỏi 1517 :

When I saw Lisa at the museum, I _______ to restaurant.

A.  was going

B.  went

C.  have gone

D.  were going

Câu hỏi 1518 :

_______ his poor English, he managed to communicate his problem very clearly.

A. Because

B. Even though

C. Because of

D. In spite of

Câu hỏi 1519 :

_______ , his employees will have worked for two hours.

A. By the time the boss arrives

B. When the boss arrives

C. After the boss arrives

D. Only when the boss arrives

Câu hỏi 1522 :

The government has _______ new proposals to tackle the problem of increasing crime.

A. brought over

B. brought through

C. brought round

D. brought forward

Câu hỏi 1523 :

She made a big _______ about not having a window seat on the plane.

A. fuss

B. complaint

C. interest

D. excitement

Câu hỏi 1524 :

The _______ prices of property in big cities may deter people on low incomes from owning a house there.

A. competitive

B. forbidding

C. prohibitive

D. inflatable

Câu hỏi 1526 :

When someone is down on their _______, friends are not easy to find.

A. mood

B. luck

C. fortune

D. merit

Câu hỏi 1528 :

Before the advent of the railways, communications were slow and difficult.

A. import

B. disappearance

C. introduction

D. arrival

Câu hỏi 1530 :

She’s a bit down in the dumps because she’s got to take her exam again.

A. sad

B. embarrassed

C. confident

D. happy

Câu hỏi 1532 :

Two friends are talking to each other at a class reunion.

     - Jimmy: “_______

     - Jack: "I work at the bank."


A. How do you do, Jack?


B. What do you do for a living Jack?

C. How are you doing, Jack?

D. Where did you work, Jack?

Câu hỏi 1533 :

Read the following passage and mark the letter A, B, C, or D to indicate the correct word or phrase that best fits each the numbered blanks.

        The "greenhouse effect" is the warming that happens when certain gases in Earth's atmosphere (26) _______ heat. These gases let in light but keep heat from escaping, like the glass walls of a greenhouse. First, sunlight shines onto the Earth's surface, (27) ________  it is absorbed and then radiates back into the atmosphere as heat. In the atmosphere, “greenhouse gases trap some of this heat, and the the rest escapes into space. The more greenhouse gases are in the atmosphere, the more heat gets trapped.

        Scientists have known about the greenhouse effect since 1824, when Joseph Fourier calculated that the Earth would be much colder if it had no atmosphere. This greenhouse effect is what keeps the Earth's climate (28) _______. Without it, the Earth's surface would be an average of about 60 degrees Fahrenheit cooler. Scientists often use the term "climate change" instead of global warming. This is because as the Earth's average temperature climbs, winds and ocean currents move heat around the globe in ways that can cool some areas, warm (29) _______, and change the amount of rain and snow falling. (30) _______, the climate changes differently in different areas.

                                                           (Source: https://www.open.edu/openlearncreate/mod/oucontent)


A. seize


B. capture

C. trap

D. grasp

Câu hỏi 1535 :

This greenhouse effect is what keeps the Earth's climate (28) _______.

A. lively

B. alive

C. livable

D. living

Câu hỏi 1537 :

(30) _______, the climate changes differently in different areas.

A. However

B. In addition 

C. On the contrary

D. As a result

Câu hỏi 1538 :

Read the following passage and mark the letter A, B, C, or D to indicate the answer to each of the question.

The relationship between Britain and the US has always been a close one. Like all close relationships it has had difficult times. The US was first a British colony, but between 1775 and 1783 the US fought a war to become independent. The US fought the British again in the War of 1812.

 In general, however, the two countries have felt closer to each other than to any other country, and their foreign policies have shown this. During World War I and World War II, Britain and the US supported each other. When the US looks for foreign support, Britain is usually the first country to come forward and it is sometimes called “the 51st state of the union”.

 But the special relationship that developed after 1945 is not explained only by shared political interests. An important reason for the friendship is that the people of the two countries are very similar. They share the same language and enjoy each other's literature, films and television. Many Americans have British ancestors, or relatives still living in Britain. The US government and political system is based on Britain's, and there are many Anglo-American businesses operating on both sides of the Atlantic. In Britain some people are worried about the extent of US influence, and there is some jealousy of its current power. The special relationship was strong in the early 1980s when Margaret Thatcher was Prime Minister in Britain and Ronald Reagan was President of the US.

                                                                  (Adapted from Background to British and American Cultures)

What is the passage mainly about?


A. The strong friendship between the UK and the US.


B. The close relationship between Britain and the US.

C. A special relationship the UK developed during the World Wars.

D. A special influence the US had on the UK during the World Wars.

Câu hỏi 1539 :

The word “They” in paragraph 3 prefers to _______.

A. countries

B. people

C. political interests

D. British ancestors

Câu hỏi 1540 :

The phrase "come forward” in paragraph 2 mostly means _______.

A. be willing to help

B. be able to help

C. be reluctant to help

D. be eager to help

Câu hỏi 1541 :

Which of the following is NOT mentioned as a reason for the special relationship between Britain and the US?

A. The people of the two countries are very similar.

B. Many Americans have British ancestors.

C. British Prime Minister and the US President are close friends.

D. Many Anglo-American businesses are operating in the two countries.

Câu hỏi 1542 :

Britain and the US are close to each other NOT because of their _______.

A. foreign policies

B. power

C. political interests

D. language

Câu hỏi 1543 :

Read the following passage and mark the letter A, B, C, or D to indicate the answer to each of the question.

Three scales of temperature, each of which permits a precise measurement, are in current use: the Fahrenheit, Celsius, and Kelvin scales. These three different temperature scales were each developed by different people and have come to be used in different situations.

The scale that is most widely used by the general public in the United States is the Fahrenheit scale. In 1714, Daniel Gabriel Fahrenheit, a German physicist who was living in Holland and operating an instrument business, developed a thermometer and the temperature scale that still carries his name. His original scale had two fixed points: 0º was the lowest temperature and 96º was what he believed was the normal temperature of the human body. Based on this scale, he calculated that the freezing point of water was 32º; in later studies, it was determined that the boiling point of water was 212º. The Fahrenheit scale came to be accepted as the standard measure of temperature in a number of countries. Today, however, the United States is the only major country in the world that still uses the Fahrenheit scale.

The scale that is in use in many other countries is the Celsius scale. Anders Celsius, a Swedish astronomer, developed a thermometer in 1741 that based temperatures on the freezing and boiling temperatures of water. On the thermometer that Celsius developed, however, 0º was used to indicate the boiling temperature of water, and 100º was used to indicate the freezing temperature of water. After his death, the scale was reversed by a friend, the biologist Carl von Linne. On the new scale after the reversal by von Linne, 0º indicated the freezing temperature of water, and 100º indicated the boiling temperature of water. At around the same time, a similar thermometer was being developed in France. After the French Revolution, the scale developed in France was adopted as part of the metric system in that country under the name centigrade, which means "a hundred units," and from there it spread worldwide. In 1948, an international agreement was made to rename the centigrade scale the Celsius scale in honor of the scientist who was first known to use a 100-degree scale, though it should be remembered that the scale that Celsius actually used himself was the reverse of today's scale.

A third scale, the Kelvin scale, is generally used today for scientific purposes. This scale was first suggested in 1854 by two English physicists: William Thomson, Lord Kelvin and James Prescott Joule. The Kelvin scale defines 0º as absolute zero, the temperature at which all atomic and molecular motion theoretically stops, and 100º separates the freezing point and boiling point of water, just as it does on the Celsius scale. On the Kelvin scale, with 0º equal to absolute zero, water freezes at 273º, and water boils at a temperature 100º higher. The Kelvin scale is well suited to some areas of scientific study because it does not have any negative values, yet it still maintains the 100º difference between the freezing point and boiling point of water that the Celsius scale has and can thus easily be converted to the Celsius scale by merely subtracting 273º from the temperature on the Kelvin scale.

                                                                (Adapted from TOEFL Reading Practice by Deborah Philips)

What is the main idea of the passage?


A. Temperature scales in use today.


B. Comparison of temperature scales.

C. The origin of thermometers.

D. The most common temperature scale.

Câu hỏi 1544 :

The word fixed in paragraph 2 could best be replaced by _______.

A. ordered

B. repaired

C. established

D. attached

Câu hỏi 1545 :

According to the passage, a temperature of 50º centigrade would be equal to _______.

A. 223º on the Kelvin scale

B. 273º on the Kelvin scale

C. 323º on the Kelvin scale

D. 373º on the Kelvin scale

Câu hỏi 1546 :

The word adopted in paragraph 3 could best be replaced by _______.

A. taken on

B. brought up

C. looked upon

D. turned down

Câu hỏi 1547 :

The word it in paragraph 4 refers to _______.

A. the Kevin scale

B. a negative value

C. a scientific research

D. a difference

Câu hỏi 1548 :

Which of the following is NOT true about the Celsius scale, according to the passage?

A. It was developed by an astronomer from Sweden.

B. It came into use in the eighteenth century.

C. One hundred degrees separated the freezing and boiling temperatures of water on it.

D. On it, 0º indicated the temperature at which water freezes.

Câu hỏi 1549 :

Which of the following can be inferred from the passage?

A. The Kelvin scale enjoys the largest popularity in the world today.

B. The Celsius scale in use was actually the original version.

C. In 1948, nations agreed to name the centigrade scale after the man who developed an earlier version of the scale.

D. Kevin, Fahrenheit and Celsius were working in cooperation to develop their own thermometers.

Câu hỏi 1551 :

They are having her house  painted by a construction company.

A. having

B. her

C. painted

D. construction

Câu hỏi 1553 :

Mark the letter A, B, C, or D to indicate the sentence that is closest in meaning to each of the following questions.

We have been cooking for the party for four hours.


A. We didn’t start cooking for the party until four.


B. We started cooking for the party four hours ago.

C. We have four cooks for the party

D. Cooking for the party will be done in four hours

Câu hỏi 1554 :

"Don't forget to submit your assignments by Thursday," said the teacher to the students.

A. The teacher reminded the students to submit their assignments by Thursday.

B. The teacher allowed the students to submit their assignments by Thursday.

C. The teacher ordered the students to submit their assignments by Thursday.

D. The teacher encouraged the students to submit their assignments by Thursday.

Câu hỏi 1555 :

I’m sure that they had practiced hard for the games as they won a lot of medals.

A. They couldn’t have practiced hard for the games as they won a lot of medals

B. They must have practiced hard for the games as they won a lot of medals

C. They shouldn’t have practiced hard for the games as they won a lot of medals

D. They might have practiced hard for the games as they won a lot of medals

Câu hỏi 1556 :

Mark the letter A, B, C, or D to indicate the sentence that best combines each pair of sentences in the following questions.

Nam was so rude to them last night. Now he feels regretful.


A. Nam regrets to have been so rude to them last night.


B. Nam regrets having so rude to them last night.

C. Nam wishes he hadn’t been so rude to them last night.

D. Nam wishes he weren’t so rude to them last night.

Câu hỏi 1557 :

Helen wrote a novel. He made a cowboy film, too

A. Helen wrote not only a novel but also made a cowboy film.

B. Helen both wrote a novel as well as made a cowboy film.

C. Helen either wrote a novel or made a cowboy film.

D. Not only did Helen write a novel but she also made a cowboy film.

Lời giải có ở chi tiết câu hỏi nhé! (click chuột vào câu hỏi).

Liên hệ hợp tác hoặc quảng cáo: gmail

Điều khoản dịch vụ

Copyright © 2021 HOCTAPSGK